Sunteți pe pagina 1din 84

Al V-lea Congres international

al matematicienilor romni
Pitesti, 22 - 28 iunie, 2003
Incepnd cu anul 1929, s-au organizat, pna n prezent, cinci congrese international ale matematicienilor romni.
Primul Congres a avut loc n anul 1929 la Cluj avnd ca promotor pe Petre
Sergescu, dar avnd concursul celor mai reputati matematicieni romni din acea
perioada.
Al II-lea Congres a avut loc la Turnu Severin n 1932, bucurndu-se ca si
primul de participarea unor mari matematicieni ai timpului, Paul Montel, Arnaud
Denjoy, Waclaw Sierpinski si altii.
Al III-lea Congres s-a desfasurat la Bucuresti n 1945, cu o participare modesta
a matematicienilor straini, date fiind conditiile dificile de la sfrsitul celui de-al doilea
r
azboi mondial.
Cel de-al IV-lea Congres a fost organizat tot la Bucuresti, cu o preg
atire speciala, n anul 1956. Statul romn a facut un efort special si prestigiul de care se
bucurau matematicienii romani (toti formati n marile scoli din occident) au permis
invitarea unui num
ar nsemnat de matematicieni str
aini, de faim
a international
a.
Din Franta au participat Jacques Hadamard si Arnaud Denjoy, din Japonia Masuo Hukuhara, din Germania - W. Blaschke, din Statele Unite - Einar Hille
si S. Eilenberg, din Uniunea Sovietica - I. Vekua, din Polonia - K. Kuratowski si
T. Wazewski. A fost un prilej de rentlnire a matematicienilor romni din generatia
lui Grigore Moisil, Gheorghe Vr
anceanu, Miron Nicolescu, Simion Stoilow,
Nicolae Teodorescu, Tiberiu Popoviciu cu fostii lor mentori sau colegi.
Dupa o ntrerupere de 47 de ani, s-a organizat cel de-al V-lea Congres la Universitatea din Pitesti, al c
arei Rector dr. Gheorghe Barbu este el nsusi matematician
(format la Iasi si Bucuresti). Domnia sa si-a asumat sarcinile dificile ale organiz
arii
congresului, eveniment organizat sub egida Academiei Romne, a Universitatii Bucuresti si a Institutului de Matematica "S. Stoilow" al Academiei Romne. Organizarea congresului a fost reusit
a, datorit
a n primul rnd comitetului local de organizare, autorit
atilor locale si sprijinului acordat de la Bucuresti. Ca o nou
a caracteristica a acestui congres, subliniem prezenta unui numar nsemnat de matematicieni
romni care-si desfasoara acum activitatea n tari straine (a se vedea Libertas Mathematica, vol. XXIII, n care se afl
a numele si adresele a peste 300 de matematicieni
romni ce detin catedre n universit
ati din str
ain
atate, pe toate continentele Terrei).
"Ziarul de Azi" din Pitesti, n timpul desfasurarii congresului, a publicat numeroase
relatari si informatii privind participarea unor reputati matematicieni straini, dar si
a multor matematicieni romni care au activat sau activeaz
a n alte tari. Printre cei
din ultima categorie vom aminti academicienii Nicolae Cristescu si Nicolae Dinculeanu, Sergiu Klainerman (Princeton), Dan Burghelea (Columbus - Ohio),
Daniel Tataru (Berkeley, CA), Henri Moscovici (Columbus - Ohio), M. Epstein
(Tel Aviv), Radu Theodorescu (Laval, Canada).
Spre deosebire de multe alte congrese sau conferinte cu participare international
a,
congresele internationale ale matematicienilor romni au fost ntotdeauna caracteri1

zate printr-o larg


a reprezentare a tuturor domeniilor de baz
a din cercetarea matematica. Astfel, cele peste 400 de comunicari anuntate pentru Congresul al V-lea,
au fost distribuite n 15 sectii, ncepnd cu Logica, Algebra si Teoria numerelor,
mergnd pn
a la Istoria si Filozofia matematicii si Pedagogia matematicii. Au fost
reprezentate Geometria, Analiza clasica si moderna, Ecuatiile diferentiale, Teoria
controlului optimal, Teoria probabilitatilor si Statistica matematica, Cercetarea operationala, Mecanica si Astronomia, Fizica matematica. Lucrarile s-au desfasurat att
n plenul congresului (ncepnd cu sedinta de deschidere la care Ambasadorul Frantei
la Bucuresti, E. S. Philippe tienne, el nsusi matematician si admirabil vorbitor,
a captivat audienta), precum si n numeroase sectii pe specialitati.
Pe lnga matematicienii straini care au participat la Congres, venind din Statele
Unite, Canada, Franta, Germania, Rusia, Ungaria, Italia si alte tari, trebuie s
a
remarc
am prezenta destul de nsemnat
a a matematicienilor din Republica Moldova.
Este destul de dificil sa prezentam o vedere de ansamblu asupra desfasurarii Congresului al V-lea al matematicienilor romni, data fiind varietatea domeniilor abordate de c
atre participanti. Vom sublinia totusi faptul c
a programul si desfasurarea
lucrarilor congresului s-au ncadrat n standardele internationale. O critic
a ce s-a
adus organizatorilor a fost aceea ca data congresului a coincis cu multiple activitati
academice, cum ar fi: examenele studentesti, examenul de licenta si altele. n felul
acesta, multi doritori din tar
a de a participa au fost absenti.
Vom ncheia subliniind faptul c
a acest al V-lea Congres a ilustrat vitalitatea
matematicii romnesti, ncadrarea ei reusita n comunitatea matematica internationala. S
a sper
am c
a urm
atorul congres va avea loc dup
a o perioad
a nu att de
ndelungat
a ca pn
a acum.
Constantin CORDUNEANU
University of Texas at Arlington

Observatorul din Iasi 90 de ani de la nfiintare


nfiintarea observatoarelor astronomice din Bucuresti (n 1908) si apoi din Iasi
(n 1913) face parte dintr-un proces mai amplu de modernizare a nv
atamntului
universitar si a cercet
arii stiintifice, proces impulsionat de Legea Haret din 1898 si
care se va maturiza n conditiile social-politice si culturale din Romnia ntregit
a.
nca din momentul nfiintarii n 1860 a Universit
atii din Iasi, n programa
"sectiei stiintelor pozitive din facultatea de filozofie" sunt prev
azute si cursuri de
mecanic
a si astronomie, dar catedrele aferente vor c
ap
ata fiinta mai trziu. Prin
legea nvatamntului din 1864, care se pune n aplicare ncepnd cu data de 25
febr. 1865, se creeaza Facultatea de stiinte, desprinsa din Facultatea de filozofie
si avnd trei sectii distincte: fizic
a, matematic
a si stiinte naturale; una din cele 12
catedre ale noii facult
ati este cea de geodezie teoretica si astronomie. La 15 febr. 1865
este numit profesor titular al acestei catedre Neculai Culianu, care o va ocupa
pn
a n 1906, anul pension
arii sale. N. Culianu trece licenta n stiinte matematice
la Sorbona, este atras de astronomie si de Observatorul din Paris, cunoaste aici
si r
amne prieten pentru toat
a viata cu astronomul francez Camille Flammarion.
N. Culianu este autor al unui Curs de cosmografie pentru liceu (doua editii, 1893 si
1902). Universitatea din Iasi a primit, chiar din momentul nfiintarii, de la Societatea
de Medici si Naturalisti din Iasi un bun instrument de observatii astronomice, care
apartinuse poetului moldovean Costache Conachi si pe care mostenitorii l donaser
a
acesteia.
Dup
a nfiintarea Catedrei de astronomie (n 1864) au fost achizitionate si alte
instrumente; ele au fost depozitate ntr-o c
am
aruta a vechiului local al universit
atii
nct nici nu puteau fi ar
atate studentilor. Cu toate insistentele nu s-a reusit timp
ndelungat obtinerea fondurilor pentru construirea unui observator astronomic.
Constantin Popovici este licentiat al Facult
atii de stiinte din Iasi (1900). Pleac
a
la Paris cu o burs
a "Adamachi" unde obtine din nou licenta n matematici (1905) si
apoi doctoratul la Sorbona (1908) n domeniul ecuatiilor diferentiale. n 1909 este
numit la Catedra de geometrie analitica a universitatii iesene, iar n 1910 este trimis
n Franta pentru specializare n astronomie si documentare n privinta construirii
viitorului observator din Iasi. Se rentoarce si este numit n 1911 la Catedra de
astronomie, geodezie si mecanica cereasca, Universitatea din Iasi.
C. Popovici este fondatorul Observatorului astronomic din Iasi, amplasat pe
dealul Copou; piatra de temelie a cl
adirii a fost pus
a la 12 sept. 1912, iar receptia
s-a f
acut la mijlocul lui decembrie 1913.
C. Popovici este primul director al Observatorului (n perioada 1913-1937). Primele instrumente intrate n dotarea acestuia au fost cele provenite de la cabinetul de
astronomie nfiintat de N. Culianu. Prin str
adaniile lui C. Popovici si ale elevului
si colaboratorului s
au, Vintil
a S
iadbei, au fost achizitionate noi instrumente: o
luneta meridiana, un ecuatorial Ressel, doua cronometre (pentru timpul mediu si cel
sideral), un fotometru Gra si altele necesare procesului didactic.
n anul 1938 Catedra de astronomie este transformat
a ntr-o conferinta iar
C. Popovici se transfer
a la Bucuresti. n perioada 1938-1944, Vintil
a S
iadbei a
suplinit conferinta de astronomie.
3

Ca urmare a evacu
arii Observatorului, prilejuit
a de cel de-al doilea r
azboi mondial,
o buna parte a aparaturii din dotarea acestuia s-a deteriorat sau a fost sustrasa.
n 1948 Victor Nadolschi ocupa prin concurs conferinta de astronomie si devine
directorul Observatorului din Iasi, functie detinuta pna n anul 1966. V. Nadolschi
este un eminent continuator al lui C. Popovici si al lui V. S
iadbei. Acesta reorganizeaz
a si relanseaz
a activitatea si pune bazele astronomiei fotografice la Iasi.
V. Nadolschi achizitioneaza un astrograf Zeiss (1956), un fotometru fotoelectric (1959),
un ecuatorial Zeiss cotit (1960), un aparat pentru masurat clisee (1963), un teodolit
zenital Meopta (1963) etc.
ncepnd cu anul 1966 activitatea didactic
a si de cercetare este coordonat
a de
Iulian Breahn
a, absolvent al Universitatii din Bucuresti, sectia de astronomie.
Din 1966 functioneaza n cadrul Observatorului din Iasi un atelier de mecanica
fina si un laborator electronic necesare ntretinerii si cercet
arii. A fost achizitionat
un orologiu cu cuart care, completat ulterior cu alte anexe, constituie si n prezent
un cronograf digital de precizie.
n anul 1980 a fost achizitionat un planetariu Zeiss destinat nvatamntului astronomiei, care a fost instalat n incinta Universit
atii din Iasi. Studentii au astfel
posibilitatea de a-si nsusi mai usor o multitudine de fenomene privind cinematica si
dinamica sistemului planetar al Soarelui. Planetariul a atras pna n prezent cteva
mii de vizitatori.
Cu prilejul eclipsei totale de Soare din 11 august 1999 s-a achizitionat un astrograf
CCD (dispozitiv cu cuplaj de sarcin
a) de performanta si o camera Astrovid pentru
nregistrari continue de imagini.
n perioada 1951-1999, pe lnga Observator si prin grija personalului acestuia a
functionat o statie seismica. Observatiile efectuate de aceasta au pus n evidenta
dou
a focare seismice: unul la circa 25 km dincolo de Prut si al doilea n zona BrladZorleni.
Activitatea de cercetare desfasurata pe lnga Observatorul din Iasi s-a concretizat
n peste 140 lucr
ari. C. Popovici a generalizat legea Newton-Coulomb prin considerarea unei forte neconservative, rezultat
a dintr-o combinatie a gravitatiei newtoniene
cu presiunea luminii. V. S
iadbei obtine rezultate noi privind traiectoriile meteorilor
si cometelor si face observatii asupra eclipselor de Luna si Soare, stabilind relatii mai
simple pentru calculul acestora. V. Nadolschi s-a preocupat de teoria statisticii
grupurilor de pete solare, continu
a traditia observ
arii eclipselor si are meritul de
a fi pus bazele astronomiei fotografice la Iasi. Abordarea unor teme din domeniul
radioastronomiei s-a dovedit deosebit de dificila, desi s-au depus eforturi sustinute
pentru crearea bazei materiale necesare unei astfel de cercet
ari.
Cu toate c
a de-a lungul timpului au fost de nl
aturat multe dificult
ati si obstacole, la Observatorul din Iasi s-a reusit sa se desfasoare o activitate care l-a facut
cunoscut n tara si n strainatate. Aceste afirmatii sunt dovedite si de acordarea titlului de membru al Uniunii Astronomice Internationale urm
atorilor astronomi ieseni:
Constantin Popovici, Vintila Siadbei,
Victor Nadolschi si Iulian Breahna.

Redactia revistei
4

Marea teorem
a a lui Fermat pentru polinoame
Temistocle BRSAN 1
1. Odata cu caderea Constantinopolului (1453), multi nvatati bizantini s-au
ndreptat spre Europa de Vest aducnd cu ei manuscrise pretioase - manuscrisele
care supravietuiser
a devast
arii Bibliotecii din Alexandria se adunaser
a de-a lungul
timpului n aceasta capitala a lumii.
Prin hazardul mprejurarilor, sase din cele 13 volume ale Aritmeticii lui Diofant
au ajuns n Franta. nv
atatul si amatorul de matematic
a francez Claude Gaspar
Bachet de Mziriac si d
a seama de importanta c
artii lui Diofant si public
a n 1621
o versiune n limba latina a Aritmeticii, care cuprinde peste o suta de probleme si
rezolvarile detaliate ale lui Diofant.
Pentru Pierre Fermat (1601-1665) Aritmetica lui Diofant a fost cartea care
l-a pus n contact cu bogatele cunostinte ale popoarelor antice n directia teoriei
numerelor si sursa de inspiratie pentru noi si subtile probleme pe care singur si le
formula. Fermat obisnuia sa noteze pe marginile cartii lui Diofant comentarii, calcule si schite de demonstratii. Nu s-a preocupat s
a-si publice rezultatele si demonstratiile, dar se amuza comunicndu-si rezultatele altor matematicieni ai timpului si
provocndu-i la rezolvarea acestora.
n Cartea a II-a a Aritmeticii, Fermat gaseste informatii bogate relativ la tripletele
pitagoreice, adic
a trei numere naturale ce verific
a ecuatia lui Pitagora
x2 + y 2 = z 2 .

(1)

S
tia c
a Euclid demonstrase c
a exist
a o infinitate de astfel de triplete. Ce se ntmpl
a,
ns
a, dac
a n loc de (1) se consider
a ecuatia
xn + y n = z n ,

(2)

unde n 3? R
aspunsul lui Fermat, notat ca observatie pe marginea c
artii lui Diofant,
este cu totul surprinz
ator: nu exista nici o solutie a ecuatiei (2) cu numere x, y, z
nenule, daca n = 3, 4, . . . . Urmeaza notat urmatorul comentariu:
Cuius rei demonstrationem mirabilem sane detex hanc marginis exiguitas non
caperet [4]. (Ma aflu n posesia unei demonstratii minunate a acestei afirmatii, dar
marginea paginii este prea strmta pentru a o cuprinde.)
Aceasta extraordinara descoperire, care astazi poarta numele de Marea teorem
a
a lui Fermat, ct si alte rezultate, ar fi putut sa ramna necunoscute lumii matematicienilor si s
a se piard
a, dac
a, dup
a moartea lui Fermat, fiul s
au cel mai mare n-ar
fi examinat nsemn
arile scrise de tat
al s
au pe margini si n-ar fi publicat Aritmetica
lui Diofant continnd si observatiile lui Pierre de Fermat (Toulouse, 1670).
Pe parcursul ctorva secole, cele mai sclipitoare minti de matematicieni au ncercat si si-au adus contributia la rezolvarea acestei enigme (si, totodat
a, provoc
ari)
l
asat
a de Fermat: Euler, Sophie Germain, Dirichlet, Legendre, Lam, Cauchy,
Kummer s. a. Drumul ce duce la demonstrarea Marii teoreme a lui Fermat este
1

Prof. dr., Catedra de matematic


a, Univ. Tehnic
a "Gh. Asach", Iasi

pres
arat cu reusite partiale, ambitii, nfrngeri, deceptii, orgolii, intrigi, tentative de
sinucidere etc. [4].
n anul 1995, dupa opt ani de munca nentrerupta, n completa izolare fata de
colegii s
ai si p
astrnd o discretie total
a asupra cercet
arilor sale, englezul Andrew
Wiles pune cap
at enigmei de peste 350 de ani: Marea teorema a lui Fermat este
demonstrata! Demonstratia data de Wiles este, nsa, accesibila unui numar restrns
de specialisti; n fapt, Wiles pentru a atinge scopul a dovedit justetea Conjecturii
Taniyama - Shimura utiliznd o aparatur
a matematic
a modern
a si sofisticat
a: curbe
eliptice, forme modulare, reprezent
ari Galois s. a. [5].
2. Este cunoscut faptul c
a inelul Z al numerelor ntregi si inelul C [X] al polinoamelor cu coeficienti numere complexe au propriet
ati asem
an
atoare. De aceea
apare ca fireasca problema rezolvarii ecuatiilor (1) si (2) n C [X].
n privinta ecuatiei (1) constat
am usor, ca si n cazul numeric, c
a are o infinitate
de solutii: p, q C [X], lu
am
2

x (X) = [p (X)] [q (X)] , y (X) = 2p (X) q (X) , z (X) = [p (X)] + [q (X)]


si verificam direct ca tripleta (x (X) , y (X) , z (X)) este o solutie a ecuatiei (1) n
C [X].
Similar cu Marea teorem
a a lui Fermat se formuleaz
a
Teorema lui Fermat pentru polinoame ([3], [5]). Daca n este un ntreg,
n 3, atunci ecuatia (2) nu are solutii n C [X] cu polinoame neconstante si relativ
prime.

Surprinz
ator, spre deosebire de Marea teorem
a a lui Fermat, pentru acest rezultat
se cunoaste o demonstratie elementara si simpla, accesibila unui elev de liceu. Rezultatul este cunoscut din sec. al XIX-lea si a fost demonstrat utiliznd cunostinte de
geometrie algebric
a. Demonstratia elementar
a la care ne-am referit se sprijin
a pe
o teorem
a de dat
a recent
a datorat
a matematicienilor W. Stothers (1981) si, independent, R. C. Mason (1983), teorema foarte importanta si n sine. Sunt necesare
cteva (putine!) pregatiri.
Fie p C [X] un polinom neconstant avnd r
ad
acinile a1 , a2 , . . . , ak cu ordinele
de multiplicitate respective m1 , m2 , . . . , mk ; deci p se scrie sub forma
k
Y
(X ai )mi , C .
(3)
p (X) =
i=1

Not
am gradul polinomului p si num
arul r
ad
acinilor sale distincte cu deg p si respectiv
a
n0 (p), adic
deg p = m1 + m2 + + mk , n0 (p) = k.
Mention
am c
a, dac
a p, q C sunt neconstante, avem
deg (pq) = deg p + deg q, n0 (pq) n0 (p) + n0 (q) ,
cu egalitate dac
a si numai dac
a p si q sunt relativ prime.
Derivata formal
a a polinomului p dat de (3) este
p0 (X) = [m1 (X a1 )
+ mk (X a1 )

m1 1

m1

(X a2 )

m2

(X ak1 )
6

(X ak )

mk1

(X ak )

mk

+ +

mk 1

si, ca urmare, cel mai mare divizor comun al polinoamelor p si p0 are forma
(p, p0 ) = (X a1 )m1 1 (X a2 )m2 1 (X ak )mk 1 .
Atunci
deg (p, p0 ) = (m1 1) + (m2 1) + + (mk 1) = deg p n0 (p) ,
de unde obtinem relatia
deg p = deg (p, p0 ) + n0 (p) .

(4)

Teorema Mason - Stothers. Fie p, q, r C [X] neconstante si relativ prime.


Daca are loc egalitatea p + q = r, atunci
max {deg p, deg q, deg r} n0 (pqr) 1.

(5)

Demonstratie (dat
a de Noah Snyder [3], p.30). Vom ncepe cu dou
a observatii
utile. Mai nti, n prezenta conditiei p + q = r, polinoamele p, q, r sunt relativ prime
daca si numai daca sunt prime doua cte doua. Apoi, ntruct enuntul teoremei este
simetric n p, q, r (c
aci putem scrie egalitatea si sub forma p + q + r = 0), nu
restrngem generalitatea dac
a vom presupune c
a polinomul r are gradul cel mai
ridicat. Ca urmare, inegalitatea de demonstrat se scrie
deg r n0 (pqr) 1.

(50 )

Avem
p0 q pq 0 = p0 (p + q) p (p0 + q 0 ) = p0 r pr0 .
Constat
am c
a (p, p0 ) si (q, q 0 ) divid membrul stng, iar (r, r0 ) divide membrul drept,
deci si pe cel stng. Cum p, q, r sunt prime doua cte doua, urmeaza ca produsul
(p, p0 ) (q, q 0 ) (r, r0 ) divide p0 q pq 0 . n consecinta,
deg (p, p0 ) + deg (q, q 0 ) + deg (r, r0 ) deg (p0 q pq 0 ) deg p + deg q 1

sau, datorita relatiei (4) si analoagelor ei,


deg p n0 (p) + deg q n0 (q) + deg r n0 (r) deg p + deg q 1,
deci
deg r n0 (p) + n0 (q) + n0 (r) 1.
Cum p, q, r sunt prime doua cte doua, obtinem n final
deg r n0 (pqr) 1,

care este tocmai relatia (50 ) de demonstrat.

Demonstratia Teoremei lui Fermat pentru polinoame. Presupunem ca


ecuatia (2) pentru n 3 ar avea o solutie (x (X) , y (X) , z (X)) cu polinoame neconn
stante relativ prime. Aplic
am teorema Mason - Stothers polinoamelor p(X) = [x(X)] ,
n
n
q (X) = [y (X)] si r (X) = [z (X)] . Obtinem
deg [x (X)]n n0 ([x (X)]n [y (X)]n [z (X)]n ) 1
sau
n deg x (X) n0 (x (X) y (X) z (X)) 1.
7


Tinnd
seama c
a x (X), y (X) si z (X) sunt prime dou
a cte dou
a si de faptul c
a
n0 (p) deg p, p C [X], vom avea
n deg x (X) n0 (x (X)) + n0 (y (X)) + n0 (z (X)) 1
deg x (X) + deg y (X) + deg z (X) 1.

Obtinem astfel inegalitatea

n deg x (X) deg x (X) + deg y (X) + deg z (X) 1,

precum si inegalitatile analoage scrise pentru y (X) si z (X), care adunate dau
n (deg x (X) + deg y (X) + deg z (X)) 3 (deg x (X) + deg y (X) + deg z (X)) 3,

adic
a

(n 3) (deg x (X) + deg y (X) + deg z (X)) 3.


Evident, dac
a n 3, aceast
a relatie ne conduce la o absurditate, ceea ce ncheie
demonstratia.
3. Analogia care exist
a ntre inelele Z si C [X] pune n mod firesc problema
"translarii" teoremei Mason - Stothers de la polinoame la numerele ntregi astfel
nct Marea teorem
a a lui Fermat s
a poat
a fi demonstrat
a elementar.
D. Masser si J. Oesterle (1986) au ajuns la asa - numita conjectura abc ca
urmare a unor consideratii de geometrie algebric
a si teoria functiilor modulare (si nu
n legatura cu teorema Mason - Stothers).
k
Q
i
Daca m N are descompunerea n factori primi m =
pm
i , atunci vom numi
i=1
k
Q
pi .
radicalul lui m numarul N0 (m) =
i=1

Conjectura abc ([2], [3]). Dat > 0, exista o constanta C () astfel nct
pentru orice ntregi a, b, c nenuli si relativ primi cu a + b = c avem inegalitatea
1+

max {|a| , |b| , |c|} C () (N0 (abc))

Aceast
a conjectur
a spune c
a, dac
a n descompunerea numerelor a, b, c exist
a
factori primi cu exponenti mari, acesti factori sunt compensati prin factori primi mai
multi, dar cu exponentul 1.
S
a enuntam acum asa - numita
Teorema lui Fermat asimptotic
a. Exista un ntreg pozitiv n1 cu proprietatea
ca, daca n n1 , atunci ecuatia (2) nu are solutii cu x, y, z ntregi si xyz 6= 0.
Cu aceleasi argumente ca n cazul polinoamelor se poate dovedi urmatoarea
Teorem
a ([2], [3]). Conjectura abc implica Teorema lui Fermat asimptotica.
Demonstratie. Fie date x, y, z pozitive si relativ prime astfel nct tripleta
(x, y, z) sa fie solutie a ecuatiei (2), adica xn + y n = z n .
Not
am a = xn , b = y n si c = z n si observ
am c
a
N0 (abc) = N0 (xn y n z n ) = N0 (xyz) xyz.

Utiliznd conjectura abc obtinem


xn C () (xyz)1+ ,

y n C () (xyz)1+ ,
8

z n C () (xyz)1+ .

Prin nmultire, rezult


a c
a
n

de unde
si cum xyz > 2, obtinem

3+3

(xyz) [C ()] (xyz)

(n 3 3) log (xyz) 3 log C ()

3 log C ()
+ 3 + 3.
log 2
Not
am

3 log C ()
+ 3 + 3 .
(6)
n1 =
log 2
Urmeaz
a c
a ecuatia (2) nu are solutii ce verific
a conditiile specificate dac
a n n1 ,
ceea ce trebuia demonstrat.
n<

Observatie. Aceast
a cale nu ofer
a o demonstratie a Marii teoreme a lui Fermat.
ntr-adevar, numarul n1 definit de (6) depinde de C () (putem considera = 1 si
C (1) pentru a fixa ideile). Determinarea efectiv
a a constantei C () nu este cunoscut
a. Dac
a, de exemplu, C (1) s-ar putea efectiv determina, atunci demonstratia
Marii Teoreme a lui Fermat s-ar reduce la un num
ar finit de cazuri, care ar putea fi
abordate prin calcul direct.
4. Interesul pentru Marea teorema a lui Fermat nu s-a stins nici dupa demonstrarea ei. Au ramas ntrebari fara raspuns, sunt formulate altele noi. Daca Fermat
nu a dat dect o demonstratie eronat
a, care ar putea fi natura greselii f
acute? Dac
a
aceast
a demonstratie ar fi corect
a, care este acel argument ingenios produs de geniul
lui Fermat ce a scapat attor matematicieni ilustri? Este posibila o demonstratie
elementara, accesibila si unor persoane cu cunostinte obisnuite de matematica?
n 1966, Andrew Beal instituie un premiu pentru demonstrarea sau infirmarea
asa - numitei Conjecturi Beal, care este o generalizare a problemei lui Fermat:
Ecuatia xp + y q = z r , p, q, r numere ntregi mai mari ca 2, nu are nici o solutie
cu x, y, z ntregi pozitivi si relativ primi ([6], [1]).
Bibliografie
1.
2.
3.
4.
5.
6.

A. Corduneanu - Despre Marea teorema a lui Fermat, Recreatii Matematice,


1 (1999), nr.1, 37-39.
S. Lang - Old and new conjectured diophantine inequalities, Bull. AMS, 23 (1990),
37-75.
S. Lang - Math Talks for Undergraduates, Springer, 1999.
S. Singh - Marea teorema a lui Fermat, Humanitas, Bucuresti, 1998.
A. Wiles - Modular elliptic curves and Fermats Last Theorem, Annals of Math.,
142 (1995), 443-551.
***
- Beals Conjecture, The New Zealand Math. Mag., 35 (1998), no.2, 38.

De la o problem
a cu matrice la transform
ari
elementare
Marian TETIVA1
1. Introducere. Problema la care ne referim n titlu este urmatoarea:
Sa se arate ca nu exista matrice patratice X, Y Mn (C) astfel nct XYY X = In ,
In fiind matricea unitate de ordinul n.
Este o problem
a cunoscut
a, care poate fi ntlnit
a n mai multe manuale sau
culegeri, care s-a dat la concursuri etc. si nu este tocmai simpl
a: un elev mediu este
ntotdeauna descurajat de enunturi de tipul "sa se arate ca exista /nu exista. . . ".
Mai mult, n aceasta situatie nu prea avem alta cale de abordare n afara celei care
utilizeaz
a notiunea de urm
a a unei matrice si propriet
atile sale. Istoria problemei
este cam asa: prin anii 70 ai secolului trecut ea era propus
a la olimpiad
a, prin anii
80 a patruns n manuale pentru ca n anii 90 sa ajunga a fi parte din diverse teste
de bacalaureat sau admitere la facultate; aceasta spune ceva despre felul n care au
evoluat programele nv
atamntului matematic elementar n Romnia. Noi credem
c
a elevul mediu din ziua de azi se afl
a n acelasi impas ca si cel de acum dou
azeci sau
treizeci de ani (sau poate chiar mai rau) atunci cnd este confruntat cu asemenea
probleme. De aceea aceasta nota i se adreseaza, dar numai daca este cu adevarat
interesat de matematic
a.
Amintim c
a urma matricei A = (aij )1i, jn Mn (C) este, prin definitie,
numarul Tr (A) = a11 + a22 + + ann (suma elementelor situate pe "diagonala
principal
a" a matricei). Sunt cunoscute urm
atoarele propriet
ati ale urmei:
1 Tr (A + B) = Tr (A) + Tr (B), A, B Mn (C),
2 Tr (A) = Tr (A), C, A Mn (C),
3 Tr (AB) = Tr (BA), A, B Mn (C).
Primele dou
a egalit
ati se mai pot scrie condensat n forma Tr (A + B) =
= Tr (A) + Tr (B), oricare ar fi , C si A, B Mn (C) si exprim
a liniaritatea urmei: Tr : Mn (C) C este aplicatie liniara (sau morfism de C - spatii
vectoriale). De aici deducem Tr (XY Y X) = Tr (XY ) Tr (Y X) = 0 pentru orice
X, Y Mn (C) si aceasta explic
a de ce egalitatea din enunt nu poate avea loc pentru
nici o pereche de matrice X, Y : matricea XY Y X are urma nul
a, deci nu poate fi
egala cu In , a carei urma este n.
Remarc
am c
a matricea In din enunt poate fi nlocuit
a cu orice matrice de ordinul
n avnd urma nenul
a, enuntul si rezolvarea r
amnnd valabile; problema poate fi
usor reformulat
a astfel:
Daca pentru o matrice A Mn (C) exista X, Y Mn (C) astfel nct A =
XY Y X, atunci Tr (A) = 0.
Atunci se naste n mod natural ntrebarea dac
a reciproca acestei afirmatii este
adev
arat
a, adic
a se pune problema valabilit
atii urm
atorului enunt:

Fie A o matrice patratica de ordin n cu elemente numere complexe. Daca urma


matricei A este nula, atunci exista X, Y Mn (C) astfel nct A = XY Y X.
1

Profesor, Colegiul National "Gh. Rosca Codreanu", Brlad

10

n cele ce urmeaz
a ne propunem s
a rezolv
am aceast
a problem
a; mai precis, s
a
aratam ca raspunsul la ntrebare este afirmativ.
Ideea rezolvarii este sa cautam niste matrice Y, Z Mn (C) astfel nct A sa poata
fi scris
a n forma A = Z Y ZY 1 (Y fiind inversabil
a, desigur);
atunci
problema

va

fi rezolvat
a: e suficient s
a alegem X = ZY 1 si avem A = ZY 1 Y Y ZY 1 =
= XY Y X.
Aici cititorul poate avea o nemultumire: de unde si pna unde aceste matrice Y, Z
n locul lui Xsi Y
a remarc
am c
a din proprietatea 3 rezult
a c
a avem si
din enun
t? S
4 Tr(A) =Tr CAC 1 , C Mn (C), C inversabil
a (se dovedeste imediat, c
aci
1
1
A = AC
sunt asemenea, iar proprietetea
C). Matricele de forma A si CAC
4 spune ca acestea au aceeasi urma.
Desigur, problemele abia ncep. Sunt necesare cteva pregatiri.
2. Matrice asemenea si transform
ari elementare. Fie K un corp comutativ;
cititorul mai putin familiarizat cu aceast
a notiune abstract
a poate considera K o
notatie pentru unul dintre corpurile numerice uzuale Q, R sau C.
Dou
a matrice X, Y Mn (K) se numesc matrice asemenea ( sau similare) dac
a
exista U Mn (K) cu det U 6= 0 astfel nct Y = U XU 1 (vom nota X Y ).
Cititorul poate verifica usor faptul c
a relatia de asem
anare (similaritate) este o relatie
de echivalenta pe multimea Mn (K).
Transformarile elementare care se fac asupra unei matrice sunt, n principiu,
cele mai simple modificari care nu i afecteaza rangul, adica interschimbarea a doua
linii (sau coloane), adunarea unei linii (coloane) nmultite cu un numar (n general:
element al corpului K) la alt
a linie (respectiv coloan
a), sau chiar nmultirea unei
linii (coloane) cu un num
ar nenul. Una din cele mai simple aplicatii ale lor este
calculul rangului unei matrice; de asemenea, se pot folosi aceste transformari pentru
rezolvarea sistemelor de ecuatii liniare.
S
a ncepem prezentarea transform
arilor elementare cu asa numitele matrice elementare; leg
atura va ap
area curnd. Vom nota cu Eij matricea p
atratic
a de ordin
n peste corpul K ale carei elemente sunt toate nule, cu exceptia elementului de pe
linia i si coloana j care este egal cu 1. Se verifica usor ca matricele Eij , 1 i n,
1 j n formeaz
a o baz
a a spatiului vectorial Mn (K) peste K, precum si relatiile
Eij Ekl = 0n , j 6= k si Eij Ejl = Eil .
Se numesc matrice elementare urmatoarele tipuri de matrice patratice de ordinul
n, cu elemente din K:
1) Matricele Tij (a) = In +aEij ; aici a K si i, j {1, 2, . . . n} sunt indici diferiti.
Matricea Tij (a) se obtine din matricea unitate f
acndu-i o singur
a modificare: elementul de pe linia i si coloana j devine a. Se constata imediat ca (vezi proprietatile
matricelor Eij )
Tij (0) = In , Tij (a) Tij (b) = Tij (a + b) ,
Tij (a) GLn (K) , Tij (a)

= Tij (a) , a, b K,

unde GLn (K) = {U Mn (K) | det U 6= 0} - grupul general liniar de ordin n peste
corpul K. Deci {Tij (a) | a K} formeaz
a, pentru i 6= j fixate, un grup izomorf cu
grupul (K, +). Sa vedem ce efect are nmultirea unei matrice oarecare cu o matrice
11

Tij (a). Fie A = (akl )1k,ln o matrice din Mn (K), care mai poate fi scris
a si
n
X
akl Ekl . Atunci
A=
k,l=1

Tij (a) A = (In + aEij )


n
X

X
n

akl Ekl +

k,l=1

akl Ekl

k,l=1
n
X

n
X

akl Ekl +

k,l=1

n
X

aakl Eij Ekl =

k,l=1

aajl Eil .

l=1

amn aceleasi ca
Ce nseamn
a asta? nseamn
a c
a elementele matricei Tij (a) A r
ale matricei A, cu exceptia celor de pe linia i: aici, n locul elementului ail apare
acum ail + aajl , adic
a matricea Tij (a) A se obtine din A prin adunarea la linia i
a liniei j nmultite cu a, cu alte cuvinte nmultirea la stnga cu o matrice Tij (a)
realizeaz
a o transformare elementar
a a matricei A. De asemenea, se poate verifica n
acelasi fel c
a matricea ATij (a) se obtine din A prin adunare la coloana j a coloanei
i nmultite cu a.
2) Matricele Qij = Tij (1) Tji (1) Tij (1) (i, j {1, 2, . . . , n} , i 6= j) intra si ele
n categoria matricelor elementare. Avem
Qij = Tij (1) Tji (1) Tij (1) = (In Eij ) (In + Eji ) (In Eij ) =
= (In + Eji Eij Eii ) (In Eij ) = In + Eji Eij Eii Eij Ejj + Eij =
= In Eii Ejj Eij + Eji ,

deci Qij este matricea care se obtine din matricea unitate prin schimbarea a patru
elemente: elementele de pe diagonala principala, de pe linia i, coloana i si de pe linia
j, coloana j se nlocuiesc cu zerouri; n locul elementului de pe linia i si coloana j
avem 1, iar n locul celui de pe linia j si coloana i se g
aseste 1. La fel ca mai sus,
s
a calcul
am
X

n
Qij A = (In Eii Ejj Eij + Eji )
akl Ekl =
=

n
X

k,l=1

akl Ekl
=

n
X

k,l=1

n
X

k,l=1

akl Eii Ekl

akl Ekl

n
X
l=1

n
X

k,l=1

akl Ejj Ekl

ail Eil

n
X
l=1

ajl Ejl

k,l=1
n
X

akl Eij Ekl +

k,l=1
n
X

ajl Eil +

l=1

n
X

akl Eji Ekl =

k,l=1
n
X

ail Eil ;

l=1

asadar, matricea Qij A se obtine din A prin nlocuirea liniei i, respectiv j, cu linia j
nmultita cu 1, respectiv cu linia i. Asemanator, se poate observa ca schimbarile pe
care le produc asupra lui A nmultirea cu matricea Qij la dreapta sunt urm
atoarele:
coloana i se nlocuieste cu coloana j, iar coloana j se nlocuieste cu coloana i nmultit
a
cu 1. Sa mai spunem ca, fiind produs de matrice inversabile, Qij este, de asemenea,
matrice inversabila; avem
1

Q1
ij = Tij (1)
si, deci,

Q1
ij

Tji (1)

Tij (1)

= Qij .
12

= Tij (1) Tji (1) Tij (1)

3) Un alt tip de matrice elementare sunt matricele Di (d) = In + (d 1) Eii ,


1 i n, d K fiind nenul; o astfel de matrice se obtine din matricea unitate
modificndu-i un singur element: n locul lui 1 de pe linia i si coloana i punem d. Nu
e greu de v
azut c
a nmultirea unei matrice A oarecare cu Di (d) la stnga (respectiv
la dreapta) i modific
a doar linia (respectiv coloana) i, anume o nlocuieste pe aceasta

cu linia (respectiv coloana) i nmul


tita cu d. De asemenea, Di (d) este inversabila si
1
1
are inversa Di (d) = Di d
.
4) Vom folosi matricele Pij pe care le definim prin
Pij = Di (1) Qij = Qij Dj (1) = In Eii Ejj + Eij + Eji .

Verificati aceste egalit


ati! Observati forma matricei Pij si efectul s
au la nmultire:
matricea APij (respectiv Pij A) se obtine din A prin interschimbarea liniilor (respectiv
coloanelor) i si j. S
i nu n ultimul rnd, aratati ca Pij este inversabila si Pij1 = Pij .
3. Rezolvarea problemei. Sa demonstram asadar urmatoarea
Propozitie. Fie K un corp comutativ infinit si A Mn (K) o matrice a carei
urma este zero. Atunci exista matricele X, Y Mn (K) astfel nct A = XY Y X.
Demonstratie. Pentru nceput vom presupune ca nu exista nici o submultime
a multimii {a11 , a22 , . . . , ann } a elementelor de pe diagonala principal
a a matricei A
pentru care suma elementelor s
a fie nul
a, desigur, cu exceptia ntregii multimi (vom
vedea imediat la ce ne foloseste aceast
a presupunere, iar la sfrsit ne vom da seama c
a
nu este prea restrictiva). Deoarece, conform ipotezei, avem a11 + a22 + + ann = 0,
se pot determina elementele a1 , a2 , . . . , an K astfel nct
a11 = a1 a2 , a22 = a2 a3 , . . . , an1,n1 = an1 an , ann = an a1 .

E clar ca, datorita ipotezei suplimentare pe care am facut-o, oricare doua dintre
elementele a1 , a2 , . . . , an sunt distincte; vom folosi acest lucru mai departe.
Putem scrie pe A n forma A = B C, unde

a1
a2 a12 . . . a1,n1 a1n
0
...
0
0
a21
0
a2
...
0
0
a3 . . . a2,n1 a2n

...
...
...
. . . , C = . . . . . . . . .
...
...
B = ...
.
an1,1 an1,2 . . . an1 0
0
0
...
an an1,n
an2 . . . an,n1 an
0
0
...
0
a1
an1

Asa cum am ar
atat la nceput, pentru rezolvarea problemei ar fi suficient s
a
ar
at
am c
a matricele B si C sunt asemenea. n acest scop, vom ar
ata c
a B B0,
C C 0 , unde

a1
a2
0 ... 0
0 ... 0
0
0
a2 . . . 0
a3 . . . 0
0

B0 =
. . . . . . . . . . . . , C = . . . . . . . . . . . .
0
0 . . . an
0
0 . . . a1

si B 0 C 0 . Folosind faptul c
a asem
anarea matricelor este o relatie de echivalenta,
obtinem c
a B C.
Sa le luam pe rnd. Ne amintim ca nmultirea unei matrice cu matricea Pij la
stnga (respectiv la dreapta) schimba ntre ele liniile (respectiv coloanele) i si j ale
acelei matrice. De aceea, pentru M Mn (K), matricea Pij M Pij1 = Pij M Pij
13

are schimbate ntre ele elementele de pe diagonala principal


a situate pe liniile (si
coloanele) i si j; de asemenea, mai sunt afectate si celelalte elemente de pe liniile si
coloanele i si j. Aceasta nu are nsa importanta n cazul unor matrice precum B 0
sau C 0 , la care toate elementele din afara diagonalei principale sunt zerouri; astfel
Pij B 0 Pij1 = Pij B 0 Pij este o matrice care difer
a de B 0 doar prin aceea c
a si-au
schimbat ntre ele locurile doua elemente de pe diagonala principala, anume ai si
aj . Cum orice permutare e produs de transpozitii, e clar ca dupa un numar finit
de asemenea transform
ari o putem aduce pe B 0 la orice form
a n care pe diagonala
principal
a apar elementele a1 , a2 , . . . , an permutate cumva (si n rest, zerouri). n
particular, B 0 este asemenea cu C 0 .
Sa aratam acum ca B B 0 (si nu vom mai face demonstratia pentru C C 0 , ea
fiind ntru totul asem
an
atoare). ncepem prin a observa urm
atorul calcul:

1 0
0
1 0

0
0
=
=
,
a 1

1
a ( ) +
0
daca a este ales convenabil, adica daca a = / ( ); desigur, asta se poate face
numai n cazul n care 6= .
Un calcul asem
an
ator se poate face si pentru matrice de ordin n. Dac
a vom
considera Tij (a) BTij (a)1 = Tij (a) BTij (a), unde i > j, elementul aij din pozitia
(i, j) se nlocuieste cu a (aj ai ) + aij si a poate fi ales astfel nct acest element sa
devin
a nul (c
aci am presupus c
a ai 6= aj ). Mai sunt afectate si celelalte elemente ale
liniei i (la care se adun
a linia j nmultit
a cu a) si ale coloanei j (la care se adun
a
coloana i nmultita cu a). Remarcam ca aceste transformari oricum nu pot modifica
zerourile de deasupra diagonalei principale, care ramn intacte, si nici elementele de
pe diagonala principal
a.
1
Acum e clar ce avem de f
acut: mai nti calcul
am T21 (a) BT21 (a) care, pentru
un a bine ales reprezinta o matrice B1 asemenea cu B care are n pozitia (2, 1) pe
zero (asta daca nu era dinainte; de altfel se poate vedea usor ca, daca a21 = 0,
atunci a care ne trebuie este a = 0 deci T21 (a) = T21 (0) este, de fapt, matricea
identica). Apoi, pentru aceasta matrice calculam T31 (a) B1 T31 (a)1 , care pentru un
anumit a este o matrice asemenea cu B1 (deci si cu B) si are 0 n pozitia (3, 1); se
poate vedea c
a elementul 0 obtinut la pasul anterior nu va fi afectat. Continu
am
astfel, lucrnd cu matrice de forma Ti1 (a) pn
a cnd toate elementele de pe prima
coloana "de sub" a1 devin zerouri, apoi trecem si facem zerouri pe coloana a doua,
"sub" a2 , folosind transformari de tip T32 (a) , . . . , Tn2 (a) (adica nmultim cu acestea
la stnga si cu inversele lor la dreapta; la fiecare pas similaritatea matricelor se
p
astreaz
a), n ordine, alegnd, desigur, de fiecare dat
a valoare care trebuie pentru a.
Elementele nule obtinute pe prima coloana nu vor fi afectate, la fel cele de pe sau de
deasupra diagonalei principale, Tot asa vom proceda pna cnd, la urma, ajungem
la o matrice care are partea de deasupra diagonalei principale neschimbat
a, la fel
diagonala principal
a, iar sub diagonala principal
a are numai zerouri, adic
a ajungem
la B 0 si la concluzia dorita ca aceasta este asemenea cu B.
n concluzie, am aratat ca matricele B si C sunt asemenea, deci am ajuns acolo
unde ne-am propus: exist
a V GLn (K) astfel nct C = V BV 1 ; atunci A =
1
= B C = B V BV
si notnd X = BV 1 , Y = V avem A = XY Y X.
Demonstratia ar fi ncheiata, daca n-ar mai fi un mic amanunt de lamurit: ce
14

facem cu ipoteza suplimentar


a pe care am impus-o (si de care, s-a dovedit pe parcurs,
avem mare nevoie, caci daca elementele de pe diagonala nu sunt distincte, nu-l putem
alege pe a astfel nct Tij (a) sa produca un zero n locul lui aij )? Raspunsul nu e att
de greu si arat
a, cum spuneam, c
a restrictia dat
a de aceast
a ipotez
a nu este chiar att
de ... restrictiv
a. E suficient s
a mp
artim multimea elementelor de pe diagonal
a n
submultimi disjuncte doua cte doua, fiecare dintre acestea avnd suma elementelor
0 si fiecare nemaiavnd alta submultime (stricta) pentru care suma elementelor este
0. S
a numim b1 , . . . , bk elementele unei asemenea submultimi (a c
aror sum
a este,
asadar, zero); pentru acestea putem determina c1 , c2 , . . . , ck astfel nct b1 = c1 c2 ,
b2 = c2 c3 , . . . , bk1 = ck1 ck , bk = ck c1 . Mai mult, oricare doua dintre
c1 , c2 , . . . , ck sunt distincte doua cte doua si proprietatile lor se pastreaza daca le
asim cte o grupare de asemenea c-uri
nlocuim cu c1 + t, c2 + t, . . . , ck + t, t K. G
distincte dou
a cte dou
a pentru fiecare submultime de b-uri a multimii elementelor
de pe diagonala principala, iar apoi alegem cte un t pentru fiecare astfel de grupare
nct toate c-urile sa fie distincte doua cte doua (ceea ce sigur se poate face n cazul
n care corpul K este infinit; gnditi-v
a de ce!). Mai departe totul decurge la fel,
deoarece putem scrie matricea noastr
a ca diferenta a dou
a matrice, una inferior, alta
superior triunghiulara, fiecare dintre aceste elemente sunt distincte doua cte doua.
Propozitia este complet demonstrat
a.
Noi ne-am propus s
a rezolv
am problema n cazul corpurilor uzuale de numere:
Q, R, C, de aceea ipoteza pe care am facut-o asupra infinitatii corpului K nu ne
deranjeaz
a foarte mult; totusi, se prea poate ca aceast
a presupunere s
a fie strict
legat
a de rezolvarea pe care am dat-o aici si s
a nu fie esential
a. Asadar, ramne
ntrebarea daca este valabil enuntul propozitiei demonstrate si n cazul unui corp
finit.
n ncheiere, s
a mai spunem c
a nu exist
a nici o pretentie de originalitate n elaborarea acestei note; este foarte posibil ca aceast
a solutie s
a fie cunoscut
a, atta doar c
a
autorul nu are nici un fel de referinta pentru problema discutat
a, pe care o cunoaste
doar din folclor (n urma cu ctiva ani aceasta problema mi-a fost comunicata "prin
viu grai" de c
atre un elev, actualmente student str
alucit al Facult
atii de Matematic
a
din Bucuresti; asa c
a i multumesc pe aceast
a cale lui Dragos Deliu, care m-a f
acut
s
a caut s
a rezolv aceast
a problem
a, c
aut
ari din care s-a n
ascut si aceast
a not
a).

Recreaii matematice
1. S
a se ndep
arteze patru segmente din figura al
aturat
a
(alc
atuit
a din sase p
atrate) astfel nct noua figur
a s
a fie formata din trei patrate.
Not
a. Solutia problemei se poate g
asi la pagina 39.

15

Trei perle ale olimpiadelor de matematic


a
Gabriel DOSPINESCU 1
Problemele propuse la testele de selectie pentru OIM sau la fazele nationale din
diverse tari se remarca prin profunzimea (si uneori simplitatea) ideilor care le rezolva.
n cele ce urmeaz
a, vom rezolva trei probleme propuse la astfel de teste de selectie
n anii 2002 si 2003, demonstrnd dificultatea rezolv
arii problemelor de "matematic
a
elementara", precum si tendinta accentuata de a mbina algebra, teoria numerelor si
analiza matematica n actul de concepere si rezolvare a unor asemenea perle matematice.
1. Un prim exemplu este urmatoarea problema propusa la unul din testele de
selectie pentru OIM n anul 2002, in Vietnam. n rezolvare vom folosi doar cteva
rezultate legate de ecuatia de gradul al doilea. Dup
a cum se stie, multe probleme
dificile se rezolv
a relativ usor folosind trinomul de gradul al doilea (metoda coborrii).
Vom da doar doua exemple, fara a insista prea mult.
a
b
c
1) Aratati ca daca numarul d =
+
+
este ntreg, iar a, b, c sunt numere
bc ca ab
naturale, atunci d este 1 sau 3.
2) Aratati ca, daca numerele naturale distincte si nenule a1 , a2 , . . . , an verifica
a21 + a22 + + a2n = na1 a2 an , atunci ele sunt prime ntre ele doua cte doua.
ncercati sa rezolvati aceste doua probleme nainte de abordarea problemei 1.
PROBLEMA 1. Sa se demonstreze ca exista un numar m 2002 si m numere
m
n
Y
X
a2i 4
a2i sa fie patrat
naturale nenule a1 , a2 , . . . , am , distincte, astfel nct
i=1

i=1

perfect.
Solutie. Vom folosi trinomul pentru a crea solutii pentru anumite ecuatii diofantice, deci n mod constructiv.
m
Y
Ar fi bine s
a dispar
a
a2i . Deci, s
a scriem expresia sub forma
i=1

m
Y

i=1

a2i

n
X

a2i

i=1

m
Y

i=1

ai k

!2

Pentru a "scapa" si de 4, luam k = 2. Asadar am adus problema la o forma mai


"acceptabila" (dar nu mai putin dificila):
Aratati ca exista m 2002 si a1 , a2 , . . . , am N distincte astfel nct
1 + a21 + a22 + + a2m = a1 a2 am .

(1)

Sa cautam m astfel nct m 3 dintre necunoscutele ecuatiei (1) sa fie 1. Aceasta


revine la ecuatia
(2)
x2 + y 2 + z 2 + m 2 = xyz.
Privind aceasta ecuatie ca una de gradul al doilea
n
z,
vom
ncerca
s
a
lu
a
m
dis

criminantul nul. Deci x2 y 2 = 4 x2 + y 2 + m 2 . Luam x = 2a, y = 2b si obtinem


1

Student, Facultatea de Matematic


a-Informatic
a, Bucuresti

16

m = 4 a2 1 b2 1 2.
a concluzion
am: putem alege b > a > 2002 diferite si
2S

2
putem lua m = 4 a 1 b 1 2 > 2002. Atunci ecuatia (2) va avea solutiile
(x, y, z) = (2a, 2b, 2ab). Rezult
a c
a ecuatia (1) are solutia (2a, 2b, 2ab, 1, 1, . . . , 1).
Dar putem scrie (1) si sub forma
12 1 2a 2b 2ab + (2a)2 + (2b)2 + (2ab)2 + m 3 = 0.

(3)

Din relatiile lui Vite rezult


a c
a si 2a 2b 2ab 1 este solutie a ecuatiei (3), n
care n loc de 1 punem t. Asadar am redus cu o unitate num
arul celor m 3 de 1 si
am obtinut o noua solutie a ecuatiei (1): (2a, 2b, 2ab, 2a 2b 2ab 1, 1, 1, . . . , 1).
Analog, scriem
2

12 1 2a 2b 2ab (2a 2b 2ab 1) + (2a) + (2b) + (2ab) +


2

+ (2a 2b 2ab 1) + m 4 = 0.

Deci obtinem o alta solutie a ecuatiei (1), cu numar si mai mic de 1:


(2a, 2b, 2ab, 2a 2b 2ab 1, 2a 2b 2ab (2a 2b 2ab 1) 1, 1, 1, . . . , 1) .

Astfel, rezulta ca putem elimina pe rnd fiecare 1 din m-upla (2a, 2b, 2ab, 1, 1, . . . , 1).
Riguros, aceasta nseamna ca folosind succesiv relatiile lui Vite, obtinem cte o ma 2a = x1 < 2b = x2 < 2ab = x3 <
upl
a (x1 , x2 , . . . , xk , 1, 1, . . . , 1) n care este clar c
< < xk . La sfrsit (c
aci dup
a cel mult m pasi am eliminat toti de 1), obtinem o
solutie (a1 , a2 , . . . , am ) a ecuatiei (1), n care a1 < a2 < < am . Aceasta m-upla
va satisface conditiile enuntului.
2. Continuam cu o frumoasa problema propusa la ultima runda a olimpiadei
poloneze n anul 2003. Simplitatea solutiei care urmeaza nu are nsa nici o legatura
cu dificultatea problemei, c
aci multe metode de atacare a problemei nu duc la nici
un rezultat.
PROBLEMA 2. Determinati polinoamele cu coeficienti ntregi f cu proprietatea ca pentru orice n natural avem f (n) | 2n 1.
Solutie. Evident, problema ar fi banala daca s-ar demonstra ca exista o infinitate
de numere n pentru care 2n 1 este numar prim. Dar, dupa cum vom vedea, problema
accept
a si solutii mai "blnde".
Cum este clar c
a nu putem afla prea multe despre divizorii si factorii primi ai lui
2n 1, vom ncerca sa lucram cu divizori ai numerelor de forma f (n). Primul lucru
care ne vine n minte, tinnd seama ca f are coeficienti ntregi, este sa folosim
rezultatul urm
ator: m n | f (m) f (n). Deci, va trebui s
a c
aut
am m si n
astfel nct f (m) | f (n). Dup
a c
aut
ari mai mult sau mai putin lungi, g
asim c
a
f (n) = n + f (n) n | f (n + f (n)) f (n). Deci f (n) | f (n + f (n)).
n acest moment, jumatate din problema este rezolvata. ntr-adevar, schimbnd
f cu f , putem presupune c
a f are coeficientul dominant pozitiv. Atunci exist
aM
astfel nct pentru n > M s
a avem f (n) N. Fix
am un n > M . Avem f (n) | 2n 1 si
f (n) | f (n + f (n)) | 2n+f (n) 1 = (2n 1) 2f (n) + 2f (n) 1 (evident, n + f (n) N),
deci f (n) | 2f (n) 1. Daca am putea demonstra ca singurul numar natural n pentru
care n | 2n 1 este 1, atunci ar rezulta c
a pentru n > M avem f (n) = 1, adic
af
ar fi constanta 1. Dar faptul c
a n | 2n 1 implic
a n = 1 este binecunoscut si destul
de simplu. Sa presupunem ca n > 1 si sa luam p cel mai mic factor prim al lui n.
17

Atunci este clar c


a (n, p 1) = 1. Dar p | n | 2n 1 si p | 2p1 1 (teorema lui
Fermat). Deci p | 2n 1, 2p1 1 . Se stie ca sirul (xn )n1 , xn = 2n 1 este sir
Mersenne (adica (xm , xn ) = x(m,n) ). Rezulta ca p | (xn , xp1 ) = x(n,p1) = x1 = 1,
contradictie. Asadar n = 1 si f este constanta 1. Cum, dac
a f este solutie, atunci si
f este solutie, deducem c
a polinoamele cerute sunt constantele 1 si 1.
3. ncheiem scurta incursiune prin matematica elementara cu o problema extrem
de dificila, propusa la un test de selectie n Vietnam, 2002. Dificultatea problemei
const
a mai ales n faptul c
a admite multe solutii (care nici nu se intrez
aresc usor), iar
frumusetea const
a n mbinarea algebrei cu analiza matematic
a si teoria numerelor.
Nu exageram daca afirmam ca urmatoarea problema este una dintre cele mai dificile
si frumoase probleme referitoare la polinoame, propuse la vreun concurs pentru elevi.
PROBLEMA 3. Determinati toate polinoamele
p Z [X] cu proprietatea ca

exista un polinom q Z [X] pentru care q 2 (X) = X 2 + 6X + 10 p2 (X) 1.


Solutie. Evident,
orice rezolvitor "srguincios" va scrie relatia din enunt sub
forma q 2 (X 3) = X 2 + 1 p2 (X 3) 1 si va nota f (X) = p(X 3), g(X) =
= q (X 3). Deci

2
(1)
X + 1 f 2 (X) = g 2 (X) + 1.
Aici este ns
a punctul de oprire, c
aci orice ncercare ulterioar
a de rezolvare esueaz
a.
Ca de obicei, vom putea presupune ca f si g au coeficientii dominanti pozitivi (caci
putem schimba f cu f sau g cu g, f
ar
a a se modifica nimic). Deci exist
a M astfel
nct pentru orice n > M s
a avem f (n) , g (n) N.
Apel
am acum la teoria numerelor. Este binecunoscut faptul c
a toate solutiile n
numere naturale ale ecuatiei Pell x2 + 1 = 2y 2 sunt date de
2n1
2n1
2n1
2n1

+ 1 2
1 2
1+ 2
1+ 2

xn =
.
, yn =
2
2 2

Ce se ntmpla daca substituim xn n (1)? Obtinem g 2 (xn ) + 1 = 2 (yn f (xn ))2 .


Da, si perechea (g (xn ) , yn f (xn )) este solutie a ecuatiei Pell si aceasta se ntmpl
a
pentru orice n > M . Deci exist
a sirurile (an )n>M , (bn )n>M astfel nct g (xn ) = xan ,
yn f (xn ) = ybn .
Acum ncepe partea analizei matematice. Fie grad g = k , grad f = m. Avem

2an 1k(2n1)
2xan
= lim
lim 1 + 2
k(2n1) =
n
n
1+ 2

k
xn
g (xn )
= 2 lim
(2n1) = finit.

n xk
n
1+ 2

Rezulta ca sirul de numere ntregi (2an 1 k (2n 1))n>M este convergent, deci
stationar.
Asadar, exist
a n0 > M astfel nct pentru n > n0 s
a avem
2an 1 k (2n 1) = u, pentru o constant
a ntreag
a u. Ca urmare, pentru n > n0
avem

2n1 !
k(2n1)+u
k(2n1)+u
2n1
+ 1 2
+ 1 2
1+ 2
1+ 2
=
.
g
2
2
18

Rezult
a c
a

u
u

1
1
k
x 1+ 2 +
1 2
x
x

x
g
(2)
=
2
2
n
o
2n1
| n > n0 . Aducnd la acelasi numitor
pentru orice x din multimea 1 + 2
n (2), obtinem o identitate polinomiala adevarata pentru o infinitate de valori ale
variabilei, deci (2) este adev
arat
a pentru orice x nenul. Dup
a ce aducem
la
u acelasi

numitor si egalam coeficientii dominanti n (2), deducem ca 2k1 1 + 2 = k ,


unde k este coeficientul dominant al lui g. Dar aceasta implic
a u = 0. Asadar,
pentru orice x nenul, avem

1
1
k
x +
x
x

x
g
.
(3)
=
2
2

t2 + 1, din (3) obtinem c


a pentru orice t avem

k
k
t + 1 + t2 + t t2 + 1
g(t) =
.
(4)
2
Lu
am n (1) x = i si obtinem c
a g 2 (i) = 1. Deci, folosind (4), obtinem i2k = 1,
adic
a k este impar. Din (4) si (1) rezult
a prin calcul c
a
"

k
k #2
X + X2 + 1 + X2 + 1 X
2

f (X) =
, k impar.
(5)
2 X2 + 1
Dac
a not
am x = t +

Cum f este polinom si are coeficientul dominant pozitiv, deducem din (5) ca

k
k
X + X2 + 1 + X2 + 1 X

f (X) =
.
(6)
2 X2 + 1
Dar, daca f verifica (1), atunci si f verifica aceeasi relatie. Mai mult, polinomul
din membrul drept al relatiei (6) are coeficienti ntregi. Rezulta ca exista doua tipuri
de polinoame care verific
a relatia (1)

k
k

X + X2 + 1 + X2 + 1 X

, k impar.
(7)

2 X2 + 1
n sfrsit, obtinem c
a polinoamele p cerute se obtin din polinoamele (7) nlocuind
X cu X + 3.
Ce-ar mai fi de ad
augat dup
a prezentarea acestor trei nestemate din siragul nesfrsit al problemelor elementare de matematic
a? S
lefuite cu r
abdarea bijutierului,
cele trei probleme adaug
a o palet
a de lumini ncepnd cu actul creator al conceperii
lor si terminnd cu solutiile propuse. Fiecare dintre noi are nevoie de asemenea perle,
iar aceast
a scurt
a prezentare se nscrie pe aceast
a linie.

19

n leg
atur
a cu o problem
a de concurs
Dan Stefan
MARINESCU 1

La etapa finala a Olimpiadei de matematica din anul 1989 prof. univ. dr. T. Precupanu a propus urmatoarea problema:
Z b
Daca f : [a, b] R este o functie integrabila, continua pe (a, b) si

f (x) dx 6= 0,

a
x1 , x2 , . . . , xn

atunci pentru fiecare n N exista n numere distincte


Z b
ca
n (b a)
f (x) dx =
.
1
1
1
a
+
+ +
f (x1 ) f (x2 )
f (xn )

(a, b) astfel
(1)

(enunt partial)
Enuntul si o solutie a problemei pot fi aflate n [3]. n cele ce urmeaz
a vom
prezenta o generalizare a acestei frumoase probleme.
Pentru ceea ce ne-am propus, avem nevoie de
Propozitia 1. Fie f, g : [0, 1] R doua functii cu urmatoarele proprietati:
i) f , g continue pe [0, 1],
ii) f , g derivabile pe (0, 1),
iii) f (1) 6= f (0) si g 0 (x) 6= 0, x (0, 1).
Atunci pentru orice n N si orice 1 , 2 , . . . , n > 0 cu 1 + 2 + + n = 1
exista x1 , x2 , . . . , xn (0, 1) cu x1 < x2 < < xn astfel nct
n
X
g 0 (xi )
g (1) g (0)
i 0
=
.
(2)
f (xi )
f (1) f (0)
i=1

f (x) f (0)
; evident h este continu
a
f (1) f (0)
pe [0, 1], derivabila pe (0, 1) si h (0) = 0, h (1) = 1.
Pentru orice k {1, 2, . . . , n 1} consideram functia continua hk : [0, 1] R,
k
P
hk (x) = h (x)
i . Cum h1 (0) = 1 < 0, h1 (1) = h (1) 1 = 1 1 > 0,
Demonstratie. Fie h : [0, 1] R, h (x) =

i=1

conchidem, din continuitatea functiei h1 , ca exista c1 (0, 1) cu h1 (c1 ) = 0


h (c1 ) = 1 . Analog, h2 (c1 ) = h (c1 ) 1 2 = 2 < 0, h2 (1) = h (1) 1 2 =
= 1 1 2 > 0, de unde acelasi rationament conduce la existenta unui c2 (c1 , 1)
astfel nct h2 (c2 ) = 0 h (c2 ) = 1 + 2 . Inductiv, gasim 0 < c1 < c2 < <
< cn1 < 1 astfel nct
h (c1 ) = 1 ,

h (c2 ) = 1 + 2 ,

...

, h (cn1 ) = 1 + 2 + . . . + n1 .

(3)

a
Fie c0 = 0 si cn = 1, atunci pentru orice k {1, 2, . . . , n} functiile h si g verific
conditiile din teorema lui Cauchy pe intervalul [ck1 , ck ]; ca urmare, deducem c
a
exista x1 (c0 , c1 ), x2 (c1 , c2 ), . . . , xn (cn1 , cn ) astfel nct
h0 (x1 )
h (c1 ) h (c0 ) h0 (x2 )
h (c2 ) h (c1 )
h0 (xn )
h (cn ) h (cn1 )
=
,
=
,
.
.
.
,
=
,
g 0 (x1 )
g (c1 ) g (c0 ) g 0 (x2 )
g (c2 ) g (c1 )
g 0 (xn )
g (cn ) g (cn1 )
de unde, mpreun
a cu (3), avem:
1

Profesor, Liceul Teoretic "Iancu de Hunedoara", Hunedoara

20

h0 (x1 )
2
h0 (x2 )
n
h0 (xn )
1
= 0
,
= 0
, ...,
= 0
g (c1 ) g (c0 )
g (x1 ) g (c2 ) g (c1 )
g (x2 )
g (cn ) g (cn1 )
g (xn )
ceea ce, tinnd seama de faptul ca 1 , 2 , . . . , n > 0, conduce la relatiile
g 0 (x1 )
g 0 (x2 )
g 0 (xn )
1 0
= g (c1 )g (c0 ) , 2 0
= g (c2 )g (c1 ) , . . . , n 0
= g(cn )g(cn1 ) .
h (x1 )
h (x2 )
h (xn )
n
X
g 0 (xi )
De aici, prin adunare, obtinem
i 0
= g (cn ) g (c0 ) = g (1) g (0). Cum
h (xi )
i=1
f 0 (x)
h0 (x) =
, x (0, 1), conchidem c
a are loc relatia (2).
f (1) f (0)
Corolarul 1 [1]. Daca f : [0, 1] R este o functie continua pe [0, 1], derivabila
pe (0, 1), f (0) = 0, f (1) = 1, atunci pentru orice n N si orice k1 , k2 , . . . , kn > 0
n
n
X
X
ki
exista x1 , x2 , . . . , xn (0, 1) distincte doua cte doua astfel nct
ki
=
f 0 (xi )
i=1
i=1
(vezi si [2], [4]).
n
.X
Demonstratie. Consideram n Propozitia 1, i = ki
ki , pentru orice
i=1
i {1, 2, . . . , n} si g : [0, 1] R, g (x) = x.
Corolarul 2. Fie f, g : [a, b] R dou
a functii integrabile, continue pe (a, b),
Z b
f (x) dx 6= 0 si g (x) 6= 0, x (a, b). Atunci, pentru fiecare n N si 1 , 2 , . . . ,
a

a n numere distincte x1 , x2 , . . . , xn (a, b)


n > 0 cu 1 + 2 + + n = 1, exist
astfel ca
Z b
Z
n
b
X g (xi )
i
g (x) dx
f (x) dx.
(4)
=
f (xi )
a
a
i=1
Z x
Demonstratie. Fie f1 , g1 : [0, 1] R, f1 (x) =
f ((1 t) a + tb) dt, g1 (x) =
Z x
0
g ((1 t) a + tb) dt. n mod evident, f1 si g1 sunt bine definite, verific
a conditiile
=
Z b
Z b
0
1
1
g (x) dx, f1 (1)f1 (0) =
f (x) dx.
din Propozitia 1 si g1 (1)g1 (0) =
ba a
ba a
De unde exist
a c1 < c2 < < cn din (0, 1) astfel nct
Rb
Rb
n
n
X
X
g (x) dx
g (x) dx
g10 (ci )
g ((1 ci ) a + ci b)
a
i 0
i

.
= Rb
= R ab
f
(c
)
f
((1

c
)
a
+
c
b)
i
i
f (x) dx
f (x) dx
1 i
i=1
i=1
a
a
a x1 < x2 < < xn si obtinem
Lund xi = (1 ci ) a + ci b, i = 1, n, evident c
formula (4).
Observatie. Egalitatea (1) se obtine lund n (4) 1 = 2 = = n = 1/n si
g (x) = 1, x [a, b].
1.
2.
3.
4.

Bibliografie
G. G. Z. Giang - Problema 1125, Math. Mag.
P. Orno - Problema 1053, Math. Mag.
I. Tomescu (coordonator) - Probleme date la olimpiadele de matematica pentru
licee (19501990), Ed. stiintifica, Bucuresti, 1992.
***
- Problema C:1791, G. M. 3/1996.
21

Asupra unei probleme propuse la O. I. M. - 1982


Neculai ROMAN 1
La O. I. M. n anul 1982 a fost propus
a problema B3 GB:
Fie ABC un triunghi si P un punct n interiorul lui astfel ca P AC P BC.
Fie L, M picioarele perpendicularelor din P pe BC, CA respectiv. Fie D mijlocul
lui [AB]. Sa se demonstreze ca DL = DM .
Enuntul si o solutie a acestei probleme se poate g
asi n [1], pag. 322 si 333. Problema are si o solutie mai simpl
a, accesibil
a si elevului de gimnaziu si care merit
a
a fi cunoscuta. De asemenea, vom arata ca problema are loc pentru o multime mai
variat
a de puncte din planul triunghiului. n acest scop, vom demonstra urm
atoarea
0

Teorem
a. Fie ABC un triunghi, D mijlocul lui [AB] si punctele A , B pe
0
dreptele AC, respectiv BC astfel ca C (AA ) si C (BB 0 ). Fie P un punct
n planul triunghiului si L, M picioarele perpendicularelor din P pe BC, respectiv AC. Sa se demonstreze ca daca P Int (ACB) Int (A0 CB 0 ) astfel ca
P AC P BC sau daca P Int (BCA0 ) Int (ACB 0 ) astfel ca m (P AC) +
+m (P BC) = 180 atunci DM = DL.
Demonstratie. Fie punctele D0 si D00 mijloacele segmentelor [P A], respectiv
[P B] (fig. 1, 2 si 3).

D
M

D
B

CB

D
A

P
M

Fig. 1

L B L

Fig. 3

Fig. 2

PA
atoare ipotenuzei n
= DD00 , ([M D0 ] mediana corespunz
2
00
4AM P si [DD ] linie mijlocie n 4AP B).
Deci
[M D0 ] [DD00 ] .
(1)
Analog,
[LD00 ] [DD0 ] .
(2)
0
00
Din P D DD paralelogram, rezulta ca
P D0 D P D00 D
(3)
Avem M D0 =

Profesor, S
coala "V. Alecsandri", Mircesti, Iasi

22

Din P Int (ACB) Int (A0 CB 0 ) astfel ca P AC P BC (fig. 1 si 2) deducem


ca P D0 M P D00 L (teorema unghiului exterior).
Daca P Int (BCA0 ) astfel ca m (P AC) + m (P BC) = 180 (fig. 3), atunci
P AC P BL si deci P D0 M P D00 L (teorema unghiului exterior). Dac
a
P Int (ACB 0 ) astfel ca m (P AC)+m (P BC) = 180 , atunci P BC P AM
si deci P D0 M P D00 L.
n concluzie,
P D0 M P D00 L.
(4)
Din relatiile (3) si (4) rezult
a c
a
M D0 D LD00 D.
0

(5)
00

Acum din relatiile (1), (2) si (5) rezulta ca 4M D D 4DD L, de unde obtinem
[DM ] [DL] si deci DM = DL.

Teorema reciproc
a. Fie ABC un triunghi, D mijlocul lui [AB] si punctele A0 ,
B pe dreptele AC respectiv BC astfel ca C (AA0 ) si C (BB 0 ). Fie punctele L,
M pe dreptele BC respectiv AC astfel ca DM = DL. Perpendicularele n M si L pe
AC respectiv BC se ntlnesc n P . Sa se demonstreze afirmatiile:
a) daca P Int (ACB) Int (A0 CB 0 ), atunci P AC P BC;
b) daca P Int (ACB 0 )Int (BCA0 ), atunci m (P AC)+m (P BC) = 180 .
Demonstratie. a) Fie punctele D0 si D00 mijloacele segmentelor [P A] respectiv
[P B].
Se arata usor ca 4M D0 D 4DD00 L, de unde rezulta ca
0

00

M D0 D DD00 L.

(6)

P D0 D P D00 D.

(7)

P D0 M P D00 L.

(8)

Din P D DD paralelogram, rezult


a
Din relatiile (6) si (7) obtinem

a) Daca P Int (ACB)Int (A CB ), atunci din (8) rezulta P AC P BC


(fig.1 si 2).
b) Dac
a P Int (BCA0 ) (fig. 3), atunci din relatia (8) rezult
a c
a P AC P BL
si, deci, m (P AC) + m (P BC) = 180 .
Dac
a P Int (ACB 0 ), atunci din relatia (8) rezult
a: P BC P AM . Deci
m (P AC) + m (P BC) = 180 .
0

Bibliografie
1. I. Cuculescu - Olimpiadele internationale de matematica ale elevilor, Ed.
Tehnic
a, Bucuresti, 1984.

23

Asupra unei ecuatii functionale


Loredana AGORE 1
Scopul acestei note este rezolvarea ecuatiei functionale
f (axy + x + y) = bf (x) f (y) + c [f (x) + f (y)] + d (a, b, c, d R)

(1)

f (x + y) = f (x) + f (y) ,

(2)

n multimea functiilor f : R R sau f : R R sau f :


n ecuatia (1) este cuprins
a ecuatia lui Cauchy

R+

R.

ale c
arei solutii se numesc functii aditive, precum si urm
atoarele ecuatii clasice ce
sunt reductibile la ecuatia lui Cauchy:
f (x + y) = f (x) f (y) ,
(3)
f (xy) = f (x) + f (y) ,
(4)
f (xy) = f (x) f (y) .
(5)
Rezolvarea n detaliu a ecuatiilor (2) (5) se poate gasi n [1]. Tot n [1], p. 23, este
studiat
a si ecuatia functional
a obtinut
a considernd n (1) a 0, c = 1, d = 0.
Rezolvarea ecuatiei functionale (1) const
a n reducerea ei, potrivit cu parametrii
a, b, c si d, la una dintre ecuatiile (2) (5). Distingem cteva cazuri.
I a 6= 0, b 6= 0. nmultim ecuatia (1) cu b si apoi punem rezultatul obtinut sub
forma

(ax + 1) 1
(ax + 1) (ay + 1) 1
+ c = bf
+c
bf
a
a

(ay + 1) 1
bf
+ c + bd + c c2 . (6)
a

t1
2
Cu notatiile = bd + c c , u = ax + 1, v = ay + 1 si g(t) = bf
+ c, t R,
a
ecuatia (6) se scrie
g (uv) = g (u) g (v) + .
(7)
Dac
a bd = c2 c, adic
a = 0, atunci (7) este de tipul (5) si se reduce la ecuatia
lui Cauchy [1]. Solutiile se exprim
a cu functiile aditive sau sunt functii constante.
Daca bd 6= c2 c, deci 6= 0, luam u = v = 1 n (7) si obtinem
g (1) = [g (1)]2 + .

1
1
1
De aici, g (1) =
1 1 4 =
1 (1 2c)2 4bd , cu conditia ca
2
2
4
1
bd c2 c + . Pe de alt
a parte, ecuatia (7) cu v = 1 devine
4

g (u) = g (1)
g (u) = g (u) g (1) + g (u) =
1 g (1)
(evident, g (1) 6= 1). Ca urmare,ecuatia (1) are, n cazul considerat,
doua solutii

q
1
1
2
date de f (x) = [g (1) c] =
(1 2c) (1 2c) 4bd .
b
2b
1

Elev
a, cl. a XI-a, Colegiul National "Mihai Viteazul", Bucuresti

24

II a = 0, b 6= 0. Ecuatia (1) devine

f (x + y) = bf (x) f (y) + c [f (x) + f (y)] + d

(8)

si se poate scrie n forma


bf (x + y) + c = [bf (x) + c] [bf (y) + c] + bd + c c2

(9)

g (x + y) = g (x) g (y) + .

(10)

sau, notnd = bd + c c si g (t) = bf (t) + c, t R,


2

Dac
a bd = c c, atunci (10) este de tipul (3) etc.
2
Daca bd 6= c2c, n (8) luam x = y = 0 si ob
tinem f (0) = b [f (0)] +2cf (0)+d (),
q
1
deci f (0) =
(1 2c) (1 2c)2 4bd (n mod necesar, (1 2c)2 4bd 0
2b
1 4 0). Dar, daca n (8) luam y = 0 si apoi grupam convenabil, avem
()

[1 bf (0) c] f (x) = cf (0) + d


[1 bf (0) c] f (x) = [1 bf (0) c] f (0) f (x) = f (0)
()

(1 bf (0) c 6= 0 bd 6= c2 c). Avem dou


a solutii:

q
1
f (x) = f (0) =
(1 2c) (1 2c)2 4bd .
2b
III a 6= 0, b = 0. n acest caz, (1) se scrie

f (axy + x + y) = c [f (x) + f (y)] + d.

(11)

Dac
a c = 1, punem (11) n forma

(ax + 1) (ay + 1) 1
(ax + 1) 1
(ay + 1) 1
f
+d = f
+d + f
+d
a
a
a
t 1
+ d, t R,
sau, notnd u = ax + 1, v = ay + 1 si g (t) = f
a
g (uv) = g (u) + g (v) ,
care este o ecuatie de tipul (4).
Dac
a c 6= 1, pentru x = y = 0 luat n (11) obtinem

f (0) = 2cf (0) + d (1 2c) f (0) = d.

(12)

Cum, pentru y = 0 n (11), avem

(12)

f (x) = cf (x) + cf (0) + d f (x) = cf (x) + f (0) cf (0)


d
(12)
(1 c) f (x) = (1 c) f (0) f (x) = f (0) f (x) =
1 2c
1
dac
a mai presupunem n plus c 6= . Este banal
a verificarea faptului c
a, n conditiile
2
d
impuse, f (x) =
este solutie a ecuatiei (11).
1 2c
1
Daca c = , avem de rezolvat ecuatia
2
1
(13)
f (axy + x + y) = [f (x) + f (y)] + d.
2
25

1
[f (0) + f (0)] + d, deci d = 0. Lund acum
2

1
1
1
f (x) + f
, de unde rezulta
y = n (13) cu d = 0, vom avea f (0) =
a
2
a
ca f (x) = k (constant), x R. Se verifica usor ca aceasta functie este ntr-adevar
solutie pentru orice k R.
Pentru x = y = 0, obtinem f (0) =

IV a = 0, b = 0. Este vorba de ecuatia


f (x + y) = c [f (x) + f (y)] + d.
(14)
Daca c = 1, (14) se poate scrie
f (x + y) + d = [f (x) + d] + [f (x) + d] ,
care este o ecuatie Cauchy n g (t) = f (t) + d, t R.
Dac
a c 6= 1, lu
am x = y = 0 n (14) si obtinem, ca si n cazul precedent,
relatiile echivalente (12). Se continua tot ca n cazul amintit si se obtine solutia
1
d
, x R, dac
a c 6= .
f (x) = f (0) =
1 2c
2
1
Dac
a c = , (14) se scrie
2
1
(15)
f (x + y) = [f (x) + f (y)] + d.
2
Lund x = y = 0, constat
am c
a d = 0. Punnd n (15) d = 0 si fixnd y = 0,
1
obtinem f (x) = [f (x) + f (0)]. Deci f (x) = f (0) = constant, x R, functie ce
2
este solutie a ecuatiei (15).

Bibliografie
1. V. Pop - Ecuatii functionale. Ecuatii clasice si probleme, Ed. Mediamira, ClujNapoca, 2002.

Recreaii matematice
2. Un calator, care nu avea la el dect un lant cu sapte verigi de aur, poposeste
ntr-o zi la un han. El se ntelege cu hangiul sa-l plateasca pentru fiecare zi petrecuta
la han cte o verig
a de aur. Dac
a st
a sapte zile si plata trebuie f
acut
a n fiecare zi,
care este num
arul minim de t
aieturi care trebuie f
acute n lant pentru a putea pl
ati
pretul convenit? (Se accepta ca, atunci cnd este cazul, hangiul sa dea calatorului
ca rest un numar de verigi (posibil toate!) pe care le-a primit deja.)
3. Care este eroarea n "demonstratia" de mai jos a egalitatii 3 = 0?

x2 x + 1 = 0 | x
x3 (1) = 0

3
2

x x
x3 = 1

2+ x =
0
3

x = 1
x x x =0
2
Punnd x = 1 n x x + 1 = 0, obtinem 3 = 0.
Not
a. Solutiile problemelor 2 si 3 se pot g
asi la pagina 39.
26

Asupra unei inegalit


ati conditionate
Cezar LUPU 1
La OBM - 2001 a fost dat
a problema urm
atoare:
Fie a, b, c numere
reale
strict
pozitive
astfel
nct a + b + c abc. Sa se arate ca

a2 + b2 + c2 abc 3.
Cristinel Mortici, Romnia
Solutia autorului utilizeaza metoda reduceriila absurd. Presupunem ca are loc
inegalitatea contrar
a, adic
a a2 + b2 + c2 < abc 3. Aplicnd inegalitatea Cauchy

1
1
2
2
Schwarz, obtinem abc 3 > a2 +b2 +c2 (a + b + c) (abc) , de unde abc < 3 3.
3
3
Pe de
alt
a parte, aplicnd
inegalitatea mediilor, avem abc 3 > a2 + b2 + c2

3
2
2
2
3 a b c si, deci, abc > 3 3. Se obtine astfel o contradictie.
Alte solutii ale acestei probleme sunt prezentate n [1] si [2].
1. Problema de mai sus poate fi nt
arit
a astfel:
Problema 1. Sa se arate ca, daca a, b, c > 0 si a + b + c > abc, atunci
ab + bc + ca abc 3.
1
1
1
Solutia I. Ipoteza si concluzia se pot scrie n felul urm
ator:
+ +
1 si
ab bc ac

1 1 1
2
am binecunoscuta inegalitate (x + y + z) 3 (xy + yz + zx),
+ + 3. Utiliz
a b c

1 1 1
1
1
1
x, y, z R (1) pentru a obtine
3
+ +
+
+
, din care rezult
a
a b
c
ab bc ac

1 1 1
ca + + 3.
a b
c
Solu
t
ia
II. Inegalitatea ceruta rezulta direct din inegalitatea ab + bc + ca
p
3abc (a + b + c), a, b, c R+ (2) (aceasta se reduce la (1), daca notam x = ab,
y = bc si z = ca).
Problema 2. Se considera a, b, c >
0 astfel nct a + b + c abc. Sa se arate ca
bc
3
ca
ab
+ 2
+ 2

.
Cezar Lupu
2
a (b + c) b (c + a) c (a + b)
2
X
Solutie. Pentru prescurtarea scrierii folosim nsumarea ciclica
. Avem

2
2
X
1 X (bc)
1
(bc + ca + ab)
ab + bc + ca
3
bc
=

2
a (b + c)
abc
a (b + c)
abc 2 (bc + ca + ab)
2abc
2
X (bc)2
2
(s-a folosit (bc + ca + ab) 2 (bc + ca + ab)
, adev
arat
a conform inegaa (b + c)
litatii Cauchy-Schwarz).
2. Avnd ca punct de plecare inegalitatea conditionata data la OBM, se pot
obtine inegalit
ati geometrice ntr-un triunghi. S
a observ
am mai nti c
a avem:
Daca a, b, c sunt lungimile laturilor unui triunghi oarecare nscris ntr-un cerc de
raza egala cu unitatea, atunci a + b + c abc.
n [2] sunt date patru demonstratii. Reproducem una dintre ele. Formulele
abc
4RS = abc si S = pr conduc la relatia
= 2Rr. Utiliznd inegalitatea
a+b+c
lui Euler si faptul c
a R = 1, obtinem inegalitatea dorit
a.
1

Elev, cl. a X-a, Colegiul National "Mircea cel B


atrn", Constanta

27

Ca urmare, n conditia impus


a triunghiului, are loc si inegalitatea
a2 + b2 + c2 abc 3. De
altfel, aceasta din urma rezulta direct din cunoscuta
inegalitate a2 + b2 + c2 4S 3 (Weitzenbck, 1919) pentru R = 1.
O nt
arire a acestor inegalit
ati este dat
a de
Problema 3.
n oricetriunghieste satisfacuta inegalitatea
b ca +c ab 4S
ab + bc + ca
a bc +
3. n particular, daca R = 1, avem
ab + bc + ca a bc + b ca + c ab abc 3.
X
X
Solutie. Prima parte a dublei inegalit
ati se dovedeste astfel:
ab
a bc
1
1
1
1
1
1
+ + + + A2 + B 2 + C 2 AB + BC + CA (s-a notat
a
b
c
ca
ab
bc
1
1
1
A = , B = si C = ), care este adevarata.

a
c
b
X 1
X

3

.
Demonstr
am acum partea a doua, adic
a
a bc 4S 3 sau
R
bc
Cu inegalitatea Cauchy-Schwarz sau utiliznd inegalitatea (2) de mai sus, obtinem

X 1
9
9
9
3
X p
. Este suficient ca p

R
bc
3 (ab + bc + ca)
3 (ab + bc + ca)
bc

sau, echivalent, ab + bc + ca 9R2 . Aceasta decurge din ab + bc + ca a2 + b2 + c2


si faptul cunoscut c
a ntr-un triunghi are loc a2 + b2 + c2 9R2 .
Problema 4. Sa se arate ca n orice triunghi nscris ntr-un
cerc de raza egala

a
1 1 1
b
c
cu 1 are loc inegalitatea
+
+
+ 32
+ +
.
Cezar Lupu
bc ca ab
a b
c

2
Solutie. Este binecunoscut
a inegalitatea a2 + b2 + c2 4S 3 + (a b) +
si Hadwiger, 1938). Este echivalenta cu 2 (ab + bc + ca)
+ (b c)2 +(c a)2 (Finsler

a2 + b2 + c2 4S 3 si tinnd seama c
a R = 1, conduce la inegalitatea cerut
a.

Problema 5. n orice triunghi nscris ntr-un cerc de raza 1 are loc urmatoarea
a
b
c
3
inegalitate:
+
+

.
Cezar Lupu
bc (b + c) ca (c + a) ab (a + b)
2 (a + b + c)
Solutie. Utiliznd rezultatul
din Problema
3 si inegalitatea
Cauchy-Schwarz,

X (a bc)2
(a bc + b ca + c ab)2
3a2 b2 c2

, de unde, prin
putem scrie:
2 (a + b + c)
2 (a + b + c)
b+c
a.
mp
artire cu a2 b2 c2 , obtinem inegalitatea dorit
3. Propunem spre rezolvare urmatoarele probleme:
1. Se consider
a a, b, c trei numere reale strict pozitive
astfel nct a + b + c abc.

b2
c2
3 3
a2
+
+

.
Cezar Lupu, G.M.
Aratati ca
bc (b + c) ca (c + a) ab (a + b)
2
2. Fie ABC un triunghi
oarecare nscris ntr-un cerc de raza egala cu 1. Aratati

3
AB BC CA, P Int (ABC).
Cezar Lupu
c
a PA + PB + PC
3
Bibliografie
1. M. B
alun
a si M. Becheanu (prezentare de) - A 18-a OBM, 3-9 mai 2001, Belgrad,
GM - 5-6/2001, 229-236.
2. Cezar Lupu - Asupra unei probleme de concurs, Rev. Mate(matic
a), 2003, 17-20.
28

O metod
a de demonstrare a concurentei unor drepte
Gabriel POPA, Paul GEORGESCU 1
Vom exemplifica n cele ce urmeaza aplicabilitatea unei metode de demonstrare a
concurentei unor drepte, prea putin utilizata n contextul introducerii noii programe
scolare de geometrie.
Date dou
a puncte A, B avnd vectorii de pozitie rA si respectiv rB , vectorul de
pozitie al unui punct al dreptei AB este de forma
rM = rA + (1 ) rB , R

(ecuatia vectoriala a dreptei AB). Avnd o dreapt


a atasat
a unui triunghi, vectorul de
pozitie al unui punct curent M al s
au poate fi exprimat functie de vectorii de pozitie
ai vrfurilor si de un parametru real . Considernd nca o dreapta (cu parametrul
notat ), pentru a afla punctul comun celor doua drepte vom avea de rezolvat un
sistem liniar n si .
Dac
a dorim s
a prob
am concurenta a trei drepte, le vom intersecta dou
a cte
doua si vom urmari daca vectorii de pozitie ai punctelor obtinute coincid. Metoda
presupune, n general, un important volum de calcule, nsa este "sigura" si permite,
n plus fata de alte metode, pozitionarea punctului de concurenta.
Problema 1. Fie ABC un triunghi si M, N (BC), P, Q (AC), R, S (AB)
puncte astfel nct BM = CN = CP = AQ = AR = BS = x, unde 0 < 2x <
< min {AB, BC, CA}. Daca A1 , B1 , C1 sunt respectiv mijloacele segmentelor (SP ),
(RN ), (M Q), aratati ca dreptele AA1 , BB1 , CC1 sunt concurente.
Constantin Cocea
x
SB
=
Solutie. Punctul S mparte segmentul orientat BA n raportul
;
c

x
SA
atunci

cx
x
x
cx
rS =
rB +
rA =
rB + rA .
c
cx
c
c
bx
x
Analog, rP =
rC + rA si atunci
b
b
1
x (b + c)
cx
bx
rA +
rB +
rC .
rA1 = (rS + rP ) =
2
2bc
2c
2b
Vectorul de pozitie al unui punct curent X al dreptei AA1 va fi

x (b + c)
(c x)
(b x)
rX = rA1 + (1 ) rA =
+ (1 ) rA +
rB +
rC ,
2bc
2c
2b
unde R. Cu totul analog, vectorul de pozitie al unui punct curent Y al dreptei
BB1 va fi

(c x)
x (a + c)
(a x)
rY =
rA +
+ (1 ) rB +
rC .
2c
2ac
2a
1

Profesori, Colegiul National si Liceul de Informatic


a "Gr. Moisil", Iasi

29

Intersectia celor dou


a drepte se obtine rezolvnd sistemul

x (b + c)
(c x)

+ (1 ) =

2bc
2c

(c x)
x (a + c)
=
+ (1 )

2c
2ac

(b x) = (a x) .
2b
2a
Sistemul este compatibil determinat, cu solutia
2bc (a x)
;
= 2
x (a + b + c) 2x (ab + bc + ac) + 3abc
2ac (b x)
= 2
.
x (a + b + c) 2x (ab + bc + ac) + 3abc
Punctul comun al dreptelor AA1 si BB1 este T , unde
1
rT = 2
[a (b x) (c x) rA +
x (a + b + c) 2x (ab + bc + ac) + 3abc
+ b (a x) (c x) rB + c (a x) (b x) rC ] .

Scriind acum ecuatia vectorial


a a dreptei CC1 si aflnd intersectia acesteia cu AA1 ,
obtinem acelasi punct T . Urmeaza ca AA1 , BB1 , CC1 sunt concurente.

Observatii.
1) Calcule foarte asemanatoare rezolva problema L.25.a) din R. M. T. 2/1990,
autor Constantin Cocea. Legat de punctul b) al acestei probleme, comparnd
notele ap
arute n R. M. T. numerele 2/1991 si 1/1996, putem observa cum uneori
calculul vectorial ajut
a la simplificarea solutiilor (a se vedea si [4]).
2) n [6] se demonstreaza concurenta naltimilor si bisectoarelor unui triunghi
folosind aceasta metoda; aceste demonstratii au constituit punctul de plecare al articolului de fata.
3) Calculele pot fi simplificate atunci cnd, din considerente geometrice, intuim
anumite simetrii verificate de punctul de concurenta.
Problema 2. Fie H ortocentrul 4ABC, M , N si P mijloacele laturilor [BC],
[CA] respectiv [AB], iar A1 (AH), B1 (BH), C1 (CH) astfel nct
AA1
BB1
CC1
=
=
. Sa se arate ca dreptele A1 M , B1 N si C1 P sunt concurente.
A1 H
B1 H
C1 H
Gabriel Popa, Paul Georgescu
Solutie. Raportam planul la un reper cu originea n centrul cercului circumscris
A1 H
a
= k, atunci
triunghiului si fie rA , rB , rC vectorii de pozitie ai vrfurilor. Dac
AA1
1
rH = rA + rB + rC , rM = (rB + rC ) ,
2
1
k
1
1
rA1 =
rH +
rA = rA +
rB +
rC .
1+k
1+k
1+k
1+k
A1 Q
Cautam un punct Q (A1 M ) astfel nct
= l, iar rQ sa se exprime simetric
QM
30

functie de rA , rB si rC :


1
l
1
l
l
1
1
rA1 +
rM =
rA +
+
rB +
+
rC ;
rQ =
1+l
1+l
1+l
2 1+k
2 1+k
l
1+k
1
2k
pentru +
=1l=
, obtinem ca rQ =
(rA + rB + rC ). Analog,
2 1+k
1+k
1 + 3k
0
00
cautam Q (B1 N ) si Q (C1 P ) care sa se exprime simetric functie de rA , rB si
asi c
a Q0 = Q00 = Q, deci cele trei drepte sunt concurente.
rC ; vom g
Problema 3. Laturile (AB), (BC), (AC) ale triunghiului ABC sunt tangente
cercului nscris de centru I n punctele C1 , A1 respectiv B1 . Daca B2 este mijlocul
laturii (AC), demonstrati ca dreptele B1 I, A1 C1 si BB2 sunt concurente.
Olimpiad
a Rep. Moldova
Solutie. Functie de vectorii de pozitie ai vrfurilor 4ABC, vectorii de pozitie ai punctelor care
A
apar n problem
a sunt:
B1
a+bc
a+cb
rA1 =
rB +
rC ;
C1
B2
2a
2a
I
b+ac
b+ca
rB1 =
rA +
rC ;
2b
2b
c+ab
c+ba
rC1 =
rA +
rB ;
B
C
A1
2c
2c
1
a
b
c
rB2 = (rA + rC ) ; rI =
rA +
rB +
rC .
2
a+b+c
a+b+c
a+b+c
Fie X un punct pe IB1 ; atunci

(b + a c) (1 ) a
+
rA +
rX = rB1 + (1 ) rI =
2b
a+b+c

(1 ) b
(b + c a)
(1 ) c
+
rB +
+
rC , R.
a+b+c
2b
a+b+c
Cautam o valoare a lui pentru care rX sa aiba o exprimare simetrica n rA si rC ,
deci
(b + a c) (1 ) a
(b + c a)
(1 ) c
b
+
=
+
=
;
2b
a+b+c
2b
a+b+c
a+c
pentru aceasta valoare a lui ,
a+cb
b
a+cb
rX =
rA +
rB +
rC .
2 (a + c)
a+c
2 (a + c)
Fie acum Y punct pe A1 C1 ; atunci
c+ab
rY = rA1 + (1 ) rC1 = (1 )
rA +
2c

(a + c b)
(a + b c) (1 ) (c + b a)
+
+
rB +
rC .
2a
2c
2a
Cautnd o valoare pentru astfel nct rY sa aiba o exprimare simetrica n rA si rC ,
a
si, pentru aceast
obtinem =
a valoare,
a+c
a+cb
b
a+cb
rA +
rB +
rC ,
rY =
2 (a + c)
a+c
2 (a + c)
31

adic
a Y = X. S
a observ
am n final c
a, datorit
a simetriei n rA si rC , acest punct se
afla si pe mediana BB2 .
Probleme propuse
1. Fie GA , GB , GC , GD centrele de greutete ale fetelor tetraedrului ABCD, iar
M un punct interior tetraedrului. Daca A0 , B 0 , C 0 , D0 sunt situate respectiv pe
semidreptele (M GA , (M GB , (M GC , (M GD , n exteriorul tetraedrului, astfel nct
M GA
M GB
M GC
M GD
=
=
=
, s
a se arate c
a dreptele AA0 , BB 0 , CC 0 , DD0
0
0
0
GA A
GB B
GC C
GD D0
sunt concurente.
Gabriel Popa, Paul Georgescu
2. Fie ABC un triunghi nscris n cercul C, A1 , B1 , C1 punctele de pe C diametral
opuse vrfurilor, iar GA , GB , GC centrele de greutate ale triunghiurilor A1 BC, B1 CA,
respectiv C1 AB. Aratati ca dreptele AGA , BGB , CGC sunt concurente ntr-un punct
situat pe dreapta lui Euler a 4ABC.
Gabriel Popa, Paul Georgescu
\
3. Fie M n interiorul 4ABC. Bisectoarele interioare ale unghiurilor BM
C,
\
\
a se
CM
A, AM
B taie laturile [BC], [CA], respectiv [AB] n A1 , B1 , respectiv C1 . S
arate c
a AA1 , BB1 si CC1 sunt concurente.
Gheorghe Neagu
4. Fie D, E, F punctele de tangenta ale cercului nscris n 4ABC cu laturile
[BC], [CA], respectiv [AB]. Paralela prin E la AB taie F D n Q, iar paralela prin
D la AB taie EF n T . S
a se arate c
a dreptele CF , DE si T Q sunt concurente.
Marcel Chirita

5. Fie tetraedrul ABCD si punctele M (AB), N (CD), P (BC), Q (AD)


AM
DN BP
AQ
astfel nct
=
,
=
. Notam {A1 } = BN DP , {B1 } = AN CQ,
MB
NC PC
QD
{C1 } = BQ DM , {D1 } = AP CM . Sa se arate ca dreptele AA1 , BB1 , CC1 si
DD1 sunt concurente.
Bibliografie
1. C. Cocea - Problema L.25, R. M. T. - 2/1990.
2. C. Cocea - Problema X.8, R. M. T. - 1/1996.
3. P. Georgescu, G. Popa - Structuri fundamentale n algebra liniara, geometria vectoriala si geometria analitica, Ed. MatrixRom, Bucuresti, 2003.
4. G. Popa - Aplicatii ale dimensiunii dreptei vectoriale, planului vectorial si a spatiului
vectorial, Matematica pentru elevi, Galati, 17-18/2001.
5. G. Popa, P. Georgescu - Dreapta lui Euler privita ca loc geometric, Recreatii Matematice - 2/2002.
6. E. Murgulescu, N. Donciu - Culegere de probleme de geometrie analitica si diferentiala (vol. I), E. D. P., 1971.
7. *** - A 46-a Olimpiada de Matematica a Rep. Moldova, R. M. T - 3/2002.

32

Teorema ariciului si cteva aplicatii


Dumitru MIHALACHE 1
n aceasta not
a ne propunem s
a prezent
am un rezultat mai putin vehiculat n
literatura matematic
a romneasc
a din ultimii ani, precum si o aplicatie oarecum
neasteptata a sa; credem ca cititorii interesati vor gasi destule alte probleme care
sa-l foloseasca. Vom prezenta teorema ariciului n mod gradat (denumirea este
justificat
a de aspectul configuratiilor ce vor ap
area); n plan mai nti pentru triunghi
si apoi pentru poligon oarecare, iar n spatiu pentru tetraedru si pe urm
a pentru un
poliedru arbitrar, cu demonstratii ntre care exista analogii.
Propozitia 1. Fie ABC un triunghi si i, j, k versori perpendiculari pe dreptele
BC, CA, respectiv AB, ndreptati spre exteriorul triunghiului. Cu notatiile uzuale,
are loc egalitatea ai + bj + ck = 0.
Demonstatia I. Notam S = ai + bj + ck; avem ca S ai = a2 + ab i j + ac i k.
b obtinem ca i j = cos C si,
Deoarece unghiul dintre i si j are masura 180 m(C),
procednd la fel, analoagele. Folosind identitatea b cos C + c cos B = a, g
asim c
a
S ai = a2 ab cos C ac cos B = a (a b cos C c cos B) = 0.

Similar, S bj = 0, prin urmare S este ortogonal pe doi vectori necoliniari, deci S = 0.


Demonstratia II. Construim, ca n figur
a,
F
reprezentanti cu originea n A ai vectorilor bj si ck,

fie acestia AE, respectiv AD; fie nca F al patrulea


D
vrf al paralelogramului construit pe acesti vectori. Se
observ
a atunci c
a 4ABC 4EF A (L.U.L.), deci
\
AF = BC = a si F[
AE \
ACB. De aici, m(M
AC) =
ck

[ m(EAF
[ ) = 90 m(ACB),
\
E
= 180 m(CAE)
A

\
adic
a m(AM C) = 90 , unde {M } = AF BC.
bj

Urmeaza ca AF este ortogonal pe BC, are lungimea

a si sens opus lui i, deci AF = ai. Pe de alt


a parte,

AF = AE + AD = bj + ck, de unde concluzia.
Sa observam ca putem considera ca teorema ariciu- B
M C
lui a fost demonstrata cu prima metoda (sau cu alta,
vom vedea c
a mai exist
a); atunci relatia ai + bj + ck = 0 conduce, avnd n vedere

figura de mai sus, la AF = ai, i.e. AF BC, AF = BC, plus o conditie privind
sensul. Prin urmare, putem afirma c
a, din punct de vedere logic, teorema ariciului
pentru triunghi este echivalent
a cu urm
atorul enunt (pb. 45, pg. 49 din [2]):
Problema 1. Se considera 4ABC pe ale carui laturi [AB] si [AC] se construiesc n exterior patratele ABGD si ACKE. Daca O este mijlocul lui DE, atunci
AO = BC/2 si AO BC.
Altfel spus, mediana din A n 4ADE este n
altime n 4ABC; atentie c
a si invers!
\ ACB
\
S
i nca un amanunt: nu trebuie ignorat cazul n care unul din unghiurile ABC,
este drept sau obtuz.
1

Profesor, Colegiul National "Gh. Rosca Codreanu", Brlad

33

Propozitia 2. Fie A1 A2 . . . An un poligon cu laturile de lungimi A1 A2 = a1 ,


A2 A3 = a2 , . . . , An A1 = an . Pentru fiecare k = 1, n, pe latura de lungime ak se
construieste un versor ik orientat spre exteriorul poligonului;
atunci
a1 i1 + a2 i2 + + an in = 0.
Demonstratie. S
a remarc
am c
a, desi poate s
a
a4 i4
a3 i3
nu fie convex, se subntelege ca poligonul nu trebuie
sa aiba autointersectii; va convingeti usor ca pentru o
A4
"fundita" format
a cu dou
a laturi opuse ale unui dreptunghi si cu diagonalele sale, proprietatea nu are loc
A5
(asta daca reusiti sa stabiliti care este interiorul si care
A3
este exteriorul ei!).
mp
artim poligonul n triunghiuri cu interioarele
A2
disjuncte, prin diagonale care nu se intersecteaz
a. a5 i5 A1
a2 i2
Aplicam apoi Propozitia 1 fiecarui triunghi, nsumam
relatiile obtinute si concluzia urmeaza daca tinem
a1 i1
seama de faptul c
a pe laturile comune pentru cte dou
a
triunghiuri (diagonale ale poligonului!) sunt construiti cte doi vectori cu suma nul
a.
Binenteles, demonstratia poate capata si o forma mai tehnica, utiliznd metoda
inductiei matematice; l
as
am acest demers n seama cititorului. S
a spunem c
a am dat
aceste demonstratii deoarece n cazul poliedrelor se va observa o temeinic
a analogie
n argumentare. Cazul plan poate fi rezolvat mult mai simplu, chiar n urm
atoarea
forma mai generala:
Propozitia 20 . Fie A1 A2 . . . An un poligon si v1 , v2 , . . . , vn vectori n planul sau,
orientati catre exteriorul poligonului, nct, pentru fiecare k = 1, n, vk are lungimea

ct [Ak Ak+1 ] si formeaza un acelasi unghi cu Ak Ak+1 . Atunci v1 +v2 + +vn = 0.


Demonstratie (aflata de autor de la prof. Marian Tetiva, Brlad). Sa ob

serv
am c
a v1 , v2 , . . . , vn se obtin din A1 A2 , A2 A3 , . . . , respectiv An A1 printr-o

rotatie de acelasi unghi . Cum A1 A2 + A2 A3 + + An A1 = 0, la fel si v1 + v2 +


+ + vn = 0.
Pentru teorema ariciului n spatiu avem nevoie de urm
atoarea
Lem
a. Fie A1 A2 A3 A4 un tetraedru; notam cu Sk aria fetei opuse vrfului Ak si
cu hk unghiul fetelor de arii Sh si Sk , format spre interiorul tetraedrului. Atunci
are loc egalitatea S1 = S2 cos 12 + S3 cos 13 + S4 cos 14 .
Demonstratie. Consider
am nti c
a A1 se proiecteaz
a pe planul (A2 A3 A4 ) n
punctul H interior 4A2 A3 A4 . Atunci SHA3 A4 = S2 cos 12 , SHA2 A4 = S3 cos 13 ,
SHA2 A3 = S4 cos 14 si SHA3 A4 + SHA2 A4 + SHA2 A3 = S1 , de unde concluzia. Rationamentul este analog n cazul n care H
/ Int A2 A3 A4 .
S
a observ
am analogia cu egalitatea b cos C + c cos B = a din cazul triunghiului.
Propozitia 3. Cu notatiile din lema, fie versorii ik , k = 1, 4, ortogonali respectiv
pe fetele de arii Sk si orientati spre exteriorul tetraedrului. Atunci
S1 i1 + S2 i2 + S3 i3 + S4 i4 = 0.
Demonstratia pe care o d
am este dup
a [3] si decurge la fel cu aceea a Propozitiei 1. Notam asadar S = S1 i1 + S2 i2 + S3 i3 + S4 i4 si, folosind Lema, obtinem
34

c
a

S S1 i1 = S12 S1 S2 cos 12 S1 S3 cos 13 S1 S4 cos 14 = 0


si nc
a trei relatii analoage. Fiind ortogonal pe trei vectori necoplanari, vectorul S
este n mod necesar 0.
Propozitia 4. Fie un poliedru convex cu ariile fetelor S1 , S2 , . . . , Sn ( n 4).
Pe planul fetei de arie Sk se construieste versorul ik perpendicular, orientat spre
exteriorul poliedrului. Are loc relatia S1 i1 + S2 i2 + + Sn in = 0.
Demonstratie. Partitionam poliedrul n tetraedre cu interioarele disjuncte, oricare dou
a tetraedre avnd n comun cel mult o fata. Pentru fiecare tetraedru construim vectorii perpendiculari pe planele fetelor, spre exterior, de lungimi egale cu
ariile fetelor respective. Aplicam pentru fiecare tetraedru Propozitia 3 si tinem seama
ca pe fiecare fata a tetraedrelor care nu este fata a poliedrului initial sunt construiti
doi vectori care se anuleaz
a reciproc.
O alta demonstratie a teoremei ariciului pentru tetraedre poate fi gasita n [3]
si utilizeaz
a produsul vectorial, iar o frumoas
a demonstratie n cazul general apare
n [4], bazat
a pe ideea c
a suma proiectiilor vectorilor pe orice dreapt
a este nul
a
(pb. M119 din Kvant). n spatiu, o demonstratie analoag
a cu cea a Propozitiei 20
nu se poate gasi.
Folosind Propozitia 3, putem obtine valabilitatea urm
atorului enunt (de altfel,
credem c
a ele sunt echivalente), care reprezint
a extinderea n spatiu a Problemei 1:
Problema 2. Cu notatiile din lema, construim punctul B2 de cealalta parte a
planului (A1 A2 A3 ) dect A2 si astfel nct A1 B2 (A1 A3 A4 ), iar A1 B2 este numeric egal cu S2 . Analog construim B3 si B4 , apoi paralelipipedul A1 B2 B30 B4 B3 B40 A01 B20

pe vectorii A1 B2 , A1 B3 , A1 B4 . Atunci A1 A01 (A2 A3 A4 ) si A1 A01 = S1 .
n ncheiere, propunem rezolvarea urm
atoarelor probleme:
1. Deduceti, cu teorema ariciului, ca fiecare latura a unui poligon este mai mica
dect suma celorlaltor laturi; generalizare n spatiu. Este reciproca adev
arat
a?
2. Demonstrati teoremele cosinusurilor pentru tetraedru:
S12 = S22 + S32 + S42 2S2 S3 cos 23 2S2 S4 cos 24 2S3 S4 cos 34 ;
S12 + S22 2S1 S2 cos 12 = S32 + S42 2S3 S4 cos 34 .

Ar
atati c
a aceste egalit
ati sunt valabile si dac
a S1 , S2 , S3 , S4 sunt lungimile laturilor
unui patrulater (redefinind hk ).
3. Rezolvati Problema 2 sintetic (sau pe orice alt
a cale) si obtineti astfel echivalenta logic
a dintre Problema 2 si Propozitia 3.
Bibliografie
1. D. Brnzei, S. Anita, C. Cocea - Planul si spatiul euclidian, Ed. Academiei,
Bucuresti, 1986.
2. J. Hadamard - Lectii de geometrie elementara. Geometrie plana, Ed. Tehnica,
Bucuresti, 1960.
3. M. Miculita - Introducere n geometria tetraedrului, Ed. Mined, Iasi, 1994.
4. Probleme din revista KVANT (traduse si selectate de H. Banea), E. D. P., Bucuresti,
1983.
35

Num
arul polinoamelor ireductibile din Zp [X]
1 , Lucian-Georges LADUNC

2
Elena ROGOJINA
A
Problema 3 propus
a la Berkeley Preliminary Exams, Fall 1985, cere determinarea
numarului polinoamelor ireductibile de grad 3 si coeficientul dominant b
1 din Z5 [X]
([2], p. 230). Mai general, Problema 150 din G. M. (seria A), nr. 1/2003, cere determinarea numarului polinoamelor ireductibile de grad 3 din Zp [X], p prim (Gabriel
Popa, [3]).n nota de fata vom urm
ari rezolvarea acestor probleme si vom ar
ata cum
poate fi aflat numarul polinoamelor ireductibile de grad n din Zp [X], p prim.
n
P
S
a observ
am mai nti c
a polinomul
ak X k este ireductibil peste Zp dac
a si
k=0
n1
P
k
numai dac
a polinomul X k +
a1
a an 6= b
0
n ak X este ireductibil (unde evident c
k=0

este inversabil peste corpul Zp ); este deci suficient sa gasim numarul polinoamelor
normate (monice) ireductibile, prin nmultirea acestui num
ar cu p1 aflnd r
aspunsul
la problem
a.
Numarul polinoamelor de forma f = X 3 + aX 2 + bX + c, a, b, c Zp , este p3 .
Ca n [2], sa vedem nti cte dintre aceste polinoame sunt reductibile. Polinoamele
reductibile din Zp sunt
fie de forma
f = (X i) (X j) (X k), i, j, k Zp , fie de
forma f = (X i) X 2 + mX + n , i, m, n Zp si X 2 + mX + n ireductibil peste
Zp . Prima dificultate care trebuie depasita n trecerea de la p = 5 la cazul general
este num
ararea polinoamelor de primul tip: observ
am c
a num
arul lor este egal cu
num
arul tipurilor de cuvinte de lungime 3 formate cu elementele multimii Zp , care
este dat de num
arul combin
arilor cu repetitie
p (p + 1) (p + 2)
3
Cp3 = Cp+2
=
.
6
a. Dac
a
Afl
am acum cte polinoame normate ireductibile de grad 2 peste Zp exist
p = 2, n Z2 [X] exista patru polinoame de grad 2, dintre care singurul ireductibil este
X2 + X + b
1. Fie p 3 prim; n Zp [X] exista p2 polinoame de forma X 2 + mX + n,
4n este p
atratul unui element
dintre care sunt reductibile cele pentru care = m2 b
p+1
a Zp . Numarul acestor "patrate perfecte" este
. ntr-adevar (v., de exemplu,
2

0}Im f ,
[2], Problema 12, Spring 1977), Q (p) = {b
0}{x Zp | x = a2 , a Zp } = {b

2
1
unde f : Zp Zp , f (a) = a este morfism de grupuri. Deoarece p 3, ecuatia a2 = b
Z / Ker f ,
are exact doua solutii, b
1 si p[
1, deci Ker f = {b
1, p[
1} si cum Im f =
p
p1
p1
p+1
atunci Card (Im f ) =
, prin urmare Card Q (p) =
+1=
. Num
arul
2
2
2
p (p + 1)
perechilor (m, n) Z2p pentru care este "patrat perfect" este
(pentru
2
p+1
fiecare valoare dat
a lui m, n ia
valori, dat fiind faptul c
ab
4 este inversabil n
2
1
2

Student
a, Universitatea "Ovidius", Constanta
Profesor, Liceul de Informatic
a "Gr. C. Moisil", Iasi

36

Zp , p fiind impar). Prin urmare, num


arul polinoamelor normate ireductibile de grad
p
(p
+
1)
p
(p

1)
2 este p2
=
, relatie adev
arat
a si pentru p = 2.
2
2
n final, numarul polinoamelor normate ireductibile de grad 3 din Zp [X] este
p (p + 1) (p + 2)
p (p 1)
p (p 1) (p + 1)
p3
p
=
.
6
2
3
Evident, aceasta metoda de numarare este sortita esecului n cazul general al
polinoamelor ireductibile de grad n din Zp [X], dat fiind faptul ca exista nu numai
cele dou
a tipuri de polinoame reductibile din cazul n = 3. Modalitatea de rezolvare a
problemei poate fi urm
arit
a detaliat n [1], pp. 188-191 si foloseste rezultate profunde
de teoria corpurilor; vom prezenta mai jos numai desfasurarea ideilor.
Pentru un polinom normat ireductibil de gradul d din Zp [X], are loc echivalenta
n

f | X p X d | n.
n
Se arata ca polinomul X p X nu are radacini multiple, deci n descompunerea sa ca
produs de polinoame normate ireductibile nu exist
a factori care s
a se repete. Conform
echivalentei anuntate, aceast
a descompunere cuprinde ca factori toate P
polinoamele
normate ireductibile din Zp [X] al c
aror grad divide pe n, de unde pn =
d (d, p);
d|n

am notat cu (k, p) numarul polinoamelor normate ireductibile de grad k din Zp [X].


Aplicatia : N {1, 0, 1}, (1) = 1, (n) = (1)r daca n este produs de
r numere prime distincte si (n) = 0 dac
a n > 1 si n nu este liber de p
atrate se
numeste functia lui Mbius. Aceast
a functie aritmetic
a are proprietatea c
a pentru
orice aplicatie f : N C, avem ca
n
X n
X

(d) F
f (n) =
F (d) =
,
d
d
d|n
d|n
P
f (d); relatia de mai sus poarta numele de formula
unde f : N C, F (n) =
d|n

de inversiune a lui Mbius. Aplicnd aceasta formula functiei f : N C, f (n) =


= n (n, p), avem c
a F (n) = pn si atunci
1 X n d
1X
(n, p) =

(d) pn/d .
p =
n
d
n
d|n

n|d

Asadar, numarul polinoamelor normate ireductibile de grad n din Zp [X] este


1P
(d) pn/d . n cazul particular n = 3, avem
n n|d

p (p 1) (p + 1)
1
1 3
1X
(d) p3/d =
(1) p3 + (3) p =
p p =
,
(3, p) =
3
3
3
3
d|3

adic
a reg
asim rezultatul problemei [3].

Bibliografie
1. T. Albu, I. D. Ion - Itinerar elementar n algebra superioara, Ed. ALL, Bucuresti,
1997.
2. C. Costara, D. Popa - Berkeley Preliminary Exams, Ed. Ex Ponto, Constanta,
2000.
3. G. Popa - Problema 150, G. M. (seria A), nr. 1/2003.
37

Functiile lui Smarandache proprieta


ti elementare
Prezenta Not
a este rezultatul unei selectii din materialul
trimis Redactiei de c
atre Minh Perez, Rehoboth, NM, SUA.
Functia Smarandache apare n literatura matematic
a cu mult timp n urm
a (date
istorice pot fi g
asite n J. Sndor - The Smarandache function introduced more
than 80 years ago! Octogon Mathematical Magazine, 9 (2001), no. 2, 920921).
F. Smarandache redescopera si cerceteaza aceasta functie si are meritul de a fi
generat un curent de preocup
ari n privinta acesteia.
Definim functia Smarandache S (n) pe multimea N prin: S (1) = 1, iar pentru
n 2, S (n) este cel mai mic numar natural pentru care S (n)! se divide cu n.

n cele ce urmeaz
a, sunt adunate o serie de propriet
ati ale functiei Smarandache
si ale unor generalizari ale ei.
1. Daca p este prim, atunci S (p) = p. Reciproca este adevarata? (Anthony
Begay)
Solutie. Dac
a p este prim, atunci r! nu este divizibil cu p pentru r < p. Pe de
alt
a parte, p! se divide cu p si, cum este cel mai mic num
ar cu aceast
a proprietate,
rezult
a c
a S (p) = p. Reciproca nu este adev
arat
a: l
asnd la o parte cazul S (1) = 1,
cu 1 neprim, avem contraexemplul S (4) = 4. Pot fi gasite alte contraexemple?
2. Daca n este liber de patrate, iar p este cel mai mare factor prim din descompunerea sa, atunci S (n) = p. (Leonardo Motta)
Solutie. Fie n = a b . . . p descompunerea n factori primi a lui n, unde
a < b < < p. Atunci p! contine n scrierea sa toti divizorii primi ai lui n, deci
S (n) p. Pentru r < p, observam ca r! nu se divide cu p, deci S (n) p. Ramne
ca S (n) = p, ceea ce doream.
n
n
n particular, S (n) = p =

(deoarece n scrierea n = p q, avem q 2)


q
2
(T. Yau)
3. Daca p este prim, atunci S (pp ) = p2 . (Alecu Stuparu)
Solutie. Deoarece S (pp ) trebuie s
a se divid
a cu p, iar p este prim, rezult
a c
a
S (pp ) trebuie s
a fie un multiplu nenul al lui p, fie acesta kp . Mai mult, fiindc
a S (pp )
se divide cu pp , trebuie sa avem kp p (se vede ca p (p 1)! se divide cu pp1 , dar
nu si cu pp ). Atunci p2 este cel mai mic numar al carui factorial se divide cu pp , de
unde concluzia.
Asem
an
ator pot fi definite a doua si a treia functie Smarandache: S2 (n) este cel
mai mic num
ar natural pentru care S2 (n)!! se divide cu n (unde m!! este produsul
numerelor nenule cel mult egale cu m, de aceeasi paritate ca si m); S3 (n) este cel
mai mic numar natural pentru care S3 (n)!!! se divide cu n (unde m!!! este produsul
numerelor nenule cel mult egale cu m, care dau acelasi rest ca si m la mp
artirea cu 3).
4. Daca n 3 este un numar par liber de patrate, iar p este cel mai mare factor
prim din descompunerea sa, atunci S2 (n) = 2p. (Gilbert Johnson)
Solutie. Fie n = 2 a b . . . p, cu 2 < a < b < < p numere prime. Dac
a
S2 (n) = 2p k, unde 1 k < 2p, atunci (2p k)!! nu se divide prin p. Evident ca
38

(2p)!! = 2 4 . . . (2a) . . . (2b) . . . (2p) se divide la n si, cum este cel mai mic
numar cu aceasta proprietate, urmeaza concluzia.

5. Fie p numar prim impar; sa se determine S2 pk+2 , unde p = 2k + 1. (Ivan


Godunov)

Solutie. Ca n rezolvarea problemei 3, se arat


a c
a S2 pk+2 = p2 .
6. Daca n este multiplu nenul al lui 3, atunci S3 (n) este tot multiplu de 3.
(K. L. Ramsharan)
Solutie. Fie m = S3 (n); daca m nu ar fi multiplu de 3, atunci m!!! = m (m 3)
(m 6) . . . nu s-ar divide nici el cu 3 si atunci m!!! nu se divide cu n. R
amne c
a
S3 (n) este multiplu de 3.
7. Sa se rezolve ecuatia diofantica S2 (x) = p, unde p este un numar prim.
(Gilbert Johnson)
Solutie. Pentru p prim fixat, vom determina numarul de numere naturale x
astfel nct S2 (x) = p. Avem c
a p!! se divide cu x, iar p este cel mai mic ntreg cu
aceast
a proprietate. Cum p este prim, x trebuie s
a fie multiplu de p.
a) Daca p = 2, atunci x = 2.
b) Daca p > 2, atunci x este produsul dintre p si o combinatie de 0, 1 sau mai multi
p3
dintre factorii 3, 5, . . . , p 2. Notnd k =
, avem Ck0 = 1 solutie cu un singur
2
factor (x = p), Ck1 solutii cu doi factori (x = p 3, p 5, . . . , p (p 2)), Ck2 solutii cu
trei factori etc. Num
arul total de solutii este cel mult egal cu Ck0 +Ck1 + +Ckk = 2k .

Recreaii matematice
Solutiile problemelor enuntate la paginile 15 si 26.
1. nl
aturnd segmentele marcate se obtine o figur
a format
a
din trei p
atrate.
2. Cu o t
aietur
a f
acut
a n veriga a treia obtinem trei buc
ati de lant formate din
o veriga, doua verigi si patru verigi. n prima zi calatorul plateste o veriga; a doua zi
da bucata formata din doua verigi si ia napoi o veriga; a treia zi da hangiului veriga
izolat
a; a patra zi d
a bucata din patru verigi si primeste ca rest celelalte trei verigi
de la hangiu; a cincea zi d
a iar
asi veriga izolat
a; a sasea zi d
a bucata din dou
a verigi
si ia veriga napoi; n sfrsit, n a saptea zi da hangiului si veriga ramasa.
Asadar, este suficient
a o singur
a n lant pentru a putea fi f
acut
a plata convenit
a
zilnic.
3. Iat
a interpretarea corect
a a calculului efectuat: dac
a exist
a o solutie n R a
ecuatiei x2 x + 1 = 0, aceasta poate fi 1. Egalitatea 3 = 0, obtinuta punnd
x = 1 n ecuatie, arata ca 1 nu este solutie si, deci, ecuatia nu are solutii n R.
39

n leg
atur
a cu o problem
a de aritmetic
a
propus
a la BAC99.
si Ioan GHIT
1
Roman
ta GHIT
A
A
n august 1999, la bacalaureat, profilul pedagogic, a fost propus
a problema:
ntr-un depozit erau 185 t carbuni, iar n altul 237 t. Din primul depozit se iau
cte 15 t carbuni pe zi, iar din al doilea cte 18 t pe zi. Dupa cte zile a ramas n
1
depozitul al doilea de 1 ori mai mult carbune dact n primul?
2
2
Solutie ([1]). Dac
a n cel de-al doilea depozit ar fi din 237 t (adic
a 158 t) si
3
2
a 12 t), dup
a num
arul de zile cerut cantit
atile
din el s-ar scoate zilnic din 18 t (adic
3
ar fi egale. Diferenta de 185 t 158 t = 27 t este anulata de diferenta de 3 t dintre
cantit
atile scoase zilnic n 9 zile.
2
Consider
am c
a aceast
a solutie necesit
a unele clarific
ari. nti,
reprezint
a in3
1
versul lui 1 . Apoi, diferenta de 3 t nu este cea dintre 18 si 15, ci dintre 15 si 12;
2
coincidenta ntre diferentele de tone scoase zilnic este una nefericita.
a n depozitul I se scade zilnic cte a, iar dintr-o
Practic, dintr-o cantitate x aflat
cantitate y aflat
a n depozitul II se scade zilnic b (x < y, a < b). Ne intereseaz
a
m
> 1 ori mai
num
arul p de zile dup
a care n depozitul II r
amne o cantitate de
n
n
n
mare dect cea ramasa n depozitul I (a > b, x > y).
m
m
n
n
Daca n depozitul II ar fi y si s-ar scoate zilnic b, dupa cele p zile ar ramne
m
m
n
n
n
cantitatea C = y p b =
(y pb), egal
a cu cea r
amas
a n primul depozit
m
m
m
m
n
(deoarece y pb =
(x pa) din ipoteza problemei). Diferenta y
este anulat
a
n
my
n
n
n
de diferenta a b n x y : a b zile.
m
m
m
Putem formula probleme asem
an
atoare, cu conditia de a alege datele n asa fel
nct s
a fim condusi la operatii cu numere naturale.
Probleme propuse.
1. ntr-o tab
ar
a scolar
a sunt 792 elevi, iar n alta 531. Din fiecare tab
ar
a pleac
a
din 5 n 5 minute grupuri de cte 36 si respectiv 23 elevi n drumetii. Dup
a cte
minute n prima tabara se vor afla de 9/7 ori mai multi elevi dect ntr-a doua?
2. n doua cosuri se gasesc 405 si respectiv 800 bomboane. n fiecare zi se vnd
cte 15, respectiv 32 bomboane. Dup
a cte zile n cosul al doilea vor fi cu 60% mai
multe bomboane dect n primul?
Bibliografie
1. Gh. Andrei si colab. - Admiterea 1999, Ed. GIL, Zal
au, 1999.
1

Profesori, Col. Nat. "I. M. Clain", Blaj

40

Concursul "Recreatii Matematice"


Editia a III-a, Iasi, 28 August 2003
Clasa a VII-a
a
b
+
= 1.
b+1 a+1
Alexandru Negrescu, Botosani (RecMat-2/2003)
2. Un triunghi are doua mediane perpendiculare, iar suma lungimilor lor constant
a. S
a se determine maximul ariei triunghiului.
Mihai Gavrilut, Roman
3. Fie XOY un unghi oarecare si P un punct n interiorul lui. Se considera
punctele A, B OX cu A (OB) si C, D OY cu C (OD) astfel nct triunghiurile P AB si P CD s
a fie echilaterale. Ar
atati c
a, dac
a dreptele OP , AD, BC sunt
concurente, atunci P se afla pe bisectoarea unghiului XOY .
Temistocle Brsan, Iasi (RecMat-1/2003)
1. Rezolvati n N N ecuatia

Clasa a VIII-a
1. Fie n N fixat. Aratati ca exista o infinitate de numere x, y, z Z astfel nct
x2n + y 2n + z 2n = x2n+1 + y 2n+1 + z 2n+1 .
Lucian Tutescu, Craiova (RecMat-1/2003)
2. Gasiti ntregii pozitivi n, x1 , x2 , . . . , xn astfel nct x1 + x2 + + xn = 2003
si produsul x1 x2 . . . xn sa fie maxim.
Agnes Constantinescu, Harghita
0 0 0 0
3. Fie ABCDA B C D un cub. Cubul este patat cu cafea pe mai putin de
jumatate din suprafata lui totala. Aratati ca exista doua puncte pe suprafata cubului
coliniare cu centrul cubului care nu sunt p
atate cu cafea.
Valerica Benta, Iasi si Mugur Rosca, Craiova

Clasa a IX-a

2
1
1
=
1. Rezolvati n R ecuatia q
+ q
, unde [x] este
[x] [x + 2]
3
3
3
2 [x]
3 [x + 1] [x]
partea ntreag
a a lui x.
Daniel Jinga, Pitesti (RecMat-1/2003)
2. Fie f : R R o functie care satisface

2
n + 3n + 3 f (n + 2) 2 n2 + n 1 f (n + 1) + n2 n + 1 f (n) = 0,

pentru orice n natural. S


tiind ca f (0) = 0 si f (1) = 1, calculati f (2003).
Andrei Nedelcu, Iasi
3. Fie p
atratul ABCD, E mijlocul lui (AB), M (CD), N (AD) astfel nct
BM k EN . S
a se arate c
a M N este tangenta cercului C (S, r) nscris n p
atrat.
Nicu Miron, Iasi

Clasa a X-a
1. Fie a, b (0, 1) (1, ) si functia injectiv
a f : (0, ) R astfel nct functia
g : R R, g (x) = f (ax ) + f (bx ) este constanta. Sa se arate ca ab = 1 si ca exista
41

functii f care satisfac ipotezele problemei.


Dan Popescu, Suceava (RecMat-1/2003)
sin + i
2. Sa se afle locul geometric al imaginilor numarului complex z =
,
sin i
(0, ).
Mihai Gavrilut, Roman
3. Un triunghi de arie S se proiecteaz
a pe trei plane perpendiculare dou
a cte
,
S
,
respectiv
S
,
s
a
se
demonstreze
c
a
dou
a. Dac
a ariile proiec
t
iilor
sunt
S
1
2
3

S S1 + S2 + S3 < S 3.
Gheorghe Iurea, Iasi

Clasa a XI-a
1. Fie D, M dou
a matrice nesingulare de ordin n, D diagonal
a, iar M triunghiular
a. Dac
a D = t M DM , s
a se arate c
a M este tot o matrice diagonal
a, avnd 1
pe diagonala principala.
Adrian Corduneanu, Iasi (RecMat-1/2003)
2. Fie (xn )n1 , (yn )n1 dou
a siruri de numere naturale mai mari ca 1. S
a se
xn yn
pxn pyn
arate c
a lim
= 0 lim
= 0, unde pn este al n-lea num
ar prim.
n
n
yn
pyn
Gabriel Mrsanu, Iasi
3. n tetraedrul ABCD se consider
a notatia (ab) = m [ (ABC; ABD)], corespunz
atoare muchiei AB si analoagele, corespunz
atoare la celelalte muchii. Ar
atati c
a

1
cos (cd) cos (bd) cos (bc)

cos (cd)
1
cos (ad) cos (ac)

= 0.
cos (bd) cos (ad)
1
cos (ab)

cos (bc) cos (ac) cos (ab)


1

Silviu Boga, Suceava


4. O pat
a de ulei curge pe un ru. La un moment dat ea intersecteaz
a umbra
unui fir de telegraf. Sa se demonstreze ca exista un moment n care umbra firului
mparte pata de ulei n doua portiuni de aceeasi arie.
Vlad Martinusi, Iasi

Clasa a IX-a (BARAJ)


1. Determinati functiile f : R R care verific
a egalitatea
3

2
xf x + x + 1 + f x + 3x 4x + 3 = x + 1,

x R.

Silviu Boga, Suceava


2

2.
Se
dau
mul
t
imile:
A
=
x
+
x
|
x

Z
,
B
=
x
+
x
|
x

Z
, C =
4

3
2
= x + x + x + x | x Z , D = 2x | x Z . Determinati multimile A C,
B D.
Andrei Nedelcu, Iasi (RecMat-2/2002)

42

Concursul interjudetean "Octav Onicescu"1


Editia a VII-a, 31 oct. - 2 nov. 2003, Botosani
Aceast
a editie a Concursului de matematic
a "Octav Onicescu" a cunoscut o participare numeroas
a si entuziast
a, antrennd elevi din 5 judete: Botosani, Iasi, Suceava,
Vaslui si Vrancea.
Ceea ce particularizeaz
a n mod deosebit acest concurs este faptul c
a se propun spre
rezolvare aceleasi subiecte pentru toti participantii de la clasa a IX-a pn
a la clasa a XII-a.
Subiectele propuse nu sunt axate pe materia studiat
a de fiecare elev la nivelul s
au de studiu,
ci ncearc
a s
a pun
a n valoare abilit
atile matematice pure ale concurentilor.
Deschiderea festiv
a a concursului si premierea s-au desf
asurat n Aula Magna a C. N.
"A. T. Laurian" din Botosani, iar al
aturi de elevi si profesori au participat si autorit
atile
locale. De partea organizatoric
a s-a ocupat I. S. J. Botosani si C. N. "A. T. Laurian".
Sarcina elabor
arii subiectului de concurs a revenit, ca n fiecare an, domnilor profesori Adrian Botan si Adrian Panaete, iar misiunea corect
arii lucr
arilor scrise, membrilor catedrei de matematic
a de la C. N. "A. T. Laurian". Presedintele comisiei a fost
prof. univ. dr. Eugen Popa de la Facultatea de Matematic
a, Universitatea "Al. I. Cuza"
din Iasi.

Public
am n continuare problemele propuse concurentilor si lista premiatilor:
1. Fie a1 , a2 , a3 , . . . , a2003 numerele 1, 2, 3, . . . , 2003 n alta ordine. Aratati
ca macar doua din numerele |a1 1|, |a2 2|, . . . , |a2003 2003| sunt egale. (20p)
2. De pe o tabla de sah 7 7 scot un patrat; aratati ca patratele ramase:
a) nu pot fi acoperite cu 24 de dominouri 1 2 daca patratul scos e A2;
b) pot fi acoperite cu 24 de dominouri dac
a p
atratul scos e D4 si indicati o
acoperire cu num
ar minim de dominouri orizontale (justificare).
(20p)
n
3. Dac
a n este natural, g
asiti restul mp
artirii lui 10 prin 999 si ar
atati c
a un
num
ar natural divizibil cu 999 are m
acar 3 cifre nenule. Cte numere cu cel mult 16
cifre fiecare au fix 3 cifre nenule si se divid cu 999?
(30p)
4. Cte p
atrate ale unei table de sah 340 121 sunt t
aiate n interior de una din
diagonalele tablei? Dar pentru o tabla 340 120?
(30p)
5. Ali Baba si cei 40 de hoti stau n cerc n jurul focului si vor s
a mpart
a n
mod egal 4100 de galbeni care initial se afla mpartiti la ntmplare la ctiva dintre
ei (posibil la unul singur). Ali Baba bate din palme si la comanda lui fiecare din cei
41 d
a un galben vecinului din stnga sa, dac
a acesta are mai putin dect el (dac
a
vecinul are egal sau mai mult nu primeste nimic!). Dac
a nu au realizat egalitatea,
Ali Baba bate din palme din nou etc. Justificati ca dupa un timp sumele se egaleaza
(toti 100 de galbeni).
(30p)
Premiatii sunt: premiul I - Chiril
a Cezar (C. N. "M. Eminescu", Botosani),
premiul II - Istrate Carmen Maria (C. N. "Unirea", Focsani), premiul III Pachitariu Marius (Colegiul National Iasi). Au fost acordate 21 mentiuni.
1 Selectiuni din materialul trimis redact iei de c
atre elevul Alexandru Negrescu si prof. Liliana
Tomita
, C. N. "A. T. Laurian", Botosani

43

Concurs de admitere 2003, Iasi


Facultatea de Informatic
a, Universitatea "Al. I. Cuza"
Algebr
a
I. 1. Se da matricea A M3 (R),
unde M
3 (R) este inelul matricelor patratice
0 0 1
de ordin 3 cu elemente reale, A = 1 0 0. S
a se arate c
a A3 = I3 si c
a are loc
0 1 0
2

relatia (A I3 ) A + A + I3 = 0.
2. Fie S3 o permutare din grupul simetric de grad 3, astfel nct 2 = e
(e noteaz
a permutarea identic
a). Demonstrati c
a exist
a k {1, 2, 3} astfel nct
(k) = k.
1
3. Demonstrati ca polinomul P = X 3 + X + 1 este ireductibil n Q [X].
2
II. 1. Fie G un grup cu n elemente, n N . Aratati ca n orice coloana a tablei
operatiei lui G apar n elemente distincte.
2. Fie (Z, +, ) inelul numerelor ntregi. Determinati toate morfismele de inele
f : Z Z.
3. Fie (C, +, ) corpul numerelor complexe. Sa se arate ca f : C C definita prin
f (z) = z este izomorfism de corpuri (z noteaza conjugatul numarului complex z).
Analiz
a matematic
a
1 1
1
I. 1. Fie Hn = 1 + + + + , n N . Demonstrati ca sirul (Hn )nN este
2 3
n
nemarginit.
2. Fie f : R R o functie continua si marginita. Demonstrati ca exista x0 R
astfel nct f (x0 ) = x0 .
1
3. Fie f : R \ {1} R, f (x) =
. Sa se calculeze f (n) (0), unde n N ,
x+1
a derivata de ordin n a functiei f .
iar f (n) noteaz
II. Pentru n N consider
am fn : R R, fn (x) = xn + xn1 + + x 1.
f3 (x) f2 (x)
a) Reprezentati grafic functia g : D R, g (x) =
, unde D este
f1 (x)
domeniul maxim de definitie al functiei g.
b) Aratati ca pentru orice n N , ecuatia fn (x) = 0 are o unica solutie reala,
un , n intervalul [0, 1].
c) Demonstrati c
a sirul (un )nN este convergent.
d) S
a se determine lim un .
n

Fac. de Electronic
a si Telecomunicatii, Univ. Tehnic
a "Gh. Asachi"

13
3
a
1. Rangul termenului din dezvoltarea
care l contine pe a4 este
+
3
3
a
a) 8
b) 6
c) 3
d) 4
e) 9
n
k
P
k
k
2. Suma
2 + 3 /6 este egala cu
k=1

1
1
1
1
a) 1 n n b) 2 n+1 n+1
2
3
2
3
3
1
1
e) n
2 2
2 3n

c)

44

3
1
1
+
+
2 2n 2 3n

d)

1
1
+
2n 3n

3. Se consider
a inecuatia (m + 1) e2x + 2 (m + 1) ex + m > 0, x R, unde
m R este un parametru. Valorile lui m pentru care inecuatia este verificata x R
sunt
a) (, 0] b) [0, +) c) [1, 0] d) (0, 1) e) (, 1)
4. Multimea tuturor valorilor m R astfel ca sistemul

mx + y z = 0
x + (m + 1) y + z = 2 + m m2

x 2y mz = 2 + 3m m2
sa fie compatibil este
a) {2} b) {2, 1, 2} c) R \ {2, 1} d) R \ {1} e) R \ {2, 1, 2}
10
5. Numarul complex z, care satisface |z| + z =
este
2i
3
3
1
3
a) 2 + i b) 2 + 5i c) 2i d) + 3i e) + 2i
2
2
2
2
x sin x
a fie finit si diferit de zero.
6. S
a se determine m astfel ca l = lim
m s
x0 (1 cos x)
Sa se precizeze si valoarea lui l.

3
2
2
2
2
3
3
c) m = , l =
b) m = 3, l =
d) m =
,l =
a) m = , l =
4
3
2
3
2
3
3
1
e) m = 2, l =
6
2 ln x 1
este
7. Multimea valorilor functiei f : (0, ) R, f (x) =
2 x2

2
b) (0, ) c) R d) (0, e) e) 1/e ,
a) , 1/e
8. Daca x1 , x2 , . . . , xn sunt radacinile ecuatiei xn + 1 = 0, atunci valoarea sumei
1
1
1
+
+ +
este
1 x1 1 x2
1 xn
a) n/2 b) n2 c) n d) n (n + 1) e) n

Z 2
x3
max x , arctg x dx.
9. Sa se calculeze I =
3
1
1
5
5
2 arctg 3
a) arctg 2 + ln 3 b) + 2 arctg 2 ln 5 c)
12
2
12
5
5
2 arctg 2 + ln 3 e) + arctg 2
d)
11
12
1
1

10. S
a se afle solutia ecuatiei arcsin
+ arcsin
= .
x

1
x
+
1
p
p
p
p

2 p

a) 3 + 5 b) 3 + 5 c) 2 + 5 d) 2 + 5 e) 1 + 5

Fac. de Automatic
a si Calculatoare, Univ. Tehnic
a "Gh. Asachi"
1. S
a se determine parametrul real m astfel nct ecuatia x2 + 2mx + (m + 4) = 0
s
a admit
ad
acinile reale
x si x2 verificnd
< x2 .
a r
! x1 < 1
! 1

1 17
1 + 17
5
3
a) m ,

,
b) m ,
c) m ,
2
2
3
5

1 17 3
d) m e) m
,
2
5
45

2. Aflati num
arul termenilor rationali din dezvoltarea binomial
a
a) 1 b) 2 c) 3 d) 4 e) 5
3. Fie sistemul
x+y+z =1
ax + ay + 2az = b
a, b R, b 6= 0.
2
a x + a2 y + 2a2 z = b2

23

3
7+ 53 .

Care din urm


atoarele afirmatii este fals
a?
a) Dac
a a = 0, sistemul este incompatibil b) Dac
a a = b, sistemul este compatibil nedeterminat c) Exist
a a, b R, b 6= 0 astfel nct sistemul are solutie unic
a
d) Daca a 6= 0 si a 6= b, sistemul este incompatibil e) Daca a = 1 si b 6= 1, atunci
sistemul este incompatibil
4. Fie M = (, 1) (1, ) si legea de compozitie intern
a pe M dat
a prin
x y = 3ax + by + xy, x, y M , unde a, b R, b 6= 0. S
a se afle a si b astfel nct
(M, ) sa fie grup abelian si sa se precizeze simetricul x0 al unui element oarecare
x M.
1
x
x
x
1
a) a = , b = 1, x0 =
b) a = 1, b = 3, x0 =
c) a = , b = 1, x0 =
3
x+1
x+1
3
x+1
1
x
1
1
d) a = 1, b = , x0 =
e) a = , b = 1, x0 =
3
x+1
3
x+1 n
5. Se d
a sirul definit prin relatia xn+1 = xn + (a) , n N , x1 = 0, unde
0 < a < 1. Care din urm
atoarele afirmatii este adev
arat
a:
a) sirul este strict cresc
ator cu limita + b) sirul este strict descresc
ator cu
a
limita c) sirul nu este monoton, dar are limita
d) sirul este strict crescaa+1
tor cu limita 1 e) sirul nu este monoton, deci nu are limita
1
3
5
1
+
+
+ . Num
arul
6. Se d
a f : R\ {0, 2, 4, 6} R, f (x) = +
x x2 x4 x6
punctelor n care graficul functiei intersecteaza axa Ox este
a) 0 b) 1 c) 2 d) 3 e) 4
1
a se precizeze intervalul pentru
7. Fie ecuatia diferential
a y 0 + y = 6x, x > 0. S
x
care y (x) > 0, unde y (x) este solutia care satisface conditia y (1) = 1.
1

a) x (1, 2) b) x 3 2, c) x
,

d)
x

(2,
3)
e)
x

0,
3
3
2
2
8. Se dau triunghiurile ABC si A0 B 0 C 0 ce au centrele de greutate G si G0 . Atunci

vectorul GG0 este egal cu


1
1
2
a) (AA0 + BB 0 + CC 0 ) b) (AA0 + BB 0 + CC 0 ) c) (AB + BC + CA + A0 B 0 +
3
4
3

1
1
B 0 C 0 + C 0 A0 ) d) (AB 0 + BA0 + AC 0 + CA0 + BC 0 + CB 0 ) e) (AB 0 + BC 0 + CA0 )
6
3
9. S
a se determine multimea punctelor din planul complex care sunt imaginile
numerelor z care verific
a ecuatia z 2 z |z| + |z|2 = 0.
a) doua drepte perpendiculare b) un cerc cu centrul n origine c) doua drepte
paralele d) dou
a semidrepte e) dou
a cercuri concentrice
1
1
1

10. Num
arul solutiilor ecuatiei arctg
+ arctg
arctg 2
= este
x1
x+1
x 1
4
a) 1 b) 2 c) 3 d) 4 e) o infinitate
46

Solutiile problemelor propuse n nr. 1 / 2003


Clasele primare
P.44. Un vecin al unui vecin al numarului 81 este egal cu un vecin al unui vecin
al numarului 77. Despre ce numar este vorba?
( Clasa I )
Mihaela Rusu, elev
a, Iasi
Solutie. Acest num
ar trebuie s
a fie mai mare ca 77 si mai mic dect 81. Num
arul
se afl
a n secventa 77 81. Este vorba despre num
arul 79.
P.45. Adunnd trei numere naturale a, b, c obtinem suma 62. Primul numar este
mai mare dect al treilea si mpreuna au suma 12. Care sunt cele trei numere?
( Clasa a II-a)
nv. Maria Racu, Iasi
Solutie. Numarul b = 62 12 = 50. Perechea (a, c) poate fi: (12, 0); (11, 1);
(10, 2); (9, 3); (8, 4) sau (7, 5). Tripletul (a, b, c) poate lua valorile: (12, 50, 0); (11, 50, 1);
(10, 50, 2); (9, 50, 3); (8, 50, 4) sau (7, 50, 5).
P.46. Mihai, Dan si Petru practica fiecare un alt fel de sport si anume: tenis,
fotbal sau volei. Mihai si voleibalistul locuiesc n acelasi bloc. Cel care joaca volei si
cel care joaca fotbal l-au urmarit pe Petru la un meci. Ce sport practica fiecare?
( Clasa a II-a)
Adina Dohotaru, elev
a, Iasi
Solutie. Din textul problemei se deduce c
a Petru nu joac
a volei sau fotbal, deci
el joac
a tenis. Mihai si voleibalistul locuiesc n acelasi bloc. Aceasta nseamn
a c
a
Mihai nu joaca volei. Solutia problemei este: Petru joaca tenis, Mihai joaca fotbal si
Dan joaca volei.
P.47. Diferenta a doua numere este 48. Aceasta diferenta este cu 22 mai mare
dect jumatatea unuia dintre ele. Determinati numerele.
( Clasa a III-a)
nv. Rodica Rotaru, Brlad
Solutie. Fie a b = 48. Avem dou
a cazuri: 1) 48 = b : 2 + 22 de unde obtinem
b = 52 si a = 100. 2) 48 = a : 2 + 22 de unde obtinem a = 52 si b = 4.
P.48. Un agricultor mparte un teren n trei parcele. n fiecare an, fiecare parcela
este cultivata numai cu una din culturile: gru, porumb sau legume. ncepnd cu
anul 2003, agricultorul se hotaraste ca pe fiecare parcela sa fie alta cultura n trei
ani consecutivi.
a) Care este primul an dupa 2003 n care se repeta culturile pe cele trei parcele?
b) Se poate preciza care este ordinea culturilor pe cele trei parcele n anul 2019?
( Clasa a III-a)
Andreea Surugiu, elev
a, Iasi
Solutie. Presupunem c
a n anul 2003 avem ordinea (gru, legume, porumb).
n anul 2004 putem avea (legume, porumb, gru) sau (porumb, gru, legume). n
anul 2005 putem avea (porumb, gru, legume) sau (legume, porumb, gru). n 2006
avem din nou ordinea (gru, legume, porumb). Raspunsul la a) este anul 2006. b)
Ordinea culturilor se mai repet
a n 2009, 2012, 2015, 2018. Nu putem preciza ordinea
culturilor n anul 2019.
P.49. La un moment dat, cernd unei persoane anul nasterii, aceasta raspunde:
"anul acesta mplinesc 25 ani, iar daca as scrie toate numerele ncepnd cu 1 si
terminnd cu anul nasterii si apoi toate numerele ncepnd cu 1 si terminnd cu
47

anul n care ne aflam mi-ar trebui 13710 cifre. n ce an ne aflam cnd am pus
ntrebarea?
( Clasa a III-a)
Prof. C
at
alin - Cristian Budeanu, Iasi
Solutie. Pentru scrierea numerelor de la 1 999 sunt necesare 2889 cifre. Rezult
a
c
a anul nasterii nu poate fi format din trei cifre. ntr-adev
ar, 2 2889 + 25 n < 13710,
n 4. Anul nasterii este de forma abcd. Fie x numarul cifrelor pentru scrierea
numerelor de la 1 la abcd. Transpunnd n ecuatie ceea ce a spus persoana, obtinem:
x + (x + 4 25) = 13710, cu solutia x = 6805. Pentru scrierea numerelor de la 1000
la abcd sunt necesare 6805 2889 = 3916 cifre, ceea ce nseamn
a c
a de la 100 la abcd
sunt 3916 : 4 = 979 numere. nseamna ca anul abcd este 1978. ntrebarea a fost pusa
n anul 1978 + 25 = 2003.
P.50. a) Cte numere trebuie adaugate sirului 1, 2, 4, 5, 7, 8, . . . , 97, 98 pentru a
obtine toate numerele de la 1 la 98?
b) Efectuati 1 + 2 + 4 + 5 + 7 + 8 + + 97 + 98 2 (3 + 4 + 5 + + 34).
( Clasa a IV-a)
Georgiana Ciobanu, elev
a, Iasi
Solutie. a) Lipsesc numerele: 3, 6, 9, . . . 96 care pot fi scrise: 3 1, 3 2, 3 3, . . . ,
3 32. Se observa ca lipsesc 32 numere.
b) Expresia de calculat se poate scrie:
1 + 2 + (4 3) + (5 3) + (7 4) + (8 4) + + (97 34) + (98 34) =
= 1 + 2 + (1 + 2 + 3 + 4 + + 63 + 64) = 3 + 64 65 : 2 = 3 + 2080 = 2083.

P.51. Produsul a doua numere naturale este 913 368. Unul din numere are cifra
unitatilor si cifra zecilor mai mare ca 2 si mai mica dect 8. Daca la acest numar
marim cifra zecilor cu 2 si micsoram cifra unitatilor cu 1, obtinem un produs egal
cu 951 425. Aflati cele doua numere.
( Clasa a IV-a)
nv. Elena Z
arnescu, Iasi
Solutie. Fie a si b numerele c
autate. Obtinem
(a + 20 1) b = 951425 ab + 19b = 951425
913368 + 19b = 951425 b = 2003 a = 913368 : 2003 = 456.
P.52. n trei cutii sunt 212 bile. Din prima cutie se scoate un numar de bile, din
a doua de 2 ori mai mult si nca doua bile, din a treia se scoate ct triplul numarului
de bile scos din a doua cutie. n fiecare cutie ramne un numar de bile egal cu
numarul total al bilelor scos din cele trei cutii la un loc. Cte bile au fost n fiecare
cutie?
( Clasa a IV-a)
nv. Maria Racu, Iasi
Solutie. Not
am cu p num
arul bilelor scos din prima cutie. Rezult
a c
a n fiecare
cutie ramn 9p + 8 bile. Deducem ca n toate cutiile au fost 36p + 32 bile. Asadar,
36p + 32 = 212, de unde p = 5. n cele trei cutii au fost 58, 65, respectiv 89 bile.
P.53. Efectund o singura cntarire, sa se ia 475 g dintr-un kilogram de zahar
utiliznd doua greutati, una de 200 g si cealalta de 150 g.
( Clasa a IV-a)
Prof. Petru Asaftei, Iasi
Solutie. Utiliz
am o balanta cu brate egale. Distribuim kilogramul de zah
ar
si cte una din cele dou
a greut
ati, pe cele dou
a talere, pn
a realiz
am pozitia de
echilibru. Pe fiecare taler vom avea 675 g. Masa cautata este pe talerul n care se
48

afl
a greutatea de 200 g: 675 g 200 g = 475 g zah
ar.

Clasa a V-a
V.36. Fie n un numar impar, iar a1 , a2 , . . . , an , n N numere care mpartite
la n dau cturi distincte si resturi distincte. Aratati ca valoarea minima a sumei
S = a1 + a2 + + an este multiplu de 12.
Dragos Ungureanu, elev, Iasi
Solutie. Conform ipotezei, avem: a1 = nc1 +r1 , a2 = nc2 +r2 , . . . , an = ncn +rn ,
unde {r1 , r2 , . . . , rn } = {0, 1, 2, . . . , n 1}. Astfel, suma
n (n 1)
S = a1 + a2 + + an = n (c1 + c2 + + cn ) +
2
este minim
a dac
a {c1 , c2 , . . . , cn } = {0, 1, 2, . . . , n 1}, deci
n (n 1) n (n 1)
n (n 1) (n + 1)
Smin = n
+
=
.
2
2
2
.
.
Cum n este impar, rezulta ca (n 1) (n + 1) .. 8, deci Smin .. 4. Pe de alta parte,
..
deoarece n, n 1, n + 1 sunt numere consecutive, rezulta ca S
. 3. Prin urmare,
min

Smin este multiplu de 12.

V.37. Comparati fractiile a =

222221
333331
si b =
.
333334
222223

Maria Cojocaru, Iasi


3
1
2
1
si = 1 +
. Cum 3 222221 > 2 333331,
Solutie. Avem = 1 +
a
333331
b
222221
1
1
rezult
a c
a > , deci b > a.
a
b
V.38. Sa se arate ca 2a + 2b + 2c + 2d + 2e 6= 2003, a, b, c, d, e N.
Irina Ispas, student
a, Iasi
Solutie. Presupunem c
a exist
a cinci numere naturale a b c d e astfel
nct
2a + 2b + 2c + 2d + 2e = 2003.
(1)
Daca a 6= 0, atunci termenul din stnga al egalitatii (1) este par si atunci avem o
contradictie. Pentru a = 0, relatia (1) devine: 2b + 2c + 2d + 2e = 2002. Deoarece
210 = 1024, rezult
a c
a numai e ar putea avea, eventual, valoarea 10.
Daca e = 10, atunci 2b + 2c + 2d = 978. n acest caz, daca b, c, d 8, atunci
2b + 2c + 2d 3 256 < 978. Asadar, d = 9 si 2b + 2c = 466, ceea ce nu este posibil.
Dac
a toate numerele b, c, d, e sunt strict mai mici ca 10, se observ
a c
a cel mult
trei numere pot fi 9 (altfel avem 2b + 2c + 2d + 2e 4 512 > 2002) si cel putin trei
trebuie sa fie 9 (deoarece, n caz contrar, avem 2b + 2c + 2d + 2e < 29 + 29 + 28 + 28 <
< 2002). Prin urmare, c = d = e = 9 si atunci 2b = 2002 3 29 = 476, absurd.
V.39. Sa se determine numerele prime p1 < p2 < p3 < p4 astfel nct numerele
p1 + p2 + p3 + p4 , p3 p2 , p4 p3 sa fie, de asemenea, prime.
Petru Minut, Iasi
Solutie. Deoarece p1 + p2 + p3 + p4 este un numar prim mai mare ca 2, rezulta
c
a el este impar si atunci unul dintre numerele p1 , p2 , p3 , p4 trebuie s
a fie par, deci
p1 = 2. Cum p2 , p3 si p4 sunt impare, nseamn
a c
a p3 p2 si p4 p3 sunt pare si
avnd n vedere ca sunt prime, rezulta ca p3 p2 = p4 p3 = 2. De aici, deducem
49

c
a p3 = p2 + 2 si p4 = p2 + 4. Se observ
a c
a p1 = 2, p2 = 3, p3 = 5 si p4 = 7 este
o solutie a problemei (2 + 3 + 5 + 7 = 17 este numar prim). Daca p2 > 3, atunci
p2 = 3k + 1 sau p2 = 3k + 2, k N . n cazul p2 = 3k + 1, avem p3 = 3k + 3 care
nu este prim, iar n cazul p2 = 3k + 2, avem p4 = 3k + 6, care nu este prim. Asadar,
p1 = 2, p2 = 3, p3 = 5, p4 = 7 este singura solutie.
V.40. Este posibila o partitionare a multimii {1, 2, . . . , 12n + 9} n 4n + 3 submultimi disjuncte, fiecare cu cte trei elemente, astfel nct n fiecare submultime un
element sa fie suma celorlaltor doua?
Titu Zvonaru, Bucuresti
Solutia I. Fie {a, b, c} o multime astfel nct a = b + c. De aici, rezult
a c
a
elementele multimii {a, b, c} sunt ori toate pare, ori dou
a impare si unul par. Asadar,
pentru ca sa fie posibila o partitie ca n problema, trebuie ca multimea data sa contina
un num
ar par de numere impare. Deoarece multimea dat
a are 6n + 5 numere impare,
rezult
a c
a partitionarea nu este posibil
a.
Solutia II. S
a presupunem c
a ar fi posibil
a o partitie n conditiile impuse. Atunci,
fiecare din cele 4n + 3 submultimi de trei elemente are suma elementelor egala cu un
num
ar par, deci suma elementelor multimii {1, 2, . . . , 12n + 9} ar trebui s
a fie num
ar
(12n + 10) (12n + 9)
par. Cum, 1 + 2 + + 12n + 9 =
= (6n + 5) (12n + 5), care
2
este un numar impar, rezulta ca partitionarea ceruta nu este posibila.

Clasa a VI-a
VI.36. Fie k N, k 3. Aratati ca printre valorile naturale ale lui n care fac
.
adevarata propozitia n2 + k .. n + k, exista cel putin trei patrate perfecte.
Claudiu S
tefan Popa, Iasi
Solutie. Din n2 + k = n2 k 2 + k2 + k = (n k) (n + k) + k 2 + k, rezult
a c
a

.
.
n2 + k .. n + k dac
a si numai dac
a k 2 + k .. n + k. Cum A = k, k + 1, k2 + k Dk2 +k ,
putem lua n + k din multimea A si atunci obtinem n 0, 1, k 2 . Astfel, am g
asit
trei patrate perfecte care verifica cerinta problemei.
VI.37. Numerele 1160, 1604 si 2270 dau acelasi rest la mpartirea prin n. Aflati
mpartitorul n.
Cristian Laz
ar, Iasi
Solutie. Conform ipotezei, avem: 1160 = nc1 +r, 1604 = nc2 +r, 2270 = nc3 +r,
unde r < n si r, c1 , c2 , c3 N. Sc
aznd aceste egalit
ati dou
a cte dou
a, obtinem

444 = n (c2 c1 ), 666 = n (c3 c2 ) si 1110 = n (c3 c1 ), deci n este divizor comun
al numerelor 444, 666, 1110. Cum (444, 666, 1110) = 222 rezulta ca n {1, 2, 3, 6,
37, 74, 111, 222}, valori care verific
a ipoteza problemei.
VI.38. Demonstrati ca nu exista numere naturale x, y, z direct proportionale cu
trei numere naturale consecutive, astfel nct x + y + z sa fie numar prim.
Alexandru Negrescu, elev, Botosani
Solutie. Dac
a presupunem contrariul, avem
x
y
z
x+y+z
(1)
=
=
=
, cu n N .
n
n+1
n+2
3n + 3
De aici, obtinem c
a 3y = x + y + z, deci 3 | x + y + z, care mpreun
a cu faptul c
a
x + y + z este prim ne conduce la concluzia ca x + y + z = 3 si deci y = 1. nlocuind
50

x
1
n
=
, adic
ax=
, care nu apartine lui N.
n
n+1
n+1
VI.39. Radu si Mihai joaca de mai multe ori un joc n urma caruia cstigatorul
primeste a puncte, iar cel care pierde primeste b puncte ( a, b N , a > b). Daca
scorul final este 61 49 n favoarea lui Radu, iar Mihai a cstigat 4 partide, aflati
a si b.
Adrian Zanoschi, Iasi
Solutie. Dac
a not
am cu x num
arul partidelor cstigate de Radu, avem: xa+4b =
= 61, 4a + xb = 49, de unde obtinem ca (x + 4) (a + b) = 110. De aici, avnd n
vedere ca x + 4 9 si a + b 3, rezulta ca x + 4 = 22 si a + b = 5 sau x + 4 = 11
si a + b = 10 sau x + 4 = 10 si a + b = 11. n primul caz, avem x = 18, dar atunci
xa + 4b este un num
ar par, diferit de 61, deci aceast
a situatie nu convine. Procednd
la fel, constatam ca nici al treilea caz nu convine. n al doilea caz, gasim x = 7, a = 7
si b = 3, care este solutia problemei.
b = 120 . Perpendiculara n C pe AC intersecteaza
VI.40. Fie 4ABC cu m(A)
mediatoarea lui [AB] n D; notam {E} = CD AB. Sa se arate ca AB = 2AC
\ = 90 si BE = 2AB.
daca si numai daca m(BDE)
Ioan S
ac
aleanu, Hrl
au
Solutie. Fie M mijlocul lui AB.
Presupunem ca AB = 2AC. n acest caz
D
\ = ADM
\ =
rezult
a c
a AM = AC, deci CDA
\
= M
DB = . Cum suma unghiurilor patrulaterului DM AC este 360 , obtinem ca
C
\ = 90 . Triunghiul DAB
= 30 , deci BDE
\ de 60 , adica
este isoscel si are unghiul BDA
este echilateral si, prin urmare, DA = AB. n B
M
A
E
\ = 60 , deci AEC
[ = 30 . Atunci
plus DBA
4ACD 4ACE (C.U.), de unde AD = AE. n concluzie, BA = AD = AE, adica
BE = 2AB.
\ = 90 si A mijlocul lui BE. Cum AC k BD, rezult
Fie acum BDC
a c
a [AC] este
1
[ = 60 si CA k BD,
linie mijlocie n triunghiul BDE, deci AC = BD. Din CAE
2
\ = 60 , deci triunghiul DBA este echilateral, ceea ce conduce la
obtinem ca DBA
1
concluzia BD = AB. Asadar, avem AC = AB sau AB = 2AC.
2
n relatia (1), g
asim

Clasa a VII-a

2
+ +
n

2n 1
< 2n 1, n N, n 2.
n
C
at
alin Calistru, Iasi
Solutia I (un grup de elevi de r
la Colegiul National din Iasi si Alexandru
1 + k/n
k+n
k
<
=
, k = 1, 2n 1. Ca
Negrescu, elev, Botosani). Avem
n
2
2n
urmare,
r
r
r

1
1
1
2
2n 1
+
+ +
<
2n 1 + (1 + + (2n 1)) =
n
n
n
2
n
VII.36. Sa se arate ca

1
+
n

51

1
1 (2n 1) 2n
2n 1 +
= 2n 1.
2
n
2
Solutia II. r
Membrul r
din stnga al inegalit
atii date se poate scrie grupnd ternk
n+k
menii de forma
,
, k {1, 2, . . . , n 1}. n acest fel, obtinem n 1
nr
n
n
paranteze si termenul
= 1. Deoarece
n
nk+n+k
r
!2
r
r
r
nk
n+k
nk n+k
n
=2+2
+
<2+2
= 4,
n
n
n
n
2
r
r
nk
n+k
+
< 2, de unde concluzia.
rezult
a c
a
n
n
VII.37. Aratati ca n baza de numeratie 7 printre numerele ce se scriu cu cifrele
0, 1, 2 exista o infinitate care sunt patrate perfecte si o infinitate ce nu sunt patrate
perfecte. Aceste afirmatii ramn valabile daca se folosesc cifrele 3, 5, 6?
Ruxandra Ioana Vlcu, elev
a, Iasi
2
n
2n
n
2
=
(7
+
1)
=
7
+27
+1
=
10
.
.
.
020
Solutie. Se observa ca 100
.
.
.
01
{z . . . 1} (7)
|
| {z } (7)
=

n+1 cifre

2n+1 cifre

este p
atrat perfect, n N , iar 10
a{z . . . 2} (7) = |10 . . . 020
{z . . . 1} (7) + 1 nu este p
| . . . 020
2n+1 cifre

2n+1 cifre

trat perfect pentru nici un n N , deoarece este cuprins ntre (7n + 1)2 si (7n + 2)2 .
Daca n N, atunci putem scrie n = 7k + r, unde k, r N, r < 7. Deoarece
n2 = 7k 0 + r0 , cu r0 {0, 1, 2, 4}, rezult
a c
a nici un p
atrat perfect scris n baza 7 nu
se termin
a cu 3, 5 sau 6. Prin urmare, r
aspunsul la ultima ntrebare este negativ.
abb . . . bc
ac
=
ca
cbb . . . ba
(termenii primei fractii continnd cte 2002 cifre b), atunci b = a + c.
Mihaela Buc
ataru, Iasi
Solutie. Daca notam n = |11 {z
. . . 1} , avem succesiv:
VII.38. Fie a, b, c cifre nenule, a 6= c. Sa se arate ca daca

2002 cifre

a 102003 + 10nb + c
10a + c
=

2003
c 10
+ 10nb + a
10c + a

102003 a2 c2 + 100nb (c a) + 10nb (a c) + 10 c2 a2 = 0

102003 (a + c) 100nb + 10nb 10 (c + a) = 0

2002
1 90bn = 0 (a + c) 10 9n 90bn = 0 a + c = b.
(a + c) 10 10

VII.39. Daca x < y < z sunt lungimile laturilor unui triunghi dreptunghic,
atunci xn + y n 6= z n , n N, n 3.
Dumitru Neagu, Iasi
Solutie. Din relatia z > y > x, rezult
a c
a z n2 > y n2 si z n2 > xn2 , oricare
ar fi n 3. De aici, obtinem ca, pentru orice n 3, avem:

z n = z n2 z 2 = z n2 x2 + y 2 > xn2 x2 + y n2 y 2 = xn + y n .
b = 60 , iar M Int ABC
VII.40. Fie ABC un triunghi ascutitunghic cu m(A)
52

\
astfel nct m(BM
C) = 150 . Notam cu P , Q, R proiectiile lui M pe BC, CA si
respectiv AB. Sa se arate ca 4P QR este dreptunghic.
Constantin Cocea, Iasi
A
Solutie. Deoarece patrulaterele M P BR si M P CQ

\
\
\
sunt inscriptibile, avem: M P R = RBM = 90 RM B
60
\
\ = 90 QM
\
si M
P Q = QCM
C. Astfel, obtinem:
\
\
\
\
\
RP
Q=M
PR + M
P Q = 180 ( RM
B + QM
C)=
\
\
= 180 (360 RM
Q BM
C)=

= 180 (360 120 150 ) = 180 90 = 90 .

Clasa a VIII-a

M
B

VIII.36. Determinati cardinalul minim al unei multimi B pentru care putem


defini functii f : R B astfel nct f (1) < 0 si f (xy) = f (x) f (y), x, y R.
Iulia Zanoschi, elev
a, Iasi
Solutie. Vom demonstra c
a multimea B trebuie s
a aib
a cel putin trei elemente
si c
a exist
a o functie care are codomeniul B format din trei elemente si ndeplineste
restul conditiilor din enunt.
Avem f (1) = f ((1) (1)) = f (1) f (1) > 0. Pe de alt
a parte, din f (0) =
f (0 (1)) = f (0) f (1), rezult
a c
a f (0) [f (1) 1] = 0, deci f (0) = 0. Prin
urmare, f (1), f (0) si f (1) sunt trei numere distincte, ceea ce nseamn
a c
a B are
cel putin
trei
elemente.
n
fine,
se
observ
a
c
a
f
:
R

{1,
0,
1},
definit
a prin

1, x < 0
0, x = 0 , verific
f (x) =
a toate conditiile cerute.

1, x > 0
VIII.37. If a, b, c (0, ) prove
the following inequalities:
a) (a + b + c)3 a3 + b3 + c3 24 where abc = 1;

3
8 3
3
3
3
b) (a + b + c) a + b + c
where ab + bc + ac = 1.
3
Zdravko Starc, Vr
sac, Serbia and Montenegro
Solutie. a) Se stie ca, oricare ar fi numerele a, b, c, are loc egalitatea:

(1)
(a + b + c)3 a3 + b3 + c3 = 3 (a + b) (b + c) (c + a) .

Avnd n vedere identitatea (1) si inegalitatea mediilor, putem scrie:

3
(a + b + c) a3 + b3 + c3 = 3 (a + b) (b + c) (c + a)

3 2 ab 2 bc 2 ca = 24abc = 24.
b) Solutia I (Irina Mustata
, eleva, Iasi). Prin nmultirea ultimelor doua paran
teze din partea dreapt
a a rela
t
cont
c
a ab + bc + ca = 1, obtinem

iei (1) si tinnd


3
3
3
3
2
(a + b + c) a + b + c = 3 (a + b) c + 1 ; similar, avem si (a + b + c)3

a + b3 + c3 = 3 (b + c) a2 + 1 si (a + b + c)3 a3 + b3 + c3 = 3 (c + a) b2 + 1 .
Prin adunarea acestora avem

(a + b + c)3 a3 + b3 + c3 = (a + b) c2 + 1 + (b + c) a2 + 1 + (c + a) b2 + 1 =
= 2 (a + b + c) + ab (a + b) + bc (b + c) + ca (c + a) =
= 2 (a + b + c) + (a + b + c) (ab + bc + ca) 3abc, adica
53

3
(a + b + c) a3 + b3 + c3 = 3 (a + b + c) 3abc.
(2)

Observam ca din 3 = 3 (ab + bc + ca) (a + b + c)2 rezulta ca a + b + c 3, iar

1
3
a abc . Revenind la (2) vom obtine
din 1 = ab + bc + ca 3 a2 b2 c2 deducem c
3
3

1
8
3
3
(a + b + c) a3 + b3 + c3 3 3 =
.
3
3
Solutia II (Marius Pachitariu, elev, Iasi). Cum ab + ac + bc = 1, avem:

a3 + b3 + c3 3abc = (a + b + c) a2 + b2 + c2 ab ac bc =
i
h
2
3
= (a + b + c) (a + b + c) 3 (ab + ac + bc) = (a + b + c) 3 (a + b + c) .

8 3
Astfel, inegalitatea de la punctul b) se va scrie 3 (a + b + c) 3abc
sau

3
8 3
a + b + c abc
.
(2)
9
Pentru a justifica inegalitatea (2), vom demonstra dubla inegalitate:
r

ab + bc + ca
a+b+c
3
(3)

abc, a, b, c > 0.
3
3
Pentru prima parte a relatiei (3), observam ca

2
a+b+c
ab + bc + ca
2

(a + b + c) 3 (ab + ac + bc)
3
3
(a b)2 + (b c)2 + (c a)2 0,

evident adevarata. Pentru partea a doua, folosim inegalitatea mediilor:


r
r
q
2

ab + bc + ca
3
3
3
ab bc ca =
abc = abc.

3
Revenind la inegalitatea (2), avem:
r
!3
r

8 3
ab + bc + ca
ab + bc + ca
=

.
a + b + c abc 3
3
3
9
Solutia III (data de autor). Din inegalitatea lui Carlson:
r
r
ab + bc + ca
3 (a + b) (b + c) (c + a)

, a, b, c > 0
8
3
si identitatea (1), rezult
a c
a:

3
(a + b + c) a3 + b3 + c3 = 3 (a + b) (b + c) (c + a)
r
!3

1
8 3
ab + bc + ca
=38 =
38
.
3
3
3 3
VIII.38. Fie n N fixat. Aratati ca exista o infinitate de numere x, y, z Z
astfel nct x2n + y 2n + z 2n = x2n+1 + y 2n+1 + z 2n+1 .
Lucian Tutescu, Craiova
54

Solutie. Dac
a lu
am z = y, atunci din relatia dat
a, obtinem:
2n
2n
2y = x (x 1) .
(1)
2n
O solutie a acestei din urm
a ecuatii putem g
asi alegnd x 1 = 2a , a Z .

2n
ntr-adev
ar, n acest caz egalitatea (1) devine 2y 2n = 2a2n + 1
2a2n , de unde
2n
g
asim y = a 2a + 1 . Deci,
a o infinitate
a:
exist
de numere cu proprietatea dat
x = 1 + 2a2n , y = a 2a2n + 1 , z = a 2a2n + 1 , a Z .
VIII.39. Fie ABCD un patrulater strmb cu [AD] [BC]. Sa se construiasca
dreptele paralele d1 , d2 , d3 , d4 astfel nct A d1 , B d2 , C d3 , D d4 si
dist (d1 , d4 ) = dist (d2 , d3 ).
Horia Mihail Teodorescu, elev, Iasi
Solutie. Fie d o dreapt
a care face unghiuri egale cu AD si BC (evident, putem
gasi o astfel de dreapta). Dreptele d1 , d2 , d3 si d4 , duse prin A, B, C, respectiv
D si paralele cu d, satisfac conditiile problemei. ntr-adevar, daca notam cu E si F
picioarele perpendicularelor din A si B pe d4 , respectiv d3 avem c
a 4AED 4BF C
(I. U.), deci AE = BF , adic
a dist (d1 , d4 ) = dist (d2 , d3 ).
VIII.40. Fie ABCDA0 B 0 C 0 D0 un cub, iar O (BB 0 ). Dreptele A0 O si C 0 O
intersecteaza (ABC) n E, respectiv F , iar AO si CO intersecteaza (A0 B 0 C 0 ) n
E 0 , respectiv F 0 .
a) Aratati ca EF E 0 F 0 nu depinde de pozitia lui O;
b) Aratati ca SBB 0 E 0 E SABCD si determinati O pentru care se atinge egalitatea.
Monica Nedelcu, Iasi
Solutie. a) Cum (A0 B 0 C 0 ) k
E
(ABC) si (EOF ) (A0 B 0 C 0 ) = A0 C 0 ,
C
B
(EOF ) (ABC) = EF , rezulta ca
F
EF k A0 C 0 , deci 4A0 OC 0 4EOF ,
E
D
A
de unde deducem c
a
EF
EO
BO
O
=
= 0 .
(1)
0
0
0
AC
OA
BO
Analog, putem demonstra ca 4AOC
C
B
F
4E 0 OF 0 , deci
0 0
0
0
EF
EO
BO
=
=
.
(2)
A
D
AC
OA
OB
0 0
EF E F
= 1, deci EF E 0 F 0 = AC 2 = const.
Din (1) si (2), obtinem
AC A0 C 0 0 0
BE
x
BE
ax
si
b) Fie B 0 O = x. Atunci, avem
=
=
. De aici rezulta ca
a ax
a
x
0
0 0
2
2
BB (B E + BE)
a
x
ax
a
SBB 0 E 0 E =
=
+

2 = a2 = SABCD .
2
2 ax
x
2
x
a
Egalitatea are loc dac
a si numai dac
a
= 1, adic
a x = , ceea ce nseamn
a c
a
ax
2
0
O este mijlocul segmentului [BB ].

Clasa a IX-a

y
IX.36. Determinati x < 0 < y astfel nct xy + = y 3 5y + 2.
x
Cezar Lupu, elev, Constanta
55

Solutie. Ecuatia dat


a este echivalent
a cu:
2
1
x + + 5 = y2 + .
(1)
x
y
1
Cum x < 0, rezult
a c
a x + + 5 2 + 5 = 3, cu egalitate numai pentru x = 1.
x
Pe de alta parte, avnd n vedere ca y > 0, putem scrie:
r
2
1 1
1 1
y 2 + = y 2 + + 3 3 y 2 = 3,
y
y y
y y
cu egalitate numai pentru y = 1. Asadar, egalitatea (1) este posibil
a dac
a si numai
2
1
2
daca x + + 5 = 3 = y + , adica pentru x = 1 si y = 1.
x
y
IX.37. Pentru x [1, ), n N , demonstrati inegalitatea

n+1
+ 1 (xn 1) 2nxn (x 1) .
x
Marius Pachitariu, elev, Iasi
Solutia I. Inegalitatea dat
a se transform
a succesiv astfel:
x2n+1 xn+1 + xn 1 2nxn+1 2nxn

x2n+1 1 (2n + 1) xn+1 xn

(1)

Inegalitatea (1) este adevarata pentru x = 1, iar pentru x > 1 este echivalenta cu
1 + x + x2 + + x2n
x2n+1 1
a din inegali (2n + 1) xn sau
xn , care rezult
x1
2n + 1
tatea mediilor n felul urm
ator:

(2n+1)2n
1 + x + x2 + + x2n
2n+1
1 x x2 x2n = x 2(2n+1) = xn .

2n + 1
Solutia II (Irina Mustata
a, Iasi). Prin inductie complet
a.
, elev
n+1
n+1
n+1
y
z
x
+ n + n x + y + z, x, y, z > 0, n N.
IX.38. Sa se arate ca
yn
z
x
Gigel Buth, Satu Mare
Solutie. n GM - 4/2002, p. 146, L. Panaitopol enunta si demonstreaz
a rezultatul
urm
ator:
Daca p 1 si ai 0, bi > 0 pentru i 1, n, atunci
p
Xn
n
ai
X
api
i=1
p1 .
p1 Xn
b
i=1 i
b
i=1

Inegalitatea din enunt rezulta imediat din aceasta.


1
2
1
+ q
=
IX.39. Sa se rezolve ecuatia q
.
[x] [x + 2]
3
3
3
2 [x]
3 [x] [x + 1]
Daniel Jinga, Pitesti
Solutie. Ecuatia are sens daca [x] > 0, adica [x] 1. Daca facem notatia
a devine:
[x] = y N , ecuatia dat
1
1
2
.
(1)
+
=
2y y 3 (y + 1) 3 y
y (y + 2)
56

y+1
y+2

Deoarece y = y 1
(2) si 3 y = 3 y 1 1
(3), rezult
a c
a
2
3
1
1
1
1
2
+
=
. Prin urmare, ecuatia
+

2y y 3 (y + 1) 3 y
y (y + 1) (y + 1) (y + 2)
y (y + 2)
(1) are solutie dac
a si numai dac
a (2) si (3) sunt simultan egalit
ati, adic
a y = 1. Deci
solutia ecuatiei date este x [1, 2).
IX.40. Fie M 6= G n planul 4ABC si D, E, F mijloacele laturilor [BC],

[CA] si respectiv [AB]. Consideram punctele X, Y, Z astfel nct XD = mXM ,

Y E = mY M , ZF = mZM , m 6= 1.
2m
3
SM .
a) Daca m 6= , atunci AX, BY, CZ sunt concurente n S, cu SG =
2
3
3
b) Daca m = , atunci AX, BY, CZ sunt paralele cu GM .
2
Virgil Nicula, Bucuresti
Solutie. a) Avem:
2m
2m
2m 3
SM SM + M G =
SM
SM = M G
SG =
3
3
3


3
1
MS =
MG MS =
MA + MB + MC .
3 2m
3 2m

1
M A + M B + M C . Se poate
Fie punctul S 0 definit prin egalitatea M S 0 =
3 2m
0
verifica, prin calcul, faptul c
a S apartine dreptelor AX, BY , CZ, deci acestea vor

2m
0
SM . S
a
arat
a si egalitatea SG =
fi concurente n S S si atunci este adev
3
0
demonstram, de exemplu, ca S AX. Pentru aceasta vom demonstra ca vectorii

XS 0 si S 0 A sunt coliniari:

0 0 M A + M B + M C
MB + MC
XS = M S M X =

=
3 2m
2 2m

(2 2m) M A M B M C
,
=
(3 2m) (2 2m)

0 0 M A + M B + M C

S A = MA MS = MA
= (2 2m) XS 0 .
3 2m
3
b) Pentru m = , avem:
2
3


3
XD = XM XM + M D = XM M X = 2M D = (M B + M C)
2
2

si atunci XA = M A M X = M A + M B + M C = 3M G. Analog se obtine


Y B = ZC = 3M G, deci dreptele AX, BY , CZ sunt paralele.

Clasa a X-a

X.36. Sa se rezolve inecuatia alogb x + xlogb x a + b, unde a, b (1, ).


Daniela Dodan, elev
a, Iasi
2
Solutie. Din egalitatea x = blogb x , x > 0, rezult
a c
a xlogb x = blogb x , x > 0.
Deci, inecuatia data este echivalenta cu
2

alogb x + blogb x a + b.
57

(1)

Dac
a facem notatia log2b x = 0 si avem n vedere obsevatiile > 1 a + b >
> a + b, 1 a + b a + b, obtinem ca inecuatia (1) este echivalenta cu
log2b x 1, deci x b1 , b .

X.37. Fie a, b (0, 1) (1, ) si functia injectiva f : (0, ) R astfel nct


functia g : R R, g (x) = f (ax ) + f (bx ) este constanta. Sa se arate ca ab = 1 si
ca exista functii f care satisfac ipotezele problemei.
Dan Popescu, Suceava
Solutie. Fie
= f(ax ) +f (bx ) = k, x R, unde k R. Atunci,
avem
logg (x)

k = f (x) +f b a x = f alogb x + f (x), x > 0, de unde rezulta ca f bloga x =


= f alogb x . Cum f este functie injectiv
a, deducem c
a bloga x = alogb x , x > 0, deci
g (x)
a a = b sau ab = 1. Dac
a a = b, atunci f (ax ) =
log2a b = 1, adic
, x R,
2
k
1
sau f (x) = , x > 0, ceea ce contrazice injectivitatea functiei f . Pentru b = si
2
a
f (x) = loga x, se obtine g (x) = 0, x R.

X.38. Fie a, b, c, d R cu a > b > c > d. Sa se arate ca a, b, c, d sunt n progresie

3
ad
aritmetica daca si numai daca (a b) (b c) (c d) =
.
3
A. V. Mihai, Bucuresti
Solutie. Dac
a a, b, c, d sunt n progresie aritmetic
a de ratie r, atunci egalitatea
3
3r
data este echivalenta cu r r r =
, care este, evident, adevarata.
3
p
Reciproc, dac
a are loc egalitateap
din enunt, atunci ad = 3 3 (a b) (b c) (c d),
sau (a b) + (b c) + (c d) = 3 3 (a b) (b c) (c d), adica media aritmetica a
numerelor a b, b c si c d este egala cu media lor geometrica. De aici rezulta ca
a b = b c = c d, deci a, b, c, d sunt n progresie aritmetic
a.
X.39. Fie ABCDA0 B 0 C 0 D0 un paralelipiped dreptunghic cu dimensiunile AB = a,
AD = b, AA0 = c. Daca M Int A0 B 0 C 0 D0 , notam cu , , masurile unghiurilor
pe care AM le face cu AB, AD si respectiv AA0 . Sa se arate ca
AM < a cos + b cos + c cos < AC 0 .
C
at
alin Calistru, Iasi
Solutie. Fie E, F si G proiectiile punctului M
D
C
pe laturile AB, AD si respectiv AA0 . Astfel avem
AE
AF
AG
si cos =
cos =
, cos =
, de A
B
AM
AM
AM
unde deducem ca AE cos + AF cos + AG cos =
M
AE 2 + AF 2 + AG2
AM 2
=
=
= AM . De aici, G
AM
AM
C
D
avnd n vedere ca AE < AB = a, AF < AD = b,
F
AG < AA0 = c si cos > 0, cos > 0, cos > 0,
rezult
a c
a AM < a cos + b cos + c cos . Pentru
A
E
B
a doua parte a inegalitatii vom folosi inegalitatea lui
Cauchy-Buniakovski-Schwarz si identitatea cos2 + cos2 + cos2 = 1:
p
p
a cos + b cos + c cos < a2 + b2 + c2 cos2 + cos2 + cos2 = AC 0 .
58

X.40. a) Pentru x, y, z 0, demonstrati inegalitatea


p

x + y + x + z + y + z xy + xz + yz 3 6xyz.

b) Cu notatiile uzuale, n orice triunghi are loc inegalitatea

2
2
a

b
+
(
a

c)
+
b

c
R
9
.
2

r
4
a+ b+ c

Marian Tetiva, Brlad


Solutie. a) Din relatiile
p
p

x + y + x + z + y + z 3 6 (x + y) (x + z) (y + z) 3 6 8xyz = 3 2 6 xyz si
q p
p

xy + xz + yz 3 3 x2 y 2 z 2 = 3 6 x2 y 2 z 2

rezult
a c
a

p

x + y + x + z + y + z xy + xz + yz 3 6 xyz = 3 6xyz.

b) Vom aplica inegalitatea de la punctul a) pentru x = pa, y = pb si z = pc.


Cu notatiile facute, avem:
x + y = c, x + z = b, y + z = a,
X
X

xy + xz + yz =
(p a) (p b) =
p2 (a + b) p + ab =
X

= 3p2 4p2 +
ab = p2 + p2 + r2 + 4Rr = r2 + 4Rr,

S2
= pr2 .
p

2
a+ b+ c
r + 4Rr 3 6pr2 sau
Astfel, inegalitatea de la a) devine

2 2
R
27 (a + b + c)
a+ b+ c
. De aici
r + 4Rr 54pr2 , deci 1 + 4

r
a+ b+ c
obtinem c
a

2
a+ b+ c
9
R
27 (a + b + c)
9 3 (a + b + c)
=
=
2

2 4
2
r
4
4
a+ b+ c
a+ b+ c

2
2
a

b
+
(
a

c)
+
b

c
9
.
=

2
4
a+ b+ c
xyz = (p a) (p b) (p c) =

Clasa a XI-a

XI.36. Fie D, M doua matrice nesingulare de ordin n, D diagonala, iar M


triunghiulara. Daca D = t M DM , sa se arate ca M este tot o matrice diagonala,
avnd 1 pe diagonala principala.
Adrian Corduneanu, Iasi
Solutie.
59

1
m11 m12 . . . m1n
0 ... 0
0
0
2 . . . 0
m22 . . . m2n

Fie D =
. . . . . . . . . . . ., cu i 6= 0, i = 1, n, M = . . .
... ...
...
0
0 . . . n
0
0
. . . mnn
mii 6= 0, i = 1, n si Mij complementul algebric al lui mij n matricea M . Not
am cu
d = det M 6= 0. Relatia dat
a este echivalent
a cu DM 1 = t M D. Deoarece

0 ... 0
M11 M21 . . . Mn1
1

1 0
2 . . . 0
M22 . . . Mn2
0
=
DM 1 =

...
... ...
...
d ... ... ... ...
0
0 . . . n
0
0
. . . Mnn

1 M11 1 M21 . . . 1 Mn1


1
0
2 M22 . . . 2 Mn2

si
=

...
...
...
...
d
0
0
. . . n Mnn

0
...
0
0 ... 0
1
m11

m12 m22 . . .
2 . . . 0
0
t
0
=
MD =

...
... ...
...
. . . . . . . . . . . .
m1n m2n . . . mnn
0
0 . . . n

0
...
0
1 m11
1 m12 2 m22 . . .
0

=
...
...
...
...
1 m1n 2 m2n . . . n mnn

i Mii
= i mii , i = 1, n. Din mij = 0,
d
Mii
1
pentru i < j, deducem ca M este matrice diagonala si atunci
, i = 1, n,
=
d
mii
a mii = 1, i = 1, n. Cazul n care M este inferior
deci avem m2ii = 1, i = 1, n, adic
triunghiulara se trateaza n mod analog.
rezulta ca mij = 0, oricare ar fi i < j si

XI.37. Fie A M3 (C) astfel nct det (A + tA) = 0, unde R\{1, 0, 1}.
2 ( 1)2
det A.
Sa se arate ca det (A + tA) =

Marian Ionescu, Pitesti si Lucian Tutescu, Craiova


a cu gradul
Solutie. P (x) = det (A + x tA), x C este o functie polinominal
mai mic sau egal cu 3. Deoarece

1
1
t
3
t
P (x) = det x
= x det
A+ A
A+ A =
x
x


1t
1
= x3 det
A + A = x3 P
, x C ,
x
x

rezult
a c
a P este polinom reciproc, deci P (x) =
(detA) x3 + ax2 + ax + det A.
1
Cum, prin ipotez
a, P () = 0, nseamn
a c
a si P
= 0. De aici, avnd n

1
vedere c
a P (1) = 0, obtinem c
a P (x) = (det A) (x ) x
(x + 1). Asadar,

60

1
2 ( 1)2
det (A + A) = P (1) = (det A) (1 ) 1
2=
det A.

XI.38. Sa se determine functiile continue f : [0, ) [0, ) pentru care


f (f (x)) + 2f (x) = 3x, x 0.
Mihail Bencze, Brasov
Solutie. Fie fn (x) = f f f (x), x 0, n N . Pentru orice n 3
{z
}
|
n ori

si x 0, avem c
a fk (x) + 2fk1 (x) = 3fk2 (x), k = 2, n, de unde prin sumare
deducem c
a fn (x) + 3fn1 (x) = f1 (x) + 3x, n 3, x 0. De aici, obtinem:
f3 (x) + 3f2 (x) = f1 (x) + 3x,
f4 (x) + 3f3 (x) = f1 (x) + 3x,
f5 (x) + 3f4 (x) = f1 (x) + 3x,
....................................
fn (x) + 3fn1 (x) = f1 (x) + 3x, n 3, x 0.

0
1
1
1
, a doua cu
, a treia cu
Mai departe, nmultind prima ecuatie cu
3
3

2
1

etc. si apoi adunndu-le, g


asim relatia
3

n3

2
n3 !
1
1
1
1

fn (x)+3f2 (x) = (f1 (x) + 3x) 1 +


+ +
+
3
3
3
3
sau

n3
n2 !
3
1
1
fn (x) + 9x 6f (x) = (f1 (x) + 3x) 1
, n 3, x 0.

3
4
3
(1)
n
3x
3
Din ipoteza rezulta ca f (x)
x, n N ,
, x 0, si atunci fn (x)
2
2
x
fn (x)
n , n N , x 0, adic
a
x 0. De aici, obtinem c
a0
3n
2
fn (x)
= 0, x 0.
(2)
lim
n 3n
3
Din (1) si (2), rezulta ca 9x6f (x) = (f (x) + 3x), x 0, deci f (x) = x, x 0.
4
Observatie. Nu este nevoie de continuitatea functiei f .
n
P
yi
este convergent. Daca
XI.39. Fie sirul (yn )n1 astfel nct sirul
i=1

n1

(x )
R+ are proprietatea ca xn xn+1 (1 + xn yn+1 ), n 1, aratati ca sirul

n n1
1
este convergent.
xn n1
Gheorghe Molea, Curtea de Arges
Solutie. Deoarece xn > 0, n 1, rezulta ca inegalitatea din enunt este echiva61

lent
a cu

1
xn+1

1
yn+1 , n 1, de unde deducem c
a
xn
n
X
1
1

y2 + y3 + + yn =
yi y1 ,
xn x1
i=1

n 1.

Cum n
partea
a a ultimei relatii este un sir convergent, deci m
arginit, rezult
a
dreapt
1
este si el m
arginit.
c
a sirul
xn n1
n
n1
P
P
1
1

yn =
yi
yi este echivalent
a cu
Pe de alt
a parte, relatia
xn
xn1
i=1
i=1
n
n1
n
P
P
P
1
1
1

yi

yi , n 2, sau, cu notatia zn =

yi , zn zn1 ,
xn i=1
xn1
xn i=1
i=1
n 2. De aici, avnd n vedere ca sirul (zn )n1 este marginit, fiind diferenta a
dou
a siruri m
arginite, rezult
a c
a sirul (zn )n1 este convergent. Prin urmare, sirul cu
n
P
1
= zn +
yi este convergent.
termenul general
xn
i=1
3
XI.40. Fie x0 [1, 1];
aratati ca pentru orice n N, ecuatia 3x4x = xn are
1 1
o singura solutie xn+1 , . Demonstrati ca sirurile (xn )n0 si (3n xn )n0
2 2
sunt convergente si calculati limitele lor.
Marian Tetiva, Brlad
Solutie. S
a ar
at
am, pentru nceput,
c
a
dac
a
a

[1,
1],
atunci ecuatia 3x4x3 = a

1 1
are o singur
a solutie n intervalul , . Pentru aceasta, consider
am functia
2 2


1
1
f : R R , definit
a prin f (x) = 3x 4x3 a. Deoarece f
f
=
2
2
= (1 a) (1 a) 0 si f este continua, rezulta ca f se anuleaz
a cel putin o

1 1
1 1
0
2
data n intervalul , . Cum f (x) = 3 1 4x 0, x , , nseamna
2 2
2 2

1 1
ca f este strict crescatoare pe , , deci ecuatia f (x) = 0 are o singura solutie
2 2

1 1
n intervalul , .
2 2

Mai putem observa c


a, dac
a = arcsin a, avem 3 sin 4 sin3 = sin = a
3
3

h i

1 1

si ,
implic
a sin , . Deci sin este tocmai solutia din in3
2 2
3
2 2
1 1
tervalul ,
a ecuatiei 3x 4x3 = a. Astfel, am demonstrat ca, pentru orice
2 2

1 1
3
si anume
a [1, 1], ecuatia 3x 4x = a are o singura solutie n intervalul ,
2 2

arcsin a
sin = sin
.
3
3
Revenind la problema noastr
a, rezult
a, din cele ar
atate mai sus, c
a, pentru
arcsin xn
. De aici, obtinem c
a
orice n N, xn+1 este bine definit si xn+1 = sin
3
62

arcsin xn
arcsin x0
, n N. Astfel avem
, n N, deci arcsin xn =
3
3n
lim xn = lim sin (arcsin xn ) = 0 si

arcsin xn+1 =
n

lim 3n xn = lim (3n arcsin xn )

xn
= arcsin x0 .
arcsin xn

Clasa a XII-a

XII.36. Sa se determine n N, n 2 pentru care ecuatia x2 = x + b


1 are solutie
unica n Zn ; rezolvati ecuatia n acest caz.
Andrei Nedelcu, Iasi
Solutie. Dac
ab
a Zn este solutie a ecuatiei x2 = x + b
1, atunci si b
1b
a este
2
2
b
b
c
b
c
solutie a acestei ecuatii ((1 b
a) = 1 2a + b
a = 1 2a + b
a+b
1 = (b
1b
a) + 1). Cum
\
ecuatia trebuie sa aiba solutie unica, este necesar sa avem b
a=b
1b
a, sau 2a
1=b
0.
cb
Deoarece b
a2 = b
a + 1 implica b
4b
a2 = b
4b
a+b
4, sau (2a
1)2 = b
5, rezulta ca b
5=b
0. De
aici, obtinem c
a n = 5 si atunci ecuatia dat
a are solutia unic
ab
a=b
3.

XII.37. Fie (G, +) un subgrup al grupului (R, +). Sa se determine morfismele


crescatoare de la (G, +) la (R, +).
Dan S
tefan Marinescu si Viorel Cornea, Hunedoara
Solutie. Daca G = {0}, atunci f : G R, f (0) = 0 este functia cautata.

f (x0 )
,
x0
observam ca a 0. Folosind definitia morfismului de grupuri se poate demonstra
prin inductie ca f (nx) = nf (x), n Z, x G. De aici, deducem ca f (nx0 ) =
= nf (x0 ) = nax0 , n Z.
Fie un element oarecare y G. Dac
a x0 > 0, avem succesiv:

n (y + x0 )
n (y + x0 )
n (y + x0 )
<

+ 1, n N
x0
x0
x0


n (y + x0 )
n (y + x0 )
f
x0 f (n (y + x0 )) f
+ 1 x0 , n N
x0
x0

n (y + x0 )
n (y + x0 )
ax0 n (f (y) + ax0 )
+ 1 ax0 , n N
x0
x0

1 n (y + x0 )
1 n (y + x0 )
1
ax0 f (y) + ax0
+
ax0 , n N ,
n
x0
n
x0
n
Daca G 6= {0}, atunci exista x0 G \ {0} si atunci daca notam a =

de unde, trecnd la limit


a pentru n , obtinem
y + x0
y + x0
ax0
f (y) + ax0 ax0
,
x0
x0
deci f (y) = ay. Daca x0 < 0, se ajunge, n mod analog, la acelasi rezultat.
n sfrsit, observ
am c
a functia f : G R, f (y) = ay, a > 0, este un morfism
cresc
ator de grupuri.
XII.38. Determinati functiile derivabile f, g : R R astfel nct f 0 (x) = g (x)+x
si g 0 (x) = f (x) x, x R.
Gheorghe Iurea, Iasi
63

Solutie. Adunnd cele dou


a relatii date, obtinem (f + g) (x) = (f + g) (x),
0
x R, sau (ex (f + g)) (x) = 0, de unde gasim f (x) + g (x) = Cex , x R, unde
C R este o constanta arbitrara. Revenind la prima ecuatie, avem
f 0 (x) = Cex + x f (x) ,

x R,
C x
sau (ex f (x))0 = Ce2x + xex ,x R, deci f (x) =
e + k1 ex + x 1, x R.
2
C x
Analog, obtinem g (x) =
a usor c
a aceste
e k2 ex x + 1, x R. Se verific
2
functii satisfac sistemul de ecuatii dat.
XII.39. Fie f, g : (0, ) R astfel nct lim f (x) = lim g (x) = , iar
x
Z 1x
f (x)
xg(n)
lim
= R. Sa se calculeze lim f (n)
dx, unde [1, ).
x g (x)
n
0 x+
Adrian Sandovici, Piatra Neamt
Solutie. Din ipoteza rezulta ca exista n0 astfel nct f (n) > 0 si g (n) > 0,
n n0 . Pentru n n0 , avem:
Z 1 g(n)
Z
x
dx
f (n) 1 g(n) 0 x
In = f (n)
x
=
dx =
x+
g (n) 0
x+
0
#
"
1
Z 1
f (n) xg(n)+1
xg(n)

=
2 dx .
g (n)
x + 0
0 (x + )
Deoarece

xg(n) dx

1
xg(n) dx =
g (n) + 1
(x + )2
0
Z 1 g(n)

x
dx
si lim g (n) = , rezulta ca lim
= 0. Asadar, avem lim In =
.
2
n
n 0 (x + )
n
1+
XII.40. Fie f : [0, 1] R o functie derivabila cu derivata continua astfel nct
f (x)
0
exista si este finita. Sa se arate ca
xf (x) f (x), x [0, 1], iar lim
x0 x
x>0
Z 1

Z 1
f (x)
f (x) dx,
f (1) min 2
dx .
x
0
0
Marcel
Chirita
,ZBucuresti
Z
0

Solutie. Din xf 0 (x) f (x), x [0, 1] rezult


a c
a
xf 0 (x) dx
0
Z 1
Z 1
Z 1
1
f (x) dx
f (x) dx, deci f (1) 2
f (x) dx (1).
sau xf (x)|0
0

f (x) dx,

f (x)
exista si este finita, rezulta ca lim f (x) = 0. Astfel, avem
Deoarece lim
x&0 x
x&0
Z 1
Z 1
Z 1
f (x)
f (x)
f 0 (x) dx = lim (f (1) f ()) = f (1) ,
dx = lim
dx lim
&0
&0
&0
x
x
0

ceea ce, mpreun


a cu relatia (1), conduce la inegalitatea din enunt.

64

Solutiile problemelor pentru preg


atirea concursurilor
din nr. 1 / 2003
A. Nivel gimnazial
G36. Fie x, n N astfel nct x divide 10n 1, nsa x nu divide 10k 1 pentru
.
k < n. Sa se arate ca x divide 10m 1 daca si numai daca m .. n.
N. N. Hrtan, Iasi
..

Solutie. Daca m . n, atunci m = 0 sau exista q N astfel nct m = nq. n


.
primul caz, avem 10m 1 = 0 .. x, iar n al doilea avem:
.
10m 1 = (10n )q 1 = (10n 1) ((10n )q1 + + 1) .. x.
.
Sa presupunem acum ca 10m 1 .. x, m 6= 0 si m = nq + r, cu 0 < r < n. De aici

si din ipoteza, obtinem ca x | 10m 1 (10nq 1) = 10nq+r 10nq = 10nq (10r 1).
Deoarece din x | 10n 1 rezult
a c
a (x, 10) = 1, deci (x, 10nq ) = 1, din relatia
r
precedent
a deducem c
a x | 10 1, ceea ce contrazice ipoteza. Prin urmare, dac
a
.
.
10m 1 .. x, atunci m .. n.
G37. 2n muzicieni (n > 2) participa la un festival. La fiecare concert, o parte
dintre ei cnta iar ceilalti asculta. Sa se determine numarul minim de concerte astfel
nct fiecare muzician sa-i asculte pe toti ceilalti.
Titu Zvonaru, Bucuresti
Solutie. Fie a1 , a2 , . . ., a2n cei 2n muzicieni. Dac
a la un concert, unul dintre
ei asculta pe un coleg care cnta, spunem ca are loc o "auditie". pAstfel, la un
concert la care cnta p muzicieni, exista p (2n p) auditii. Deoarece p (2n p)
p + 2n p
a c
a num
arul maxim de auditii are
= n, adic
a p (2n p) n2 , rezult

2
loc atunci cnd n muzicieni cnt
a si n ascult
a.
Sa presupunem ca la primul concert cnta muzicienii a1 , a2 , . . ., an . Pentru a
putea fi ascultat de a2 , a3 , . . ., an , muzicianul a1 trebuie sa mai cnte la un concert
n care s
a nu cnte a2 , a3 , . . ., an , apoi a2 trebuie s
a cnte ntr-un concert n care
nu cnt
a a1 , a3 , . . ., an si asa mai departe. Deci, num
arul minim de concerte este
cel putin n + 1. Sa aratam ca acest numar este n + 1 indicnd o aranjare a celor
n + 1 concerte astfel nct sa fie ndeplinita cerinta problemei. Pentru aceasta, facem
notatiile:
Ak = {an+1 , an+2 , . . . , a2n } {an+k } ,

Bk = {a1 , a2 , . . . an } {ak } ,

k = 1, n

si repartiz
am muzicienii astfel:
Muzicieni care cnta Muzicieni care asculta
1)
a1 , a2 , . . ., an
an+1 , an+2 , . . ., a2n
2)
a1 , A1
B1 , an+1
3)
a2 , A2
B2 , an+2
...
................
....................
n)
an1 , An1
Bn1 , a2n1
n + 1) an , An
Bn , a2n
G38. Multimea A Z are cinci elemente. Adunnd n toate modurile posibile
65

cte trei elemente din multime, obtinem urmatoarele 10 sume: 3, 6, 8, 10, 11, 13,
15, 16, 18, 20. Determinati multimea A. (n legatura cu o problema de concurs din
Iugoslavia.)
Gabriel Popa, Iasi
Solutie. Fie A = {a, b, c, d, e} cu a < b < c < d < e si fie S = a + b + c + d + e.
Adunnd cele 10 sume din enunt obtinem 120. Deoarece n aceasta suma fiecare
element al multimii A apare exact de 6 ori, rezult
a c
a 6S = 120, deci S = 20.
Prin urmare, adunnd n toate modurile posibile cte dou
a elemente din multime,
obtinem urmatoarele 10 rezultate: 17 (= 20 3), 14 (= 20 6), 12, 10, 9, 7, 5, 4,
2, 0. Acum, din a + b = 0 si a + c = 2, gasim b = a si c = 2 a. Apoi, din
e + d = 17 si e + c = 14, deducem c
a d = e + 17 si c = e + 14, deci e = a + 12.
Asadar, putem scrie A = {a, a, a + 2, a + 5, a + 12}. A treia sum
a (n ordine
crescatoare) este a + d sau b + c si este egala cu 4. Cum a + d = a a + 5 = 5, ramne
doar b + c = a a + 2 = 2a + 2 = 4, de unde rezulta ca a = 1. n concluzie,
avem A = {1, 1, 3, 6, 11}.
G39. Fie xi R, i = 1, n, unde n 2003, astfel nct

x1 (n + 1) x2 + nx3 n 1

................................................
xn2 (n + 1) xn1 + nxn n 1

xn1 (n + 1) xn + nx1 n 1 n2

xn (n + 1) x1 + nx2 2n 1.
Daca x1 = 1, sa se calculeze x2003 .

Romeo Cernat, Iasi


Solutie. Cu notatiile yk = xk k + 1, k = 1, n, sistemul dat devine:

y1 (n + 1) y2 +ny3 0, y2 (n + 1) y3 +ny4 0, . . . , yn (n + 1) y1 +ny2 0, (1)

de unde, prin adunare, obtinem 0 0. Asadar, toate inegalit


atile din sistemul (1)
trebuie sa fie egalitati, adica
y1 y2 = n (y2 y3 ) , y2 y3 = n (y3 y4 ) , . . . , yn y1 = n (y1 y2 ) . (2)

nmultind membru cu membru relatiile din sistemul (2), obtinem

(y1 y2 ) (y2 y3 ) . . . (yn1 yn ) (yn y1 ) =


= nn (y1 y2 ) (y2 y3 ) . . . (yn1 yn ) (yn y1 ) ,

de unde rezulta ca una din paranteze este zero si atunci conform cu (2), avem y1 =
= y2 = = yn . Prin urmare, 1 = x1 = x2 1 = x3 2 = = xn n + 1, deci
x2003 = 2003.
G40. Comparati numerele reale a si b, stiind ca a2 14a + b2 + 6b + 33 = 0.
Bogdan R
aducanu, elev, Iasi
Solutia I (un grup de elevi de la Colegiul National din Iasi si Alexandru
Negrescu, elev, Botosani). Relatia din enunt se scrie (a 7)2 + (b + 3)2 = 25.
Deducem c
a |a 7| 5 si |b + 3| 5, de unde a [2, 12] si b [8, 2]. Deci
b 2 a si cum a 6= b (prin verificare direct
a), rezult
a c
a a > b.
Solutia II. Dac
a facem notatia a = b + t problema revine la compararea lui t cu
zero. nlocuind pe a n egalitatea dat
a, obtinem 2b2 + 2b (t 4) + t2 14t + 33 = 0,
ecuatie care are solutie daca si numai daca = t2 8t + 16 2t2 + 28t 66 =
66

= t2 + 20t 50 0. De aici, rezult


a c
a t 10 5 2, 10 + 5 2 , deci t > 0.
Asadar, avem a > b.
G41. Daca 0 < x y z, sa se arate ca
x z
y2
z2
y
x y
z
x
z
x2
3 + + + + + 1+ 2 + 2 + 2.
z
y x
y
z
x
z
x
y
z
x
Ovidiu Pop, Satu Mare
Solutie. Prima inegalitate rezulta din inegalitatea mediilor. Inegalitatea a doua
este echivalent
a cu x2 y + z 2 x + y 2 z x2 z + y 2 x+ z 2 y, sau (y x) (z x) (z y) 0,
care este adev
arat
a n virtutea ipotezei 0 < x y z. Inegalitatea a treia este
x
y
x
echivalenta cu + + 1, adica (z y) (x y) 0, care este adevarata. n
y
z
z
sfrsit, pentru a demonstra ultima inegalitate vom folosi din nou inegalitatea medi 2
2 2 2
2 2
y
x
y
z
z
z
x
x
+
2 ,
+
+
3,
+1 2 .
ilor. Avem:
y
z
z y
z
x
x
x
x
x2
y2
z2
z
Adunnd aceste relatii, obtinem 2
+
+

2
+
1
+
, q.e.d.
y2
z2
x2
z
x
G42. Determinati a, b R, daca [x] + [x + a] = [bx], x R.
Gheorghe Iurea, Iasi
Solutie. Lund x = 0 n egalitatea data gasim [a] = 0, deci a [0, 1). Din relatiile
x 1 + x + a 1 < [x] + [x + a] = [bx] bx, x R, rezult
a c
a a 2 x (b 2),
x R, deci b = 2. Asadar,
avem
[x]
+
[x
+
a]
=
[2x],
x

R,

de unde, avnd
1
1
n vedere c
a [x] + x +
= [2x], deducem c
a [x + a] = x + , x R (1). De
2

1
1
1 3
1
aici, lund x = , obtinem a +
[1, 2), adic
a a
,
[0, 1) =
,1 .
2
2

2
2 2
1
1
1
si avem 0 < x0 + <
Dac
aa
, 1 , atunci putem alege un x0 1 a,
2
2
2

1
a x0 +
< 1 < x0 + a. n acest caz ns
= 0, iar [x0 + a] 1, deci relatia (1) nu
2
1
este valabil
a. Prin urmare, avem a = , b = 2, valori care verific
a egalitatea dat
a.
2
d un unghi oarecare si P un punct n interiorul sau. Se considera
G43. Fie xOy
punctele A, B [Ox cu A (OB) si C, D [Oy cu C (OD) astfel nct triunghiurile P AB si P CD sa fie echilaterale. Aratati ca dreptele OP , AD si BC sunt
concurente daca si numai daca P se afla pe bisectoarea unghiului dat.
Temistocle Brsan, Iasi
Solutie. Fie OP BD = {M }, P R CD,
D
P Q AB (R CD, Q AB) si P Q = d1 , P R = d2 ,
R
OP = d. Avem:
C d2
q
MB
OB d1
OB sin 1
=
, OQ = d2 d21 ,
=
M
MD
OD sin 2
OD d2
P
q

2
d
OA = OQ AQ = d2 d21 d1 / 3,
d1

q
1

2
2
OB = OQ + QB = d d1 + d1 / 3,
O
A Q B
67

q
q
q

OR = d2 d21 , OC = d2 d22 d2 / 3, OD = d2 d22 + d2 / 3.


Cu aceste observatii, putem scrie succesiv: OP , AD si BC sunt concurente

M B CD AO
OB d1 2d2 / 3
OA
= 1 OA OB = OC OD

=1

M D CO AB
OD d2
OC
2d1 / 3
q
q
q

2
2
2
2
2
2
2
2
d d1 d1 / 3
d d1 + d1 / 3 = d d2 d2 / 3
d d2 + d2 / 3
d2 d21 d21 /3 = d2 d22 d22 /3 d1 = d2 1 = 2 .

G44. Fie V ABC o piramida, iar G centrul de greutate al 4ABC. Un plan


ce trece prin G taie dreptele V A, V B, V C n A0 , B 0 si respectiv C 0 . Sa se arate ca
VB
VC
VA
+
+
= 3.
0
0
VA
VB
V C0
Constantin Cocea, Iasi
Solutie. Fie {N } = B 0 C 0 BC si {M } = A0 C 0 AC.
V
Aplicnd teorema lui Menelaus n triunghiurile V AC
A0 A
C 0V
MC
si V BC, obtinem:
0
= 1 si
C
0
A
V
C
C
MA
0
0
B B C V NC
M
A
C

= 1, de unde rezult
a c
a
B0V C 0C N B

G
CC 0 AM
NB
CC 0
A0 A B 0 B
+
=
sau
+
=
N
B
0
0
0
0
VA
VB
V C MC
NC
VC
0
0
0
VA VA VB VB
VC VC
+
=
sau
A
V A0
V B0
V C0
B
VA
VB
VC
AM
BN
+
+
=
3.
(Am
folosit
c
a
G

M
N
implic
a
+
= 1.)
0
0
0
VA
VB
VC
MC
NC
G45. Fie SABC un tetraedru n care 4ABC nu este echilateral, iar muchiile [SA] , [SB] , [SC] nu sunt toate congruente. Demonstrati ca exista sase puncte
A1 , B1 , C1 , A2 , B2 , C2 pe dreptele SA, SB, SC, BC, AC si respectiv AB astfel ca patrulaterele A1 B1 A2 B2 , B1 C1 B2 C2 si A1 C1 A2 C2 sa fie trapeze isoscele (A1 B1 kA2 B2 ,
A1 C1 kA2 C2 , B1 C1 kB2 C2 ) daca si numai daca

SA2 AB 2 AC 2 + SB 2 BC 2 BA2 + SC 2 CA2 CB 2 = 0.


Daly Marciuc, Satu Mare
Solutie. S
a presupunem c
a A1 B1 A2 B2 , B1 C1 B2 C2 si A1 C1 A2 C2 sunt trapeze
isoscele n modul indicat. Din A1 B1 k A2 B2 rezulta ca B2 A2 k AB si apoi, analog,
rezulta ca B2 C2 k BC si A2 C2 k AC. De aici, deducem ca AB2 A2 C2 si BA2 B2 C2
sunt paralelograme, deci C2 este mijlocul lui AB. Analog, obtinem c
a B2 si A2 sunt
mijloacele laturilor AC si BC.
Din A1 B1 k AB, A1 C1 k AC si B1 C1 k BC rezulta ca
A1 A
B1 B
C1 C
=
=
= k.
(1)
SA
SB
SC
Notnd BC = a, AC = b si AB = c, avem: A1 B22 = A2 B12
b2
a2
SA2 + b2 SC 2
SB 2 + a2 SC 2
k 2 SA2 +
k
= k 2 SB 2 +
k
, deci
4
2
4
2
2
2
2
2
2
2
(2)
A1 B2 = A2 B1 2k SB SA = a b .
68

n mod analog, g
asim echivalenta:

B1 C22 = C1 B22 2k SB 2 SC 2 = c2 b2 .

n fine, din (2) si (3) rezult


a c
a
2

2
2
SA c b + SB 2 a2 c2 + SC 2 b2 a2 = 0.

(3)

(4)

Reciproc, relatia (4) poate fi scrisa astfel:

a2 b2
c2 b2
not
= k.
=
2
2
2 (SB SA )
2 (SB 2 SC 2 )

(5)

Alegem A1 , B1 , C1 pe SA, SB, SC astfel nct s


a avem relatia (1). n acest caz din
(5) rezulta ca A1 B1 A2 B2 si B1 C1 B2 C2 sunt trapeze isoscele, unde A2 , B2 si C2 sunt
mijloacele laturilor BC, AC si AB (A1 B1 k AB k A2 B2 etc.). Daca A1 B1 A2 B2 si
a c
a A1 A2 = B1 B2 si B1 B2 = C1 C2 , deci
B1 C1 B2 C2 sunt trapeze isoscele nseamn
A1 A2 = C1 C2 , adic
a si A1 C1 A2 C2 este isoscel.

B. Nivel liceal
L36. Fie 4ABC si M triunghiul sau median. Daca P este un punct aflat n
interiorul sau pe laturile lui M, iar A0 , B 0 , C 0 sunt intersectiile dreptelor AP , BP ,
1
8
AP BP CP
CP cu laturile BC, CA si respectiv AB, atunci <

.
4
AA0 BB 0 CC 0
27
Marian Ionescu, Pitesti
Solutie. Not
am S1 = (P BC), S2 = (P CA), S3 = (P AB) si S = (ABC).
S2 + S3
AP
si analoagele si se deduce relatia lui
=
Se stabilesc cu usurinta relatia
AA0
S
AP
BP
CP
Gergonne
+
+
= 2.
Cu inegalitatea mediilor obtinem
BB 0
CC 0
r AA0
AP BP CP
2 33

, de unde deducem a doua parte a dublei inegalitati din


AA0 BB 0 CC 0
enunt. Pentru prima parte, observ
am mai nti c
a, dac
a P se afl
a n interiorul sau pe
laturile triunghiului M, au loc inegalit
atile S2 +S3 S1 , S3 +S1 S2 si S1 +S2 S3 .
1
1
1
Notnd x =
(S2 + S3 S1 ), y =
(S3 + S1 S2 ), z =
(S1 + S2 S3 ),
2
2
2
t = x + y + z si observnd ca x 0, y 0, z 0 (numai unul poate fi nul),
t > 0, avem:
AP BP CP
(S2 + S3 ) (S3 + S1 ) (S1 + S2 )

=
=
AA0 BB 0 CC 0
S3
t3 + t2 (x + y + z)
2t3
1
(t + x) (t + y) (t + z)
>
=
= .
=
8t3
8t3
8t3
4
Not
a. Solutie corect
a s-a primit de la Marius Pachitariu, elev, Iasi.
Not
a. Aceasta problema apare n articolul "About elementary inequalities in triangle" (M. Dinc
a, M. Bencze) din revista Octogon Math. Magazine, 9 (2001),
no. 1B, p. 472. Aici nu se cere ca punctul P s
a fie n interiorul sau pe laturile triunghiului M, dar solutia prezentat
a este incorect
a.

L37. Fie cercurile C1 , C2 si C astfel nct C1 si C2 sunt tangente exterior n


D, iar cercurile C1 si C2 sunt tangente interior lui C n B, respectiv C. Tangenta
69

comuna interioara cercurilor C1 si C2 taie cercul C n A si A1 , dreapta AB taie C1


1
2
1
n K, iar AC taie C2 n L. Sa se arate ca
=
+
.
DA DA1
KL
Neculai Roman, Mircesti (Iasi)
Solutie. Fie {M } = C1 BC, {N } = C2 BC si T
A
un punct pe tangenta n B la cercurile C1 si C2 .
C
Ar
at
am c
a dreapta KL este tangenta comun
a
exterioara cercurilor C1 si C2 . ntr-adevar, avem
K
\
\
[ ) = m(CAB),
\ deci
m(M
KB) = m(M
BT ) = m(CBT
L
\
\ Cum
M K k CA. Ca urmare, M
KL KLA.
D
\ CBA,
\ deoarece 4KLA 4CBA (fapt ce
KLA
N C
2
M
decurge din AK AB = AL AC = AD ), rezulta ca
C2
\
\ Deci M
\
\
M
KL CBA.
KL M
BA, adic
a KL este
B
C1
tangenta la cercul C1 . Analog se arata ca drepta KL
A1
T
este tangenta la C2 .
Aplic
am teorema lui Casey pentru cercurile C1 , C2 , cercurile degenerate A, A1
tangente interior la C si obtinem relatia AD A1 D + AD A1 D = AA1 KL sau
AA1
2
1
1
2
=
, adic
a
=
+
.
KL
AD A1 D
KL
AD A1 D
L38. Fie 4ABC si punctele D, D0 BC conjugate armonic n raport cu vrfurile B si C. Cercul circumscris 4ADD0 intersecteaza AB n M si AC n N .
b
Aratati ca, daca M N BC, atunci [AD si [AD0 sunt bisectoarele unghiului A

b
(interioara si exterioara) sau m(A) = 90 .
Temistocle Brsan, Iasi
Solutie. Avem M N DD0 M D2 + N D02 = M D02 + N D2 (1). Daca notam
DB
D0 B
a
= 0 = , atunci BD =
, CD =
M
DC
DC
1+
a
a
a
0
0
, BD =
, CD =
(2). Ex=
A
1+
1
1
primnd puterea punctelor B si C fata de cercul
(ADD0 ), vom obtine relatiile: c BM = BD BD0
D
si b CN = CD CD0 sau
B
D
C
2 2
2
a
a
N
si CN =
BM =
. (3)
c (2 1)
b (2 1)
Utiliznd teorema cosinusului n 4BM D, 4CN D0 , 4BM D0 si 4CN D, (1) se scrie

BM 2 + BD2 2BM BD cos B + CN 2 + CD02 2CN CD0 cos C =


= BM 2 + BD02 2BM BD0 cos B + CN 2 + CD2 + 2CN CD cos C

43 a3
4a3
si, tinnd seama de (2) si (3), g
cos B
cos C = 0.
asim 4 2 1 a2 +
c
b
Din nou utiliznd teorema cosinusului, obtinem

2 1 2b2 c2 + 2 b2 a2 + c2 b2 c2 a2 + b2 c2 = 0 sau

b2 a2 b2 c2 2 c2 a2 b2 c2 = 0,
c
a concluzia.
ultima echivalent
a cu = sau a2 = b2 + c2 , de unde rezult
b
70

L39. Determinati toate numerele naturale nenule n pentru care


patrat perfect, unde a, p N .

an (an + 2)
este
p (p + 1)

Mihai Haivas, Iasi


a2 n2 + 2an
2
2

Solutie. Fie
= y , y N . Avem (an + 1) p (p + 1) y 2 = 1, de
p (p + 1)
unde, cu x = an + 1, obtinem ecuatia lui Pell: x2 p (p + 1) y 2 = 1, care are solutia
fundamentala (x0 , y0 ) = (2p + 1, 2) si solutia generala

k
k
p
p
1
xk =
=
x0 + y0 p (p + 1) + x0 y0 p (p + 1)
2

k
k
p
p
1
=
=
2p + 1 + 2 p (p + 1) + 2p + 1 2 p (p + 1)
2

1 p
2k p
2k
=
p+1+ p
+
p+1 p
,
2

k
k
p
p
1
x0 + y0 p (p + 1) x0 y0 p (p + 1)
yk = p
=
2 p (p + 1)

p
1
2k p
2k
p+1+ p

p+1 p
.
= p
2 p (p + 1)
Prin urmare, avem:

1 p
2k p
2k
nk =
p+1+ p
+
p+1 p
2
2a
h
i
2k
2k
care este solutie dac
a 2a |
p+1+ p
+
p+1 p
2 .
Not
a. Solutie corect
a s-a primit de la Marius Pachitariu, elev, Iasi.

L40. Fie A, B Mn (Z) astfel nct det A2 B + AB 2 este impar. Sa se arate


ca A + B este inversabila pentru orice Q.
Marian Urs
arescu, Roman

Solutie. Deoarece det A2 B + AB 2 = det A det (A + B) det B este un numar


impar, rezult
a c
a det A, det (A + B) si det B sunt numere impare. Fie polinomul p (X) = det (A + XB) = det A + a1 X + + an1 X n1 + (det B) X n . Cum
p (1) = det (A + B) = det A + a1 + + an1 + det B este numar impar, nseamna
ca si a1 + a2 + + an1 este numar impar. Sa presupunem acum ca polinomul
p
p are o r
ad
acin
a rational
a = , cu p, q Z, q 6= 0, (p, q) = 1. n acest caz,
q
avem p | det A si q | det B, deci p si q sunt impare. Din p () = 0, rezult
a c
a
n1
n1
n
n
n
+

+
a
p
q
+
(det
B)
p
=
0,
sau
(det
A)
q
+
(det
B)
p
+
(det A) q n + a1 pq
n1

+a1 pn1 q 1 + + an1 pn1 q 1 = (a1 + + an1 ), egalitate care este


fals
a deoarece membrul din stnga este par, iar cel din dreapta este impar. Prin
urmare p () = det (A + B) 6= 0, pentru orice num
ar rational , adic
a matricea
A + B este inversabila oricare ar fi Q.
L41. Demonstrati ca grupul simetric S32 nu are elemente de ordin 2002.
Paul Georgescu si Gabriel Popa, Iasi
Solutie. Presupunem c
a exist
a S32 un element de ordin 2002. Fie =
= 1 2 n descompunerea sa n produs de cicli disjuncti cu ordinele k1 , k2 , . . . ,
71

kn . Avem k1 + k2 + + kn = 32 si [k1 , k2 , . . . , kn ] = 2002. Cum 2002 = 2 7 11 13,


rezulta ca exista ki1 , ki2 , ki3 , ki4 , nu neaparat distincte, astfel nct 2 | ki1 , 7 | ki2 ,
11 | ki3 , 13 | ki4 . Daca ki1 , ki2 , ki3 , ki4 sunt distincte, atunci ki1 +ki2 +ki3 +ki4 33,
ceea ce este fals. Dac
a dou
a, sau mai multe, din cele patru ordine coincid, atunci
ordinul corespunz
ator se divide cu produsul factorilor ce-i corespund, fiind mai mare
sau egal dect produsul acelorasi factori si deci mai mare sau egal dect suma lor.
Astfel, n acest caz obtinem iarasi ca suma ordinelor este mai mare sau egala cu 33,
ceea ce este fals.
L42. Fie (A, +, ) un inel comutativ
si finit, cu celputin 5 elemente

si cu
1 + 1 A inversabil. Fie M = x A | x2 = 1 , I = x A | x2 = x . Sa se
arate ca card M = card I < card A /2.
Ovidiu Munteanu, Brasov
Solutie. Dac
a a A, atunci 21 (1 + a) A si avem: 21 (1 + a) I
2
1

2 (1 + a) = 21 (1 + a) 22 1 + 2a + a2 = 21 (1 + a) 1 + 2a + a2 =
= 2 (1 + a) a2 = 1 a M , de unde rezult
a egalitatea card M = card I.
Sa demonstram acum partea a doua a relatiei date. Daca avem card I = 2, atunci
card A 5 > 2 card I. n continuare ne ocupam de cazul n care card I > 2. n
aceast
a situatie, fie a I \ {0, 1} si atunci 1 a I \ {0, 1, a}. ntr-adev
ar, dac
a
1a = a, rezult
a c
a a = 21 , adic
a a este inversabil si din a2 = a obtinem a = 1, ceea
ce este fals. Avem deci card I > 3. Fie J = {x A | x I} si atunci I J = {0},
pentru ca x I J nseamna x = x = x2 , deci 2x = 0, adica x = 0. Pe de alta
parte, avem I M = {1} si J M = {1}. Cum I, J, M au acelasi num
ar de
elemente, rezult
a c
a are loc card A 3 card I 3 > 2 card I.
L43. Determinati polinoamele P R [X] pentru care P (z) C\R, z C\R.
Gheorghe Iurea, Iasi
Solutie. Polinoamele de gradul 1, P (X) = aX + b (a, b R, a 6= 0) verific
a
ipoteza, deci sunt solutii ale problemei. Aratam ca acestea sunt singurele solutii.
Fie P R [X] cu grad P = n 2 si f : R R, f (x) = P (x) = a0 xn + a1 xn1 +
+ + an (a0 6= 0) functia polinomial
a asociat
a acestuia. Fie a0 > 0 (la fel se va
proceda dac
a a0 < 0). Observ
am c
a m R ecuatia f (x) = m are numai solutii
reale (n solutii), n caz contrar ar exista z C\R si f (z) = m R.
Daca n este par, atunci lim f (x) = . De aici si din continuitatea lui f , rezulta
x

c
a Im f = [m, ), unde m = inf {f (x) ; x R}. Pentru k < m ecuatia f (x) = k nu
are solutii reale, fals.
Dac
a n este impar, avem f 0 (x) = na0 xn1 + , deci lim f 0 (x) = + si
x

f 0 (x) > 0 pentru |x| suficient de mare. Deci f este strict cresc
atoare pe intervalele
(, ) si (, ) (, R convenabil alesi). De aici, din continuitatea functiei
f (deci marginirea ei pe orice interval [, ]) si din faptul ca lim f (x) = +,
x+

deducem ca R astfel nct f () f (x), x (, ]. Ca urmare ecuatiile


f (x) = k, cu k > f () au solutie real
a unic
a, fals.
L44. Fie n 2 numar natural, iar f0 , f1 , f2 , . . . un sir de polinoame definit
prin f0 = (X + 1)n , fp+1 = X fp0 , p 0. Definim nca hp = fp 1p1 fp1 +
X
p1 p1
+ + (1)
p1 f1 , p 1, unde nk =
i1 i2 . . . ik , k {1, 2, . . . , n}
1i1 <<ik n

72

sunt sumele simetrice fundamentale ale numerelor 1, 2, . . . , n. Sa se arate ca


hp = n (n 1) (n p + 1) X p (X + 1)np , p = 1, 2, . . .
Marian Tetiva, Brlad
p
X
Solutie. Sa aratam ca hp+1 = Xh0p php . Avem hp+1 =
(1)k pk fp+1k =
k=0

p
p1
p

X
X
X
k p1
p1
=
(1)k p1
+
p
=
(1)

f
+
(1)k p p1
f
p+1k
p+1k
k
k1
k
k1 fp+1k =
k=0
p1
X

= X

k=0

k=0

0
(1) p1
fpk
p
k

p
X

k=1

k1

(1)

k=1

0
p1
k1 fp+1k = Xhp php (am considerat

n0 = 1 si nk = 0, pentru k < 0 sau k > n).


Demonstratia se poate face prin inductie si se bazeaz
a pe formula stabilit
a. Direct
n1
din enunt, deducem c
a h1 = f1 = Xf00 = nX (X + 1)
. S
a presupunem acum c
a
are loc egalitatea hp = n (n 1) (n p + 1) X p (X + 1)np . n acest caz, putem
scrie:
np
hp+1 = Xh0p php = n (n 1) (n p + 1) pX p (X + 1)
+
+n (n 1) (n p + 1) (n p) X p+1 (X + 1)np+1
pn (n 1) (n p + 1) X p (X + 1)np =

= n (n 1) (n p + 1) (n p) X p+1 (X + 1)
n

np+1

n1

Sa mai observam ca, deoarece hn = n! X , hn+1 = Xn! nX


n n! X n = 0, rezulta
c
a hp = 0, pentru orice p n + 1.
L45. Fie f : [0, ) [0, ) continua. Daca functia F : [0, ) R, F (x) =
Z x
Z 1
=
f (t) dt este marginita, sa se arate ca lim n xf (nx) dx = 0.
n
0
0
Adrian Zanoschi, Iasi
Z 1
Solutie. Dac
a n integrala In =
nxf (nx) dx facem schimbarea de variabil
a
0
Z n
t
nx = t, obtinem In =
f (t) dt. Fie (0, 1). Avem
n
0
Z n
Z n
Z n
Z n
n
n
f (t) dt +
f (t) dt.
(1)
f (t) dt +
f (t) dt =
|In |
n
0
n n
0
n
Deoarece F este m
arginit
a, exist
a M > 0 astfel nct F (x) < M , x R+ (2). Cum
f este continua, rezulta ca F este derivabila si F 0 (x) = f (x) 0, x R+ , deci F
este crescatoare. De aici, avnd n vedere marginirea functiei F , deducem
ca exista
Z
x

lim F (x) si este finita. Prin urmare, lim (F (x) F (x)) = lim

f (t) dt = 0,

de unde rezult
a c
a exist
a n0Z N astfel nct
Z nn n0 are loc:
n

f (t) dt =
f (t) dt < .
(3)

n
nZ n
f (t) dt + < (M + 1), n n0 ,
Din relatiile (1), (2), si (3) obtinem |In | <
0
ceea ce nseamn
a c
a lim In = 0.
n

73

Probleme propuse
Clasele primare
P.64. ntr-o pies
a de teatru sunt 12 personaje, copii si adulti. Cti copii joac
a n
pies
a, dac
a la fiecare doi adulti corespunde un copil?
( Clasa I )
Alexandra Radu, elev
a, Iasi
P.65.

Se dau jetoanele AT

II

CRE

TII

ATII

RECR

EA

RE

REC . Care este num


arul cel mai mare de jetoane cu care se poate forma cuvntul

"RECREATII"?
( Clasa I )
Oxana Pascal, elev
a, Rep. Moldova
P.66. ntr-o livad
a sunt tot attia peri ct si meri. Sunt 6 rnduri cu peri si 4
rnduri cu meri. Numarul merilor de pe un rnd ntrece cu 5 numarul perilor de pe
un rnd. Cti pomi sunt n acea livad
a?
( Clasa II-a)
nv. Maria Racu, Iasi
P.67. Dintr-o multime de 5 copii, orice grupare de trei contine cel putin o fat
a.
Cti b
aieti pot fi n multime?
( Clasa II-a)
Andreea Surugiu, elev
a, Iasi
P.68. Dac
a Ina ar mp
arti num
arul nucilor culese de ea la num
arul nucilor culese
de sora sa, ar obtine 7 rest 6. S
tiind c
a Ina a cules cu 78 nuci mai mult dect sora
sa, aflati cte nuci a cules fiecare.
( Clasa III-a)
nv. Doinita Spnu, Iasi
P.69. ntr-o mpartire cu rest, n care mpartitorul este mai mare ca noua, marind
mpartitorul cu o unitate si efectund din nou mpartirea obtinem ctul 9 si restul
0. Aflati ctul si restul mp
artirii initiale.
( Clasa III-a)
nv. Mariana Toma, Muncelu de Sus (Iasi)
P.70. ntr-o tab
ar
a international
a de matematic
a sunt elevi din patru tari: Bulgaria, Grecia, Republica Moldova si Romnia. Dac
a 21 elevi nu sunt din Bulgaria,
23 nu sunt din Grecia, 22 elevi nu sunt din Republica Moldova si 21 elevi nu sunt
din Romnia, cti elevi sunt din fiecare tara?
( Clasa III-a)
Georgiana Ciobanu, elev
a, Iasi
P.71. Fiecare patrat din figura alaturata se coloreaza cu o alta culoare. n
cte moduri putem face acest lucru avnd la dispozitie patru culori?
( Clasa IV-a)
nv. C
at
alina Rata
, Coarnele Caprei (Iasi)
P.72. Arunc
am dou
a zaruri si adun
am punctele de pe cele dou
a fete de deasupra.
a) Cte sume diferite putem obtine? b) Cte sume se pot forma n trei moduri
diferite?
( Clasa IV-a)
nv. Gheorghe Toma, Muncelu de Sus (Iasi)
P.73. n figura alaturata este pus n evidenta un
B
drum format din sase segmente care pleaca din A si
ajunge n B. Cte drumuri de felul acesta se pot A
construi?
( Clasa IV-a)
nv. Constantin Rata
, Coarnele Caprei (Iasi)
74

Clasa a V-a
V.46. Aflati n N pentru care 11n + 9n si 11n 9n sunt simultan patrate
perfecte.
Andrei - Sorin Cozma, elev, Iasi
V.47. Sa se arate ca numarul 51a51a nu poate fi scris ca produsul a patru numere
prime.
C
at
alin Budeanu, Iasi
9
10
11
V.48. Se consider
a fractiile x1 =
, x2 =
, x3 =
, . . . . Scrieti fractia
14
21
28
ator primele 1000 de fractii.
x1000 si apoi ordonati cresc
Dumitru Gherman, Pascani
V.49. Determinati numarul tripletelor (a, b, c) N3 daca 3a + 2b + c = 598 si
a + 2b + 3c = 602. Dac
a n plus a < b < c, determinati a, b si c.
Gheorghe Iurea, Iasi
V.50. Cte numere de 7 cifre se pot scrie folosind cifrele 1, 2 si 3, astfel nct 1
sa apara de 2 ori, 2 sa apara de 3 ori si 3 sa apara de 2 ori? Dar daca n locul cifrelor
1, 2 si 3 consider
am cifrele 0, 1 si respectiv 2?
Petru Asaftei, Iasi

Clasa a VI-a
VI.46. Suma dintre opusul unui numar natural si inversul altui numar natural
este 119, 992. S
a se determine numerele.
Ciprian Baghiu, Iasi
VI.47. Aflati restul mpartirii numarului N = 28442844 + 41074107 + 63986398
prin 79.
Tamara Culac, Iasi
VI.48. a) ntr-o proportie cu termeni nenuli, un extrem este suma celorlalti
trei termeni daca si numai daca celalalt extrem are inversul egal cu suma inversilor
celorlalti trei termeni.
b) Dac
a din patru numere rationale nenule distincte unul este suma celorlalti trei,
iar altul are inversul egal cu suma inverselor celorlaltor trei, atunci numerele sunt
termeni ai unei proportii.
Claudiu - S
tefan Popa, Iasi
VI.49. Sa se arate ca orice numar natural relativ prim cu 10 admite un multiplu
care se scrie folosind numai cifra 3.
Lucian - Georges L
adunc
a, Iasi
VI. 50. Fie 4ABC cu [AC] [BC], D mijlocul lui [AB], P un punct pe dreapta
AB, iar M si L picioarele perpendicularelor din P pe AC, respectiv BC. Sa se arate
c
a [DM ] [DL].
Neculai Roman, Mircesti (Iasi)

Clasa a VII-a
VII.46. Sa se rezolve n R inecuatiile:
a) x100 + x77 + x50 + x21 + x10 + x5 + 1 > 0;
b) x100 x77 + x50 x21 + x10 x5 + 2 < 0.
Vasile Solcanu, Bogd
anesti (Suceava)
75

VII.47. S
a se rezolve n Z2 ecuatia u2 v + uv 2 = 2u2 + 2v 2 40.
Mihai Cr
aciun, Pascani

VII.48. Dac
a ai = i + i, i = 1, 2004, precizati dac
a num
arul
N = a1 a2 a3 + a4 + a5 a6 a7 + a8 + + a2001 a2002 a2003 + a2004
este negativ, pozitiv sau nul.
Viorel Cornea si Dan S
tefan Marinescu, Hunedoara
VII.49. Fie 4ABC echilateral si D (BC). Not
am cu M1 , M2 mijloacele
segmentelor [BD], respectiv [CD]. Paralela prin M1 la AC intersecteaza AB n F ,
iar paralela prin M2 la AB intersecteaza AC n E. Sa se arate ca dreptele AD, M1 E
si M2 F sunt concurente.
Nicolae Gross si Lucian Tutescu, Craiova
VII. 50. Fie ABCD un trapez cu bazele [AB] si [CD]. O paralela la baze
intersecteaz
a AD, AC, BD si BC n punctele E, F , G si respectiv H. S
a se arate
c
a EH = 3F G dac
a si numai dac
a DF , CG si AB sunt drepte concurente.
Adrian Zanoschi, Iasi

Clasa a VIII-a

m
n
VIII.46. Sa se demonstreze ca nu exista m, n N pentru care
+
= 2003.
n
m
Alexandru Negrescu, elev, Botosani

VIII.47. Pentru x (0, ), s


a se demonstreze inegalitatea

5 3 2 3 2 4 3
x +x +x +1 x +x +2 + x +x +x+1 x3 +x+2 + x3 +x2 +x+1 x2 +x+2
6.
x6 + x5 + x4 + 2x3 + x2 + x + 1
Mircea Cosbuc, elev, Iasi
VIII.48. Gasiti numerele prime p si q pentru care p2 + q = 37q 2 + p.
Liviu Smarandache, Craiova
VIII.49. Fie 4ABC dreptunghic n A cu AB = AC = a. Consideram M A

\
(ABC), M A = a 2 si N AM astfel nct m(CN,
BM ) = 60 . S
a se afle
lungimea segmentului [AN ].
Romanta Ghita
si Ioan Ghita
, Blaj
b = m(C)
b = 90 ,
VIII.50. Fie patrulaterul convex ABCD cu AB = BC, m(A)
b 90 si fie O mijlocul lui [BD]. Pe perpendiculara n O pe planul (ABC) se
m(B)
ia un punct V astfel nct OV = OB. S
a se arate c
a d (D, (V AB)) = 2 d (D, (V AC))
\ = 60 .
dac
a si numai dac
a m(ABC)
Monica Nedelcu, Iasi

Clasa a IX-a
IX.46. S
a se rezolve ecuatia

x 1 + 3 x 2n x 2 = 2, unde n N .
Dan Popescu, Suceava

IX.47. Sa se determine sirul (an )n1 de numere strict pozitive pentru care
a21 a22 + a23 + (1)

n1

a2n = (1)

n1

(a1 + a2 + + an ) , n 1.
Marian Urs
arescu, Roman

76


IX.48. Fie a, b, c (0, ) cu a + b + c + abc = 4. S
a se arate c
a
2
2
2
b
3
a
c
+
+
.
2
b
+
ca
a + bc
c + ab
Cezar Lupu, elev, Constanta
IX.49. Sa se arate ca 4ABC este isoscel n fiecare din ipotezele:
a) 2ma + b = 2mb + a; b) 2ma + a = 2mb + b.
Marius Pachitariu, elev, Iasi
IX.50. Fie I centrul cercului nscris n triunghiul ascutitunghic ABC. Daca A, B,

C sunt masurile n radiani ale unghiurilor triunghiului, iar A IA+B IB+C IC = 0 ,


s
a se arate c
a 4ABC este echilateral.
Constantin Micu, Melinesti (Dolj)

Clasa a X-a
2

X.46. S
a se determine a R astfel nct ecuatia 2x1 + 2x

y 2 + ay + a2
s
a
y 2 + a2

aib
a solutii n Z Z.

Petru R
aducanu, Iasi
X.47. Fie z1 , z2 , z3 C distincte, cu z2 +z3 = 2 si astfel nct |z1 1| = |z2 1| =
= |z3 1|. S
a se arate c
a (z1 z2 ) (
z1 z3 ) este num
ar complex pur imaginar.
Lidia Nicola, Craiova
X.48. Se considera planele paralele si aflate la distanta h unul de celalalt si
4ABC echilateral inclus n planul .
a) S
a se afle locul geometric al punctelor M pentru care M A2 + h2 =
2
= M B + M C2.
b) Sa se determine M astfel nct suma M A2 + M B 2 + M C 2 sa fie minima.
Viorel Cornea si Dan S
tefan Marinescu, Hunedoara
3
3
3
X.49. S
a se arate c
a sin x + sin y + sin z 3 sin x sin y sin z
3
[sin x (1 cos (y z)) + sin y (1 cos (z x)) + sin z (1 cos (x y))] ,
4
x, y, z [0, /3].
Marian Tetiva, Brlad
X.50. Fie ak , bk , ck N, k 1, n; notam cu f (p) numarul tripletelor (A, B, C)
de submultimi (nu neap
arat nevide)
a
Pcu reuniunea
P M = {1,
P 2, . . . , n}, oricare dou
disjuncte si astfel nct num
arul
ai +
bi +
ci p s
a fie multiplu
iM\A
iM\B
iM\C
P
de 3 (convenim ca
xi = 0). Ar
atati c
a dac
a f (0) = f (1) = f (2), atunci exist
ai
i
..
pentru care ai + bi + ci . 3.
Gabriel Dospinescu, student, Bucuresti

Clasa a XI-a
XI.46. Determinati A, B Mn (Z) pentru care det (A + B) = 2 si det (A + 3B) = 5.
Cezar Lupu, elev, Constanta

a
a, dac
ai=j
XI.47. Fie A Mn (R) matrice cu aij =
, unde b 6= 0 si
/ Z.
b, daca i 6= j
b
Ar
atati c
a A este inversabil
a si determinati A1 .
Gheorghe Iurea, Iasi
77

XI.48. Se defineste sirul (xn )n0 prin xn = x2n1 [xn1 ], n 1; x0




0, 1 + 5 /2 . S
a se arate c
a lim xn = 0.
n
aeru, Suceava
C
at
alin Tig

P
XI.49. Fie (xn )n0 , (an )n0 siruri de numere reale astfel nct
an < ,
n=1

a se arate c
a (xn )n0
|xn+1 2xn + xn1 | + |xn+1 3xn + 2xn1 | an , n 1. S
este convergent.
Paul Georgescu si Gabriel Popa, Iasi

nx + y
XI.50. Fie n 2 N, iar f : R R o functie cu proprietatea c
af

n+1

f ( n+1 xn y), x, y R. Sa se arate ca functia este descrescatoare pe (, 0] si


crescatoare pe [0, ). (n legatura cu Problema 2819 din Crux Mathematicorum,
nr. 2/2003.)
Titu Zvonaru, Bucuresti

Clasa a XII-a
XII.46. Sa se determine functia f : R R daca (R, ) este grup abelian
cu proprietatea c
a simetricul oric
arui element x [1, 1] se afl
a n [1, 1], unde
x y = f (x) + f (y), x, y R.
Ioan S
ac
aleanu, Hrl
au
XII.47. Fie G = (a,b), a,b R, iar "" nmultirea numerelor reale. Sa se
determine a, b astfel nct R+ ,
= (G, ) printr-un izomorfism de forma f : R+ G,
x +
f (x) =
, x R+ , cu , , , R.
x +
Alexandru Blaga si Ovidiu Pop, Satu Mare
XII.48. Fie (G, ) grup de element neutru e si x, y G penrtru care avem:
a) k N \ {1} a. . xk = e;
b) p N \ {1} a. i. xy = y p x.
S
a se arate c
a:
1) xy n xk1 = y np , n N ;
2) xy = yx y n(p1) = e, n N .
Mihai Haivas, Iasi
XII.49. Se considera numerele reale b > a 0, c 1 si functiile continue
Z nb
f, g : R+ R+ astfel nct lim
g (x) dx = d R. S
a se arate c
a sirul (un )n1 ,
n na
Z b
1
un =
dx este convergent si sa se afle limita sa.
c
+
f
(x)
+ g (nx)
a
D. M. B
atinetu - Giurgiu, Bucuresti
s (n!)
,
XII.50. Fie s (n) suma cifrelor num
arului natural n. S
a se calculeze lim k
n ln ln n
unde k N este fixat.
Gabriel Dospinescu, student, Bucuresti

78

Probleme pentru preg


atirea concursurilor
A. Nivel gimnazial
G56. Fie m Z, n 2 Z + 1 fixate. Sa se arate ca ecuatia nx + y = m, x, y Z
are o unica solutie (x0 , y0 ) cu proprietatea ca |y0 | < |n| /2.
Petru Asaftei, Iasi
G57. Un seic a l
asat mostenire celor doi fii ai s
ai cinci c
amile, cu conditia ca
unul sa primeasca jumatate, iar celalalt o treime. Mostenitorii nu si-au putut mparti
averea, asa ca au apelat la un ntelept care trecea pe acolo, calare pe o camila. Cum
a procedat nteleptul?
Cte probleme asem
an
atoare mai putem formula (n care mostenirea este de n
camile, iar fiii primesc a p-a si a q-a parte)?
Gabriel Popa, Iasi
2
x
y
G58. S
a se rezolve n N ecuatia 2 + 1 = 5 .
Irina Mustata
a, si Valentina Blendea, Iasi
, elev
G59. Fie A = {n N | s (2000n) + s (2002n) = 2s (2001n)}, unde prin s (x) am
notat suma cifrelor lui x. Demonstrati ca orice numar natural nenul are un multiplu
ce apartine lui A.
Gabriel Dospinescu, student, Bucuresti
G60. Sa se demonstreze ca pentru orice a, b, c (0, ) are loc
ab
bc
ca
1
4abc
2 +
2 +
2 4 + (a + b) (b + c) (c + a) .
(a + b)
(b + c)
(c + a)
Gabriel Dospinescu, student, Bucuresti
G61. S
a se demonstreze c
a pentru orice a, b, c (0, ) are loc
p

(a2 +b2 )(b2 +c2 )(c2 +a2 )


a+b b+c c+a
(a+b)(b+c)(c+a)
54 2
+
+
27
.
c
a
b
abc
abc
Marian Tetiva, Brlad
G62. Fie ABCD un patrulater convex n care se poate nscrie patratul M N P Q
de centru O (M (AB), N (BC), P (CD), Q (AD)). S
a se arate c
a
AB + BC + CD + DA 2 (AO + BO + CO + DO). Cnd are loc egalitatea?
Lucian Tutescu, Craiova si Ioan S
erdean, Or
astie
b = 10 si m(B)
b = 100 construim M (AB) si
G63. n 4ABC cu m(A)
\
\
\
N (AC) astfel ca m(M
CB) = 40 si m(N
a se afle m(AM
N ).
BC) = 75 . S
Octavian Bondoc, Pitesti
G64. Prin punctul P al laturii (AC) a 4ABC se duc paralele la medianele AA0
si CC 0 , care intersecteaz
a laturile (BC) si (AB) n E, respectiv F . Fie {M } =
= EF AA0 , {N } = EF CC 0 , iar L si Q mijloacele segmentelor [F P ], respectiv
[P E]. Sa se arate ca dreptele M L, N Q si A0 C 0 sunt concurente.
Andrei Nedelcu, Iasi
G65. Fie SABCD o piramida cu baza ABCD dreptunghi, M proiectia lui D
pe SB si N proiectia lui C pe SA, iar {P } = AM N B. S
tiind c
a M (SB) si
N (SA), s
a se arate c
a N P SA M B = SM AN P B.
Daniel S
tefan Ninu, elev, Iasi
79

A. Nivel liceal
L56. Fie ABCD patrulater convex si {P } = AB CD, {Q} = AD BC.
Consideram J (AQ), L (BQ), K (DP ), N (AP ) astfel nct QJ = AD,
QL = CB, P K = DC si P N = AB. S
a se arate c
a JL k N K.
Carmen Nejneru, Iasi
L57. Fie 4ABC nscris n cercul C si punctele D (CB, D0 (BC astfel nct
\ Se mai considera cercul C1 tangent la AD, BD si la
\ ABC,
\ BAD
\0 ACB.
CAD
C, cercul C2 tangent la AD0 , CD0 si la C, iar {E} = C1 [BD], {F } = C2 [D0 C]. S
a
se arate c
a cercul circumscris 4AEF si cercul nscris n 4ABC sunt concentrice.
Neculai Roman, Mircesti (Iasi)
L58. Pe muchiile (Ox, (Oy si (Oz ale unui triedru oarecare se consider
a punctele
A, L (Ox, B, M (Oy si C, N (Oz astfel nct OA = OB = OC = a si OL =
\
\
\
= OM = ON = b (a < b). Not
am = m(Oy,
Oz), = m(Oz,
Ox), = m(Ox,
Oy)
si {P } = (AM N ) (BN L) (CLM ), {Q} = (LBC) (M CA) (N AB). Sa se
calculeze distanta P Q n functie de a, b, , , .
Temistocle Brsan, Iasi
L59. Care este probabilitatea ca latura si diagonalele unui romb, luate la ntmplare, s
a fie laturile unui triunghi?
Petru Minut, Iasi
L60. Fie A1 A2 . . . An si B1 B2 . . . Bn (n > 2) dou
a poligoane nscrise n acelasi
cerc de centru O si avnd centrele de greutate tot n O. S
a se arate c
a putem renumerota vrfurile poligonului A1 A2 . . . An pentru a obtine un nou poligon Ai1 Ai2 . . . Ain
n care Aij 6= Bj pentru j {1, 2, . . . , n}.
Gabriel Dospinescu, student, Bucuresti
L61. Fie n 3. Sa se determine maximul expresiei E = x31 x22 + x32 x23 + +
2(n1) 3 3
3 2
x1 x2 x3n , cnd numerele nenegative x1 , x2 , . . . , xn au suma 1.
+xn x1 +(n1)
Gabriel Dospinescu, student, Bucuresti
2
L62. Rezolvati ecuatia 2x = y (y + 1); x, y N.
Mircea Brsan, Iasi
L63. Fie G Mn (R) un grup netrivial n raport cu produsul uzual al matricelor.
Presupunem ca exista X G astfel nct pe fiecare linie, respectiv coloana a sa sa
existe cel mult un element nenul si acesta egal cu 1. Sa se demonstreze ca exista
k {1, 2, . . . n} astfel nct G este izomorf cu un subgrup al lui GLk (R) (s-a notat
GLn (R) = {A Mn (R) | det A 6= 0}).
Ovidiu Munteanu, Brasov
[xn+1 , xn ]
L64. Fie sirul (xn )n1 definit prin: x1 , x2 N , xn+2 =
, n 1. Daca
xn+1
a sirul nu este convergent.
x2003 = 2004, demonstrati c
Iuliana Georgescu si Paul Georgescu, Iasi
L65. Fie n N si functiile f, g : R R, unde f (x) = x2n cos (1/x), x < 0,
f (0) = 0, f (x) = x2n sin (1/x), x > 0, iar g (x) = x2n+1 sin (1/x), x < 0, g (0) = 0
si g (x) = x2n+1 cos (1/x), x > 0. Sa se afle cel mai nalt ordin de derivabiliate al
acestor functii si s
a se studieze problema continuit
atii acestor derivate n origine.
Gheorghe Costovici, Iasi
80

Pagina rezolvitorilor
BOTO
SANI
Colegiul National "A. T. Laurian". Clasa a IX-a. NEGRESCU Alexandru:
VII(39,40,42,44), VIII(36,40,42), IX(36,37,39), X(39,40), G(42,54).
BRA
SOV
Bogdan: V.37, VI (37,38,40),
Scoala
gen. nr. 5. Clasa a VII-a. POSTEUCA

VII.39; POSTEUCA Raluca: V.37, VI (37,38,40), VII.39.

CONSTANTA
Colegiul National "Mircea cel Batrn". Clasa a X-a. LUPU Cezar: VIII.37,
IX(38,39,40), X(38,40), XI(37,38).
CRAIOVA
Angela). STANCIU
Scoala
nr. 22 "M. Eliade". Clasa a IV-a (nv. VANTU

Ioan: P(54-63).
(Iasi)
HRLAU
Liceul "Stefan
cel Mare". Clasa a VI-a. CIOFU Alexandra: P(50,52), V.37,

VI.39, VII.41; SAVA Cristina Amelia: P(52,61,63), V(41,44); SCRIPCARIU Gabriela:


P.61, V(41,44), VI(37,42); SPIRIDON Florin: P(50,61), V(37,41), VII.41; SURUGIU
Ionut: V(37-39), VI(38,39). Clasa a VIII-a. ANTOCI Bogdan: VI(39,40,44,45),
VII.44; BURICAN Bogdan - Alexandru: VI(37,38,42,44,45), VII(41,44); ROTARU
Lucian: VI(39,40,44,45), VII.44.
HUNEDOARA

Liceul "Iancu de Hunedoara". Clasa a VII-a. CRACIUN


Maria: V(43,44),
VI(42,44), VII.41.
IA
SI
Colegiul National "C. Negruzzi". Clasa a V-a. OLARIU Tudor: P(51,52,61-63),
V(43,45), VI.39; TIBA Marius: P(58,61-63), V(42,45), VI.42.
Colegiul National. Clasa a V-a. ANDRONIC Adrian: V(36,38,39,41-45), VI.42;
ANDRONIC Adrian Constantin: V(36,38,39,41-45), VI.42; BALAN Elena-Lavinia:
V(36,38,39,41-44); BARBACARIU Ioana: V(36,38,39,41,42); BERCU Tudor: V(36,
Andreas: V(36,
38,39,41-44); CAPRARU M
ad
alina: V(36,38,41-44); CHELSAU
38,39,40-45); CHIDIU Alexandru: V(36,38,39,41-45); DOBROVICEANU C
at
alina:

V(36,38,39,41-43); GAFITANU
Oana: V(36,39,41-43); GEORGESCU Anca: P(61,
TEANU

62), V(36,38,41,42,44); MNZA


Maria-Adelina: V(36,38,39,41-44); MIHAI
C
Monica: V(36,38,39,41-44); MOSNEGUTU
at
alina Elena: V(36,38,39,41-44); PALAGHIA Irina: V(36,38,39,41-45); POPA Ana-Maria: V(36,38,39,41-43,45); POTUR George: V(36,38,39,41-44); SMARANDA Sava: V(36,38,40-44); TOMA Alexandra: V(36,38,39,41,42). Clasa a IX-a. CAZACU Roxana: VII.41, IX(37,38),
Marta: VII.41, VIII.42, IX(38,39), G.40; HAMCIUC Adrian:
G(40,47); CHIRUTA
VII(41,42), VIII.42, IX(36,39); PRODAN Diana: VII.41, IX(36,38), G(40,47); TIMOFTE Diana: VII.41, IX(36,38), G(40,47). Clasa a X-a. DUMITRESCU Ro
xana: VIII(37,42), IX(37,39), X(36-38,42), G(40,50); PACHITARIU
Marius: G(46 Irina: X.42, XI(41,43), XII.45,
A
50,52), L(46,47,49,50). Clasa a XI-a. MUSTAT
G(46,47,52), L(46,47).
Liceul "M. Eminescu". Clasa a V-a. BOHOTIN Alexandru: P(48,49,51,53,61,62),
81

V.37; COHAL C
alin: P(48,58,60,63), V(38,39,42), VI.38. Clasa a VI-a. CIMPOI
Mihaela: V(42-44), VI(37,42); CIURARU Ionela Alexandra: V(42-44), VI(37,42);
IPATE Cristina Alexandra: V(42-44), VI(37,42).
Liceul "G. Ibraileanu". Clasa a VII-a. UNGUREANU Dragos: V(39,42,45),
VI(39,42).
Scoala
nr. 7 "N. Tonitza". Clasa a II-a (nv. TUDOSE Elena). CRNU Alina:

P(54-57,60); DOBRIN Diana - Maria: P(54-57,60); LEONTE Anca: P(54-57,60);


Simona - Alexandra: P(54-57,60); ROTARIU Larisa: P(54-57,60); SAVIN
POSTICA
R
azvan: P(54-57,60). Clasa a II-a (nv. MELINTE Rodica). BACIU Ciprian: P(5457,60); BRZU Constantin: P(54-57,60); BOTOSANU Bianca - Mihaela: P(54 Danut - Vasilica: P(5457,60); BUZDUGAN Petru - Catalin: P(54-57,60); CEUCA
57,60); CONSTANTINESCU Diana - Gabriela: P(54-57,60); CUCUTEANU Paul C
at
alin: P(54-57,60); GUSOVATE Diana - S
tefana: P(54-57,60); LEOGAN Larisa Diana: P(54-57,60); MIRON Vlad - S
tefan: P(54-57,60); MOTAN Geanina - Diana:
P(54-57,60); ROTARIU Marian: P(54-57,60); SUCIUC Raluca: P(54-57,60); TEIU
- COSTIN Andrada - Mihaela: P(54-57,60). Clasa a IV-a (nv. MARCU Monica).
Alin: P(52,58-62).
A
BUTNARU Valentin: P(52,58-62); ONUT
Scoala
nr. 13 "Alexandru cel Bun". Clasa a III-a (nv. SPNU Doinita).

Marta:
BURLACU Ionut: P(54-57,61); DAMIAN Daniel: P(54-57,61); FURTUNA
P(54-57,61); IFTENIE Ioana - C
at
alina: P(54-57,61); RUSU Alexandru: P(5457,61); URSU Gina - Ioana: P(54-57,61).
A
Scoala
nr. 22 "B. P. Hasdeu". Clasa a II-a (nv. TRZIORU Iuliana). ADASC

LITEI
Victor: P(54-57,60); APOSTOL Ana - Maria: P(54-57,60,61); BALAN An C
drei: P(54-57,60); BURUIANA
at
alina: P(54-57,60,61); CUBERSCHI Paula: P(5457,60,61); ESANU Geogiana: P(54-57,60); GREIEROSU Claudia: P(54-57,60,61);
ATIC

GNDU Alexandra - Livia: P(54-57,60,61); LAM


Ioana: P(47,54-57,60); REBEGEA Andrada: P(54,56,57,60,61); UNGUREANU Teofana: P(54-57,60,61). Clasa a II-a. (nv. TUTU Laura). ANDRONICIUC Ana - Miruna: P(54-57,60,61);
Sebastian: P(54-57,60,61); BUZA
Eduard
BUHU Vlad: P(54-57,60); BURUIANA

- Andrei: P(54-57,60); CEOBANU Andrei - Nicolae: P(54-57,60); CHICHIRAU

Alexandra - Elena: P(54-57,60,61); COSTACHESCU


Ioana: P(54-57,60); DOROHOI Ovidiu: P(54-57,60,61); GELIP Ioana: P(54-57,60); GHERAN Ana - Maria:
Raluca - Claudia: P(54P(54-57,60); GRIGORE Georgiana: P(54-57,60); GURAU
57,60); HATESCU Iustina: P(54-57,60); HORBOVANU Bianca - Alexandra: P(54
57,60); NASTASE
Andrei Ionut: P(54-57,60,61). Clasa a II-a (nv. DOHOTARU
Liliana). TURCU Andrei - Daniel: P(54-57,60,61).
Beatrice). TUDOScoala
nr. 23 "T. Maiorescu". Clasa a IV-a (nv. CHIRILA

RACHE Alexandru - Gabriel: P(54-63).

Scoala
nr. 26 "G. Cosbuc". Clasa a III-a (nv. RACU Maria). BARABULA

Ioana - Mioara: P(54-57,61); BULGARU Ionela - Alexandra: P(54-57,61); BURLA


CU Claudiu: P(54-57,61); CALOIAN Andrei: P(54-57,61); CALIN
Georgiana - An
Adriana: P(54dreea: P(54-57,61); CRACIUN
Madalina: P(54-57,61); IFROSA
Petru - Alexandru: P(54-56,59,61); LEAGAN
Crina - Alexandra:
57,61); IOJA
P(54-57,61); MOISA Bogdan: P(54-57,61); PINTILIE R
azvan - Florin: P(54-57,61);
RUSU Flavia: P(54-57,61). Clasa a III-a (nv. GALIA Paraschiva). ALUPEI An82

Oana - C

dra - M
ad
alina: P(54-57,61); CIOABA
at
alina: P(54-57,61); GHERCA
Marius - Catalin: P(54-57,61); HOMEA Liviu: P(54-57,61); HUIDES Gina: P(54
57,61); MANOLIU Madalina: P(54-57,61); MIHAILESCU
Laura: P(54-57,61); PISI Alexandru: P(54-57,61); POPA Florin: P(54-57,61); SCUTARIU Constantin:
CA
P(54-57,61).

Premii acordate rezolvitorilor


Pentru aparitia de trei ori la rubrica "Pagina rezolvitorilor" redactia revistei
"Recreatii matematice" acord
a cte o diplom
a si un premiu n c
arti urm
atorilor
elevi:
Ioana (Sc. nr. 26 "G. Cosbuc", cl. a III-a): 1/2003 (5pb), 2/2003
BARABULA
(5pb), 1/2004 (5pb);
BURLACU Ionut (Sc. nr. 13 "Alexandru cel Bun", cl. a III-a): 1/2003 (6pb), 2/2003
(5pb), 1/2004 (5pb);
BUTNARU Valentin (Sc. nr. 7 "N. Tonitza", cl. a IV-a): 1/2003 (5pb), 2/2003
(6pb), 1/2004 (6pb);
CALOIAN Andrei (Sc. nr. 26 "G. Cosbuc", cl. a III-a): 1/2003 (5pb), 2/2003 (5pb),
1/2004 (5pb);

CALIN Georgiana (Sc. nr. 26 "G. Cosbuc", cl. a III-a): 1/2003 (5pb), 2/2003 (5pb),
1/2004 (5pb);
Oana - C
CIOABA
at
alina (Sc. nr. 26 "G. Cosbuc", cl. a III-a): 1/2003 (5pb),
2/2003 (5pb), 1/2004 (5pb);

CRACIUN
M
ad
alina (Sc. nr. 26 "G. Cosbuc", cl. a III-a): 1/2003 (5pb), 2/2003
(5pb), 1/2004 (5pb);
DAMIAN Daniel (Sc. nr. 13 "Alexandru cel Bun", cl. a III-a): 1/2003 (6pb), 2/2003
(5pb), 1/2004 (5pb);
Marta (Sc. nr. 13 "Alexandru cel Bun", cl. a III-a): 1/2003 (5pb),
FURTUNA
2/2003 (5pb), 1/2004 (5pb);
IFTENIE Ioana - C
at
alina (Sc. nr. 13 "Alexandru cel Bun", cl. a III-a): 1/2003
(6pb), 2/2003 (5pb), 1/2004 (5pb);

LEAGAN Crina - Alexandra (Sc. nr. 26 "G. Cosbuc", cl. a III-a): 1/2003 (5pb),
2/2003 (5pb), 1/2004 (5pb);

MIHAILESCU
Laura - Ioana (Sc. nr. 26 "G. Cosbuc", cl. a III-a): 1/2003 (5pb),
2/2003 (5pb), 1/2004 (5pb);

MOISA Bogdan (Sc. nr. 26 "G. Cosbuc", cl. a III-a): 1/2003 (5pb), 2/2003 (5pb),
1/2004 (5pb);
NEGRESCU Alexandru (C. N. "A. T. Laurian", Botosani, cl. IX-a): 1/2003 (15pb),
2/2003 (17pb), 1/2004 (14pb);
Alin (Sc. nr. 7 "N. Tonitza", cl. a IV-a): 1/2003 (5pb), 2/2003 (6pb),
A
ONUT
1/2004 (6pb).
PINTILIE R
azvan - Florin (Sc. nr. 26 "G. Cosbuc", cl. a III-a): 1/2003 (5pb),
2/2003 (5pb), 1/2004 (5pb);
83

Bogdan (Sc. nr. 5, Brasov, cl. a VII-a): 1/2002 (5pb), 1/2003 (6pb),
POSTEUCA
1/2004 (5pb).

POSTEUCA Raluca (Sc. nr. 5, Brasov, cl. a VII-a): 1/2002 (5pb), 1/2003 (6pb),
1/2004 (5pb).
RUSU Alexandru (Sc. nr. 13 "Alexandru cel Bun", cl. a III-a): 1/2003 (8pb), 2/2003
(5pb), 1/2004 (5pb);
RUSU Flavia (Sc. nr. 26 "G. Cosbuc", cl. a III-a): 1/2003 (5pb), 2/2003 (5pb),
1/2004 (5pb);
SCUTARU Constantin (Sc. nr. 26 "G. Cosbuc", cl. a III-a): 1/2003 (5pb), 2/2003
(5pb), 1/2004 (5pb);
URSU Gina - Ioana (Sc. nr. 13 "Alexandru cel Bun", cl. a III-a): 1/2003 (7pb),
2/2003 (5pb), 1/2004 (5pb).

LISTA MEMBRILOR FILIALEI IA


SI a S. S. M.
110.
111.
112.
113.
114.
115.
116.
117.
118.
119.
120.
121.
122.
123.
124.
125.

continuare din nr. 1/2000, 1/2001, 1/2002 si 1/2003

MIHAILA Marcela
S
coala "D.D.Patrascanu", Tomesti (Iasi)
BOBOC Romela
S
coala "D.D.P
atr
ascanu", Tomesti (Iasi)
TEMNEANU Mitica
Univ. Tehnica "Gh. Asachi", Iasi
MIRON Mirel
Liceul "C. Negruzzi", Iasi
ROTUNDU Raluca
S
coala gen. Gropnita, jud. Iasi
APETREI Laura
NAVROTESCU Mariana
Gr. sc. "Al. I. Cuza", Iasi
CHIORESCU Daniela Marinela Gr. sc. "D. Mangeron", Iasi

AVADANI
Adela
S
coala gen. nr.37, Iasi
Monica
STRACHINA
S
coala gen. nr.37, Iasi
BRGHISAN Mariana
Gr. sc. "Tehnoton", Iasi
SPIRIDON Ana - M
arioara
S
c. nr. 3 "Iordache Cantacuzino", Pascani
TUDORACHE Nelu
Liceul "V. Alecsandri", Iasi

DASCALU
Cristina
Liceul "M. Eminescu", Iasi
CORDUNEANU Adrian
Univ. Tehnic
a "Gh. Asachi", Iasi
ROSU M
arioara
Liceul de art
a, Iasi

IMPORTANT
n scopul unei leg
aturi rapide cu redactia revistei, pot fi utilizate urm
atoarele adrese
e-mail: tbi@math.tuiasi.ro, popagabriel@go.com . Pe aceasta cale colaboratorii
pot purta cu redactia un dialog privitor la materialele trimise acesteia, procurarea
numerelor revistei etc.
La problemele de tip L se primesc solutii de la orice iubitor de matematici elementare
(indiferent de preocupare profesionala sau vrsta ). Fiecare dintre solutiile acestor
probleme - ce sunt publicate n revista dupa un an - va fi urmata de numele tuturor
celor care au rezolvat-o.
Adres
am cu insistenta
amintea ca materialele trimise revistei s
a nu
rug
fie (s
a nu fi fost) trimise si altor publicatii.
84

S-ar putea să vă placă și